X



俺の英文多読日記【英語の質問していいよ】
■ このスレッドは過去ログ倉庫に格納されています
0001◆iiDSZQODZc
垢版 |
2021/02/16(火) 00:51:33.22ID:OvtHxzaU
Time For Kidsとか読んでいく

俺の知らない表現
能動語彙・表現になっていない表現
発音が不安な表現
知ってるけど受験でポイントとなる表現をまとめていく
0002名無しなのに合格
垢版 |
2021/02/16(火) 00:53:46.61ID:gPj28tEC
まずはこれを読む

commotion/k�mo�n/騒動,騒音
sound pollutionって表現面白いな
-er than everこれまで以上に、今までにないぐらい
marine life/m�ri�n/海洋生物
0004◆iiDSZQODZc
垢版 |
2021/02/16(火) 01:48:42.23ID:UDkFwGAN
over the years長年にわたって長年かけて
have a negative impact on Oに負の影響を与える
sea animals海洋生物
underwater水中で
Sounds travels very far underwater.→
underwaterは条件に近い。水中ならば音はかなり遠くまで伝わるということ
sense Oを感じる
drown Oを溺死させる、水浸しにする[drowned]
drown A outをかき消す圧倒する
flee Oから逃げる
habitats shrink→
habitat生息環境、生息地
shrinkサイズが小さくなる
address O対処する
「対処する」や「取り組む」を意味する動詞は他にもapproach, tackle, wrestle with, grapple with, work onなど様々ある
There are way to address this problem, the study's authors write.
"---", someone says.という形と同じだろう
wind-powered風力の
X-powered: Xを動力源とした
turbineタービン
lower the volumeを下げる
pass a low:法案を通す
regulate Oを規制する
lead author筆頭著者
Why should we not have them for ships?
あまり聞き慣れない表現だが、通常のshouldの意味で、否定疑問文にwhyをつけたものだろう。修辞技法であくまでも意味は「我々はstandardsを持つべき」
stop and think立ち止まって考える
音の進む距離速さについては次のurlを参照
http://www.infonet.co.jp/ueyama/ip/episode/in_water.html#:~:text=%E9%9F%B3%E3%81%AF%E7%A9%BA%E6%B0%97%E4%B8%AD%E3%82%92,%E3%81%AF%2018%2C000m%20%E3%81%A8%E4%BA%91%E3%81%86%E3%80%82
タービンについては
https://ja.m.wikipedia.org/wiki/%E3%82%BF%E3%83%BC%E3%83%93%E3%83%B3
0005名無しなのに合格
垢版 |
2021/02/16(火) 02:16:44.32ID:2Rf0HlmR
単語帳とかやってないの?
0006◆iiDSZQODZc
垢版 |
2021/02/16(火) 02:19:40.60ID:flrWzhgC
次はこれ
https://www.timeforkids.com/g56/social-snakes/
発音記号は打てないから気になったら調べてくれ

scalyウロコに覆われた※2重母音
これは知らなかったがscaleウロコから推測は容易
slitherスルスルと滑るように進む 
slithery滑りやすい、ツルツルとした
恐らくハリーポッターのスリザリンはこれと関連があるのだろう
picture a snake →
picture Oを頭の中に描く、描写する
loner孤独が好きな人、一匹狼※2重母音
lead a study研究を指揮する
search for other snakes to spend time with
不定詞の形容詞的用法のVO関係としていい例文 
simply put簡単に言えば(副詞の塊)
putは「言う」系の意味
seek A outを追求する
social contact社会的接触
findings発見
a growing body of research
これは知らなかった
最初文章におけるbodyの意味かと思ったが
a body of Aで「多数のA」という意味になるので
an increasing number of と同じだろう
ちなみにa growing number ofも当然あるようです
conduct an experiment実験を行う
garter snakeアメリカに分布する蛇の種(特に南東っぽい)
test A for B:AにたいしてBがあるかをテストする
personality trait人柄、人格の特徴
boldness厚かましさ
日本語でも文字に関してボールド体という言葉を使うからわかりやすい
at a time一度に、同時に
occasionally時折
rejoin Oに再び加わる、くっつく
cognition認知能力認識能力
tell A apartを見分ける
be choosy about Oの好みがうるさい
socialize with Oと社交的に付き合う
"---," Skinner adds.
言い足すの意味のaddも重要
表現として使いたい
have nothing to do withwOと関係がない
reason why SV ≒ reason (that) SV
SがVする理由

今日はこんなところかな
また明日
0007◆iiDSZQODZc
垢版 |
2021/02/16(火) 02:25:36.75ID:rcrSt/qf
>>5
やってたから色々持ってるよ
実はもう大学生だから単語帳を毎日眺めて覚えるってのはしてない
今年中に英検一級取るからたまに語彙問題とかやるけどガチるのはもう少し後かな
でも使える語彙や表現を増やすために単語帳を眺めて単語ってより例文をankiに入れることはたまにある

例えばa familiar voiceってのを見たら分かるし、少し考えたら日本語からこの表現も出せるだろうけど
一瞬でこういうのが口をついて出るようにしたい
0008◆iiDSZQODZc
垢版 |
2021/02/16(火) 02:35:42.29ID:rcrSt/qf
安井稔博士の言葉で耳が痛くなる

「否定疑間文を用いるのは,日本人には離しいものであるらしく,日本人の英会話には,奇妙に欠けているものである,もっと活用すべきものであると考えられる.」(安井稔:英文法総覧より)

ちなみに否定疑問文では、Vn't SとV S notの2つがありえる
と丁寧に書かれている
0009名無しなのに合格
垢版 |
2021/02/16(火) 02:43:49.90ID:L8cOHkwU
共通テストレベルの英文が時間内に読み終わらないんだけど、速読ってどうやって勉強するん?
0010◆iiDSZQODZc
垢版 |
2021/02/16(火) 03:29:08.92ID:rcrSt/qf
>>9
音読する以上のスピードで読むことは基本無理だから速く音読する練習は有効らしい(熟達したら別だろうけど)
論理展開を把握するのも有効だけど共通テスト的なのだと微妙そう

ダークソウルのプロローグ読んでみた。暗唱したくなる英文だね
・In the Age of Ancients,
The world was unformed, shrouded by fog
unformed未熟な、発展途上の
be shrouded 包まれる隠れる(in/by〜に)
・A land of grey crags, archtrees, and everlasting dragons
crag(巨大で切り立った)ごつごつした岩; 絶壁
arch-名詞につけて:主要な、最高位の
everlasting不滅の
・But then there was Fire
And with Fire came Disparity. Heat and cold, life and death, and of course... Light and Dark.
disparity不均衡、差異
ここではMVSの倒置
2つセットのものが並べられているから
A and B, C and D, E and Fの形
・Then, from the Dark, They came
And found the Souls of Lords within the flame.
Nito, the first of the dead
the dead死者
・The Witch of Izalith, and her daughters of chaos
chaos混沌
・Gwyn, the Lord of Sunlight, and his faithful knights
faithful忠実な
faith信仰、誠意
・And the furtive pygmy, so easily forgotten
furtiveコソコソした、人目を盗んだ
pygmy小人、小物、取るに足らない人
・With the Strength of Lords, they challenged the dragons.
challenge Oに挑む
・Gwyn's mighty bolts peeled apart their stone scales
mighty強大なbolt稲妻、大矢
peel A apartを剥ぐ
調理器具のピーラーですね
scale鱗
・The witches weaved great firestorms
weave Oを編む
・Nito unleashed a miasma of death and disease
unleash Oを引き起こす、もたらす、解き放つa miasma瘴気、重苦しい雰囲気
・And Seath the Scaleless betrayed his own, and the dragons were no more
人名 the 名詞/形容詞 のよくある同格関係
耳なし芳一 Hoichi the Earlessアレキサンダー大王Alexander the GreatくまのプーさんWinnie the Pooh切り裂きジャックJack the Ripper
betray Oを裏切る
・there were no more dragons
これ以上ドラゴンは生まれないってことかな
・Thus began the Age of Fire
But soon, the flames will fade, and only Dark will remain
fade消える衰える薄らぐ
・Even now, there are only embers, and man sees not light, but only endless nights
seesってあまり見ないよね
ember残り火、燃えさし
・And amongst the living are seen, carriers of the accursed Darksign.
amongstはamongのイキった言い方
carrier運ぶ人
ofでcarryの意味上の目的語を表す
accursed呪われた、忌々しい
M be Vpp Sの受動態+倒置
0011◆iiDSZQODZc
垢版 |
2021/02/16(火) 03:29:34.05ID:rcrSt/qf
公式?の日本語ver
古い時代
世界はまだ分かたれず、霧に覆われ
灰色の岩と大樹と、朽ちぬ古竜ばかりがあった

だが、いつかはじめての火がおこり
火と共に差異がもたらされた
熱と冷たさと、生と死と、そして光と闇と

そして、闇より生まれた幾匹かが
火に惹かれ、王のソウルを見出した

最初の死者、ニト
イザリスの魔女と、混沌の娘たち
太陽の光の王グウィンと、彼の騎士たち
そして、誰も知らぬ小人

それらは王の力を得、古竜に戦いを挑んだ

グウィンの雷が、岩のウロコを貫き
魔女の炎は嵐となり
死の瘴気がニトによって解き放たれた

そして、ウロコのない白竜、シースの裏切りにより、遂に古竜は敗れた
火の時代のはじまりだ


だが、やがて火は消え、暗闇だけが残る

今や、火はまさに消えかけ
人の世には届かず、夜ばかりが続き
人の中に、呪われたダークリングが現れはじめていた…
0013◆iiDSZQODZc
垢版 |
2021/02/16(火) 03:58:21.80ID:rcrSt/qf
スレってどれぐらい持つかな
落ちないように暇な人いたらテキトーに書き込んでくれたら喜ぶ
質問でもしりとりでもなんでもいいよ
0014◆iiDSZQODZc
垢版 |
2021/02/16(火) 04:26:40.21ID:rcrSt/qf
in an unusual mannerこれはフレーズで覚えたいね
mutilate Oをバラバラに切断する
substantial十分な、しっかりした に関してはここでの a substantial knowledgeだけじゃなくてsubstantial evidenceもフレーズとして覚えたい
anatomy解剖学 TOEFLで覚えたけど「グレイズ・アナトミー」って海外ドラマの名前聞いたことある人もいそう
culprit犯人
Jack the Ripper remains one of England’s, and the world’s, most infamous criminals.
こういう英文が書けるかどうか
killing殺人
be attribute to Oに原因がある、Oに帰属するOの作品である
All five killings attributed to Jack the Ripper took place within a mile of each other
このwithinの使い方は英文解釈教室でも出てきたな
スラスラと出るほど使えるようにはなってなさそう
Several other murders occurring around that time period have also been investigated as the work of “Leather Apron” (another nickname given to the murderer).
このaroundはなかなか慣れていない
workしわざ
The crime was the work of outsiders.(wisdom英和より)
allegedly伝えられるところでは、申し立てによると
taunt Oを挑発する
gruesome恐ろしい
-someは形容詞が多いみたいだね
でもthreesome:3Pが最初に思い出される
speculate on Oについて推測する
A to come :まだこれから起こるはずのA
monikerあだ名
originate from Oに由来する
hoax偽装、でっちあげ
at the time of ーの時に
publish Oを公開する

ここまでで本当に終わる
明日続き読む
0016名無しなのに合格
垢版 |
2021/02/16(火) 06:41:20.89ID:dZUYqx5p
ちょこちょこレベル低い単語あるけど英語力はどれくらいなの? 英検とかTOEICとか
0017◆iiDSZQODZc
垢版 |
2021/02/16(火) 06:49:59.11ID:rcrSt/qf
>>16
覚えてるのも書いてるよ
比率でいうと初見の単語かなり少ない(ダクソは除く)
0018◆iiDSZQODZc
垢版 |
2021/02/16(火) 06:50:41.26ID:rcrSt/qf
900は超えてる
英検は準2級1次通って二次サボって以来まだ受けたことない
0019名無しなのに合格
垢版 |
2021/02/16(火) 09:14:47.78ID:hRsIKN7T
If you're going to do this on a forum, why don't you have a conversation using those words instead of just listing them?
0020◆iiDSZQODZc
垢版 |
2021/02/16(火) 10:03:48.18ID:V/TS9XtG
>>19
あくまでここは多読にしたいな
読むのなんてめんどくさいしめんどくさくなる要因を増やしたくない
0022◆iiDSZQODZc
垢版 |
2021/02/16(火) 10:31:25.59ID:V/TS9XtG
>>21
多読めんどくさい
そこから出てくる単語で英文作るまでやると
更にめんどくさくなる
0023◆iiDSZQODZc
垢版 |
2021/02/16(火) 10:57:13.58ID:V/TS9XtG
ではhttps://www.google.com/amp/s/www.history.com/.amp/topics/british-history/jack-the-ripper
Despite countless investigations claiming definitive evidence of the brutal killer’s identity, his or her name and motive are still unknown.から
brutalは容赦ない残酷な ですね
definitive evidence決定的な証拠 これもフレーズで覚えたい substantial evidenceと一緒に
evidence of one's identity招待に関する証拠
motive, motivation動機
このクレイムは主張するでいいかな
He claimed the land as his own. みたいに

Various theories about Jack the Ripper’s identity have been produced over the past several decades, which include claims accusing the famous Victorian painter Walter Sickert, a Polish migrant and even the grandson of Queen Victoria. Since 1888, more than 100 suspects have been named, contributing to widespread folklore and ghoulish entertainment surrounding the mystery.
accuse Oを責める
a Polish migrantポーランド人移民
a suspect容疑者
name Oの名前を挙げる
contribute to Oに寄与する
folklore民間伝承
ghoulish悪趣味な悪鬼のような※グール
contributing以降に関しては主節全体を主語とするタイプの分詞構文と捉えたほうが筋が通りますね

とりあえずここまで
お出かけします
0024◆iiDSZQODZc
垢版 |
2021/02/16(火) 11:03:33.28ID:V/TS9XtG
主節全体が主語となるタイプはそもそも
, leading toみたいな因果関係を表すのが典型ですね
0025◆iiDSZQODZc
垢版 |
2021/02/16(火) 11:52:36.24ID:V/TS9XtG
電車の中で

The ‘Whitechapel Butcher’
In the late 1800s, London’s East End was a place that was viewed by citizens with either compassion or utter contempt. Despite being an area where skilled immigrantsmainly Jews and Russians�came to begin a new life and start businesses, the district was notorious for squalor, violence and crime.

butcherは肉屋とか人殺しです
アブドーラザブッチャーとか耳に馴染んでますね(よく知らないけど)
compassion哀れみ
contempt軽蔑
where節でSVの間に挿入があるが主述をしっかりと把握
Jewユダヤ人
start in businessならば商売を始める
だが今回は複数形であり、可算名詞であることに注意
会社を始めるの意味
squalorみすぼらしさ
infamousとnotoriousの両方が載ってるね
infamousは発音に注意
notoriousといえばThe Notorious BIG(伝説のラッパー)
0026◆iiDSZQODZc
垢版 |
2021/02/16(火) 11:56:55.12ID:V/TS9XtG
immigrantsmanilyは
immigrants ― mainely
で話してダッシュ挿入して
cameの前の文字化けもダッシュ
0027◆iiDSZQODZc
垢版 |
2021/02/16(火) 16:08:03.69ID:V/TS9XtG
Prostitution was only illegal if the practice caused a public disturbance, and thousands of brothels and low-rent lodging houses provided sexual services during the late 19th century.
prostitution売春 この単語を見て思い出すのはprostate前立腺ですねpro前にstate立つ
practice行為、実践、やり方、営業
disturbance騒動
only ifが分離してonly 形容詞 if になっていることに注意
brothel売春宿
lodging宿


At that time, the death or murder of a working girl was rarely reported in the press or discussed within polite society. The reality was that “ladies of the night” were subject to physical attacks, which sometimes resulted in death.

構文としてほんの少し面白いのでは?
名詞構文でof a working girlがdeathにとっては主語,murderにとっては目的語を表している
the Press報道機関(しばしば大文字)
be subject to Aにかかりやすい
このsubjectは形容詞(無冠詞で複数形でもない)
physical attacks肉体攻撃
lady of the night [名] 《C》 英 やや古
夜の女 ((売春婦の婉曲(えんきょく) 的な表現

Among these common violent crimes was the attack of English prostitute Emma Smith, who was beaten and raped with an object by four men. Smith, who later died of peritonitis, is remembered as one of many unfortunate female victims who were killed by gangs demanding protection money.

prostitute売春婦
beat Oを殴る
die of Oが原因で死ぬ
peritonitis腹膜炎 
protection moneyみかじめ料
with an object of Oの目的でっていう表現も浮かぶが恐らく違う
なぜbeaten with an object and rapedではないのだろうと思った
TIMEの記事に次の英文があったのでobjectはrapeの道具で(も)あったのだろう
ただそれが脅しなのか挿入の道具なのかはわからない
次の英文では挿入の道具だろう
after her nine-year-old daughter was gang-raped with a beer bottle by three teen-age girls and a boy.
Smithは腹膜炎でなくなっているが、道具などを無理に入れられたのかもしれない
0028◆iiDSZQODZc
垢版 |
2021/02/16(火) 16:22:15.05ID:V/TS9XtG
あ、最後のパラグラフの最初はMVSの倒置ね
書いてなかった
0029名無しなのに合格
垢版 |
2021/02/16(火) 18:55:47.72ID:sjDpYB8R
>>13
自分も少し前に「センター英語満点だったけど質問ある?」みたいなスレを受サロに立てたことあるけど、
3日ぐらい放置したらスレ落ちしちゃった

3月前半ぐらいまでは毎日か、もしくは2日に1回ぐらいのペースで何かしら書き込まないとスレ落ちしちゃうかもね
3月の半ばを過ぎると受験生がいなくなるから、比較的ゆったりできるようになるし、スレ落ちの心配もしなくてよくなる

自分も3月半ばぐらいに、またスレ立ててみるつもり
0031◆iiDSZQODZc
垢版 |
2021/02/16(火) 20:05:57.85ID:T5nuWoej
そろそろ読もうかな
読んでる英文のことでも関係ないことでも質問してくれたらできる限り答えるよ
0032◆iiDSZQODZc
垢版 |
2021/02/16(火) 20:26:50.21ID:T5nuWoej
However, the series of killings that began in August 1888 stood out from other violent crime of the time: Marked by sadistic butchery, they suggested a mind more sociopathic and hateful than most citizens could comprehend.
marked by Oによって特徴付けられている
theyはkillingsのこと(the series of killingsではない、a series of Aは通例単数扱い)
markedはtheyを主語にする分詞構文であり理由を表している
sadistic加虐的な残酷なbutchery虐殺
suggest Oを連想させる
この場合は人々、つまりcitizenに連想させるだろう
sociopathic社会病質者の
hateful憎しみを持つ、とても嫌な
mind気質
名詞を修飾するmore...が名詞の後ろに回っている形
more sociopathic and hatefulという形容詞句が長いからという理由だろう
comprehend Oを理解する
つまり(直訳ですまんな)
残酷な殺戮により特徴づけられていたため、それらの殺人は市民には理解できないほどに不愉快で社会病質的な気質を連想させた。
0033◆iiDSZQODZc
垢版 |
2021/02/16(火) 20:28:41.40ID:T5nuWoej
めっちゃ眠いな
毎日丁寧に読んで結果的にたくさん読みたいだけだから勘弁してね
多読じゃないとかは言わないで
0034◆iiDSZQODZc
垢版 |
2021/02/16(火) 23:26:35.84ID:0oasR2DA
Jack the Ripper didn’t just snuff out life with a knife, he mutilated and disemboweled women, removing organs such as kidneys and utereses, and his crimes seemed to portray an abhorrence for the entire female gender.
snuff A outを消滅させる
disembowel Oの内臓を引き抜く
bowel腸
dis-分離-em-≒-ex-外に
kidney腎臓
uterus子宮※恐らく本文はタイポ
portray Oを描く
adhorrence of Oに対する憎悪
0035◆iiDSZQODZc
垢版 |
2021/02/17(水) 00:13:22.39ID:QSy4PfS7
The Legacy of Jack the Ripper
legacy 遺産、異物
Jack the Ripper’s murders suddenly stopped in the fall of 1888, but London citizens continued to demand answers that would not come, even more than a century later. The ongoing casewhich has spawned an industry of books, films, TV series and historical tourshas met with a number of hindrances, including lack of evidence, a gamut of misinformation and false testimony, and tight regulations by the Scotland Yard.
…answers that would not come, even more than a century laterは100年以上経っても得られない回答 の意味
ongoing進展中
spawn Oをもたらす(マイクラのおかけで小学生でも分かる)
the Shakespeare industryシェイクスピア産業
みたいなものっぽいね
meet with O(賛否や反応)を受ける
hindrance障害妨害 これも動詞は簡単だから推測は容易だけどパッと出せるか
gamut(全)範囲、音域、色域
misinformation(意図的に流された)誤報
testimony証言
falseうその
a gamut ofはmisinformationとfalse testimonyの両方に掛かることが意味と形から分かる
tight regulations厳しい規制
the Scotland Yardロンドン警視庁
ここで一つ問題が発生
この英文はA, including B, C and D, and Eの形
B,C and Dとなっていることからincludingの句はDまででAとEが並列が普通、しかしここではa gamutがCとDの両方に掛かっているためカンマではなくandになっており、BからEまで塊であると判断することも可能。このようなことはsupply and demandのようなペアで1つの意味を成すような場合でも起きる。
今回の英文では意味の上で考える
一見どっちでもよさそうだが複数形が並ぶのは形が良い。また「証拠不足、誤った証言や情報」と「警察による規制」はかなり種類が違う
証拠不足、誤った証拠や情報などを含む妨害

警察による規制
とするのが筋がいいかな
規制は妨害なのかという話も出てくる
ちなみにdeep L翻訳してみると
「証拠の不足、誤報や虚偽の証言の範囲の広さ、スコットランドヤードによる厳しい規制など、多くの障害に直面しています。」
includingはregulationsまで含んでいると解釈されている
だが信用しない

書き込むスレ間違えた
0036◆iiDSZQODZc
垢版 |
2021/02/17(水) 01:10:56.37ID:Xs9whPNm
Jack the Ripper has been the topic of news stories for more than 120 years, and will likely continue to be for decades to come.
これは無問題
More recently, in 2011, British detective Trevor Marriott, who has long been investigating the Jack the Ripper murders, made headlines when he was denied access to uncensored documents surrounding the case by the Metropolitan Police.
make headlines新聞で大見出しとして取り上げられる
uncensored検閲されていない、検閲で削除されていない
deny O1にO2を与えない使わせない阻む
これが受動態に
access to Oへの利用権接近出入り
the Metropolitan Policeロンドン警視庁
According to a 2011 ABC News article, London officers had refused to give Marriott the files because they include protected information about police informants, and that handing over the documents could impede on the possibility of future testimony by modern-day informants.
protected informationだけだと辞書で出てこないがGuardianやNYTなどでもよく出ているようだからフレーズとして覚えよう
confidential informationに近いかな
informant情報提供者、密告人
handle over Oを引き渡す
impede Oを妨害する

なにこのthat
becauseと同じように理由を表してるよね?
こんな用法ある?
昔のbecause thatっていうひとかたまりの接続詞は関係ないだろけど 
so thatのso省略は意味的におかしいし
これでいいかな?
(wisdomより)
名詞節をつくる
that …すなわち(つまり)…ということだ
しばしば(セミ)コロンのあとで
The terms present tense and past tense have this justification: that the tenses they name typically have reference to present and past time respectively.
わからん
0037◆iiDSZQODZc
垢版 |
2021/02/17(水) 01:56:42.12ID:Xs9whPNm
わかる人いたらおしえて
0039◆iiDSZQODZc
垢版 |
2021/02/17(水) 07:08:49.82ID:Xs9whPNm
Alexander the Great, a Macedonian king, conquered the eastern Mediterranean, Egypt, the Middle East, and parts of Asia in a remarkably short period of time. His empire ushered in significant cultural changes in the lands he conquered and changed the course of the region’s history.
Macedonianマケドニア王国のこと
マケドニア:バルカン半島中央部の地域
Alexander the Great, a Macedonian kingは同格の非制限用法
名詞, 名詞の同格におけるカンマの有無は関係詞のそれと基本的に同じ
ただAlexander the Greatではカンマ入れないし、こういったこと関しては自分の中での正確な理解はない
いつか詰める
conquer Oを支配する
the Mediterranean Sea地中海
a period of time期間
まあここの前置詞句は一瞬でってことですね
usher A in をもたらす、の到来を告げる
the course of history 歴史の流れ、推移、経過、成り行き

Alexander the Great, also known as Alexander III or Alexander of Macedonia is known as one of the greatest generals in all history.
in history 歴史上で、史上 [過去]において
の強いバージョンかな
general大将、将軍

Alexander was born in 356 B.C.E. in Pella, Macedonia, to King Philip II. As a young boy, Alexander was taught to read, write, and play the lyre. He developed a life-long love of reading and music. When Alexander was a teenager, his father hired Aristotle to be his private tutor. He studied with Aristotle for three years and from Aristotle’s teachings, Alexander developed a love of science, particularly of medicine and botany. Alexander included botanists and scientists in his army to study the lands he conquered.
B.C.がbefore Christで紀元前なのは知ってたがB.C.E.って表現でも同じことを表せれるのねbefore the Common Eraの意味でキリスト教的要素を抜くための表現らしいね
https://ja.m.wikipedia.org/wiki/%E7%B4%80%E5%85%83%E5%89%8D
Pellaは知らなかったけどウィキが参考になる
Pella (Greek: Πλλα) is an ancient city located in Central Macedonia, Greece, best known as the historical capital of the ancient kingdom of Macedon and birthplace of Alexander the Great.
https://en.m.wikipedia.org/wiki/Pella_(disambiguation)
ここのto King Philip Uを「フェリペ2世へ」としても意味がわからない
born に対してin -とto -の両方が掛かっている
be born to Oのもとに生まれる〜の家に生まれるの
この表現を知っていたらin-に惑わされず離れた修飾を見抜かなければならないし、知らなかったとしても「〜へ」と訳すだけでは意味がわからないと気づいて意味が通るように解釈できるのが学力
lyre古代ギリシャの竪琴
an ancient musical instrument consisting of a U-shaped frame with strings attached to it
life-long一生の
develop (感情などを)持つようになる
hire O to V:Oを雇ってVさせる
private tutor家庭教師
teachings(権威者の)教え、教訓[しばしば-s] 
medicine医学
botany植物学 ボタニカルですね
ちなみにzoologyは動物学
include A in B:AをBに入れる
0040◆iiDSZQODZc
垢版 |
2021/02/17(水) 07:49:43.84ID:Xs9whPNm
In 336 B.C.E., at age 20, Alexander became king of Macedonia when a political rival assassinated his father. Alexander began his reign by subduing rivals in the Greek and Macedonian regions. At a council of the League of Corinth, he was chosen as the commander of a military invasion of Asia. King Alexander began his invasion of the Middle East in 334 B.C.E. He spent most of his reign on a military campaign through northeast Africa and southwestern Asia.
年齢は色々の表現を覚えないとね
at the age of 〜≒at age 〜という年齢で[の]
a nine-year-old boy≒a boy of 9
She is twelve years old
(かたい)She is twelve years of age
a political rival政敵
assassinate Oを暗殺する
reign治世、統治、支配
subdue Oを征服する(これは鉄壁にもあったね)
the League of Corinthコリントス同盟
https://ja.m.wikipedia.org/wiki/%E3%82%B3%E3%83%AA%E3%83%B3%E3%83%88%E3%82%B9%E5%90%8C%E7%9B%9F#:~:text=%E3%82%B3%E3%83%AA%E3%83%B3%E3%83%88%E3%82%B9%E5%90%8C%E7%9B%9F%EF%BC%88%E3%82%B3%E3%83%AA%E3%83%B3%E3%83%88%E3%82%B9%E3%81%A9%E3%81%86%E3%82%81%E3%81%84,%E5%85%A8%E3%83%9D%E3%83%AA%E3%82%B9%E3%81%8C%E5%8A%A0%E7%9B%9F%E3%81%97%E3%81%9F%E3%80%82
choose A as B ≒ choose O to be C
OをCに選ぶ
be chosen Xに選ばれる 受け身ではしばしばasもto beも入れない
commander司令官
invasion侵略
military軍事の
spend A on[in] Bに費やす
この場合のreignを単に統治と解して「統治を費やす」としては意味がわからない
reign統治期間
a campaign軍事行動

Alexander built many new cities in the lands he conquered, including Alexandria in Egypt. He went on to conquer the lands of the Persian Empire, establishing more cities, and like Alexandria, often naming them after himself. His conquest continued through Asia until he reached the shores of the Ganga (Ganges) River in India. At this point, his army refused to continue further into India, exhausted and discouraged by heavy rains.
go on ingし続ける
go on to V(なにかの終了後)引き続きVする
Persianペルシアの
after 〜に因んで
name A after BにちなんでAに名付ける
この文は分詞構文のestablishingとnamingがandで並列されている。and like Alexandria, oftenのように修飾語があるが惑わされない
through〜まで
the Ganga river≒ the Ganges riverガンジス川
at this pointこの時点で、現時点で
exhausted以降は理由を表す分詞構文
exhaust Oを疲れ果てさせる
heavy rains土砂降りの大雨(1回分)
congratulationsやthanks
the waters海、湖、川、洪水、大量の水
こういったものを強意複数と呼ぶ
この場合のheavy rainsは一回分と限定されているかはわからない
ただ強意ではあるだろう

Alexander was 32 when he died in 323 B.C.E.

During his 13-year reign as the king of Macedonia, Alexander created one of the largest empires of the ancient world, stretching from Greece to northwestern India.
stretch from A to B:AからBまでに及ぶ
このstretchingは分詞構文というよりempiresに掛かる分詞の非制限用法
分詞構文であれば主語が主節のそれと一致しないため懸垂分詞構文ということになる
どっちで解釈してもさほど変わらない。
0042名無しなのに合格
垢版 |
2021/02/17(水) 08:21:15.59ID:lKd3WqYq
>>36
thatで理由を表す用法あるよ
now thatやin thatの形ならよく見るだろ?
0043◆iiDSZQODZc
垢版 |
2021/02/17(水) 08:29:08.81ID:Xs9whPNm
>>42
それはわかる
ただ辞書でこれ!と指摘できるやつある?
文法書はたくさんあるけど辞書は一冊ぐらいしか持ってないからうまく対応できない
昔becauseがbecause thatの形で使ってたし(関係あるかわからんが)
because -- and because --
の後ろをthatにしたver

あるいは先の書き込みで挙げた
すなわち、つまり

どちらかの意味だとは思うけど
0044◆iiDSZQODZc
垢版 |
2021/02/17(水) 08:37:40.88ID:Xs9whPNm
他で質問して回答もらった
流石色んな意味をもつ接続詞のthatって感じ

that (as a conjunction) についての OED Second Edition による解説
8. Used (like French que) as a substitute instead of repeating a previous conjunction, or conjunctive adverb or phrase. Now rare or arch.

用例のうちの一部
(1) 1700 J. Tyrrell Gen. Hist. Eng. II. 823
★So soon as★ the Death of King John was..known, and ●that● the Earls..could agree where to meet.

(2) 1797 E. Burke Lett. Peace Regic. France iii, in Wks. (1815) VIII. 330
★When★ one of the parties to a treaty intrenches himself..in..ceremonies,..and ●that● all the concessions are upon one side.

(3) 1829 W. F. Napier Hist. War Peninsula (Rtldg.) II. ix. iii. 16
★Although★ the rear was attacked,..and ●that● 50 men..were captured.

これで確定っぽいね
もう少しこの用法について調べてみる
0045◆iiDSZQODZc
垢版 |
2021/02/17(水) 08:51:40.64ID:Xs9whPNm
ここで堀田さんのブログが出てくるとは思わなかった
http://user.keio.ac.jp/~rhotta/hellog/2015-08-29-1.html
これで解決ですね
0046◆iiDSZQODZc
垢版 |
2021/02/17(水) 08:55:15.76ID:Xs9whPNm
because thatも関連があった
if that, when thatみたいにthatは汎用の従属節マーカーであった
このことは知ってたけど
substituteとしてのこの機能は知らなかったとしても
0047名無しなのに合格
垢版 |
2021/02/17(水) 09:01:57.73ID:Gk5qVIz6
つまりbecause that … and because that …から
becauseの繰り返しを避けてbecause that … and that …
thatは念の為のマーカーだから省いてbecause … and that …
面白いな
0048◆iiDSZQODZc
垢版 |
2021/02/17(水) 09:20:44.97ID:Xs9whPNm
>>47
ね!
こんなのに出会うなんて運が良かった
becauseの意味を表しているのはわかってたけどそれをまさしくこの用法と指摘してくれる人が現れたのが奇跡的
0050名無しなのに合格
垢版 |
2021/02/17(水) 13:42:21.89ID:Myh1aEjL
Free care, coward to become miss note
0051◆iiDSZQODZc
垢版 |
2021/02/17(水) 13:59:06.73ID:WHEJhgXb
やっぱり次は科学系の文章読みます
0052◆iiDSZQODZc
垢版 |
2021/02/17(水) 14:44:37.24ID:WHEJhgXb
>>36
impede は他動詞のはずだけどLudwigでimpede onの例が出てくる
0053◆iiDSZQODZc
垢版 |
2021/02/17(水) 16:31:37.06ID:WHEJhgXb
とりあえずバイトで高校生に英語教えてくる
終わったら面白そうな記事探すね
0054◆iiDSZQODZc
垢版 |
2021/02/17(水) 17:51:46.31ID:pNOwDKWo
次の英文決定
規範文法に関するwikipediaを見ます
murrayとか出てくるかな
俺の規範主義に関する前提知識はここで述べられているもの程度

http://www.tufs.ac.jp/ts/personal/ykawa/jafle.files/workshop_no5.files/urata.htm
http://user.keio.ac.jp/~rhotta/hellog/2011-05-14-1.html

https://www.google.com/url?sa=t&;source=web&rct=j&url=https://www.toyo.ac.jp/uploaded/attachment/8348.pdf&;ved=2ahUKEwj-qOjTw_DuAhURrZQKHRydBiEQFjAAegQIARAB&usg=AOvVaw0kltxqt5FEj_gyiMI0wF1f
https://www.google.com/url?sa=t&;source=web&rct=j&url=https://toyo.repo.nii.ac.jp/%3Faction%3Drepository_action_common_download%26item_id%3D5177%26item_no%3D1%26attribute_id%3D22%26file_no%3D1&;ved=2ahUKEwj-qOjTw_DuAhURrZQKHRydBiEQFjADegQICRAB&usg=AOvVaw0qYhIeiGytAnh_7Yx4ydW5
https://www.google.com/url?sa=t&;source=web&rct=j&url=http://jaell.org/gakkaishi25th/shibusawa.pdf&;ved=2ahUKEwj-qOjTw_DuAhURrZQKHRydBiEQFjAEegQIDhAB&usg=AOvVaw1oiygJodeXK9Hs8crpubXM
0055◆iiDSZQODZc
垢版 |
2021/02/17(水) 17:52:13.27ID:pNOwDKWo
とりあえずこれです
https://en.m.wikipedia.org/wiki/Traditional_grammar

Traditional grammar is a framework for the description of the structure of a language.[1] The roots of traditional grammar are in the work of classical Greek and Latin philologists.[2] The formal study of grammar based on these models became popular during the Renaissance.[3]

Traditional grammars may be contrasted with more modern theories of grammar in theoretical linguistics, which grew out of traditional descriptions.[3] While traditional grammars seek to describe how particular languages are used, or to teach people to speak or read them, grammar frameworks in contemporary linguistics often seek to explain the nature of language knowledge and ability.[4] Traditional grammar is often prescriptive, and may be regarded as unscientific by those working in linguistics.[5]

Traditional Western grammars generally classify words into parts of speech. They describe the patterns for word inflection, and the rules of syntax by which those words are combined into sentences.[6]
0056◆iiDSZQODZc
垢版 |
2021/02/17(水) 18:01:00.89ID:pNOwDKWo
Traditional grammar is a framework for the description of the structure of a language.[1] The roots of traditional grammar are in the work of classical Greek and Latin philologists.[2] The formal study of grammar based on these models became popular during the Renaissance.[3]
まず一文目のa structure以降は名詞構文
a framework which describes the structure of a language
とパラフレーズ可能
philologist言語学者
work 研究
ここでのformalは「形における」の意味だろう
formal grammarという表現がある
このパラグラフにある通り規範文法とは英語の実情というよりラテン文法を参考にして作られていったという側面がある
バイト終わったら次やります

言語系の人間ではなく経済学部なので英語学の勉強をしたことがなく、専門的な知識がないため訂正があったら教えて下さい
0057◆iiDSZQODZc
垢版 |
2021/02/17(水) 18:02:20.55ID:pNOwDKWo
あ、パラフレーズではないか?
まぁ主述の関係は上の通り
バイト戻ります
0058◆iiDSZQODZc
垢版 |
2021/02/17(水) 20:28:09.29ID:O81ZUwfe
続き

Traditional grammars may be contrasted with more modern theories of grammar in theoretical linguistics, which grew out of traditional descriptions.[3] While traditional grammars seek to describe how particular languages are used, or to teach people to speak or read them, grammar frameworks in contemporary linguistics often seek to explain the nature of language knowledge and ability.[4] Traditional grammar is often prescriptive, and may be regarded as unscientific by those working in linguistics.[5]
grow out of Oから発展する、Oをやめる
be contrasted with Oと対比される
whileは時間ではなく対比を表す
whileは基本的に主節に先行するwhile SV, SVが基本
後ろにおいてもよいがSV, while SVとカンマを忘れない
どちらにせよ後ろにある節に力点がある
while は文頭に置かれると譲歩の場合があることに注意
これは後ろに力点が置かれることと整合する
the nature本質、特質[しばしばthe -]
prescriptive規範的
後ほど続きを
電池切れます
0059◆iiDSZQODZc
垢版 |
2021/02/17(水) 20:32:53.79ID:O81ZUwfe
those ingは
ingしている人々
の意
those who
0061◆iiDSZQODZc
垢版 |
2021/02/17(水) 21:21:08.51ID:zLf32ctK
Traditional Western grammars generally classify words into parts of speech. They describe the patterns for word inflection, and the rules of syntax by which those words are combined into sentences.[6]
parts of speechというフレーズは馴染み深い人も多いだろう
品詞の意味
inflectionは屈折、つまり活用の意味
see→sees, sawみたいなことね
ちなみに定形動詞と非定形動詞という用語は知っといても良い
定形動詞
主語や時制によって形が定まる動詞のこと
He loves her. 主語がHeで時制が非過去(現在時制のこと)だからlove→lovesのように屈折している
He loved her. は時制が過去だからlovedのように屈折している
つまり定形動詞とは形が1つに定まっている動詞ではなく周りの要素によって形が定められる(すなわち周りの要素なくしては形が定められない)動詞のこと
他方非定形動詞とは形が周りによって定められない動詞のことで、還元すれば唯一の形を持っているものであり、準動詞のことである。
I want to sing.のsingは例えば主語がHeでも変わらない(非定型)
この2つは混同しやすい。他の要素によって形が定められるかどうかである。
不定詞の不定が表すところは非定形の非定形が表すところと同じである。
まだ(未来志向)だから不定詞という説明は眉唾であり、子供向けの説明。
syntaxは統合の意味

統語論についてはこちらをどうぞ
https://ja.m.wikipedia.org/wiki/%E7%B5%B1%E8%AA%9E%E8%AB%96#:~:text=%E7%B5%B1%E8%AA%9E%E8%AB%96%EF%BC%88%E3%81%A8%E3%81%86%E3%81%94%E3%82%8D%E3%82%93,%E3%81%B6%E3%82%93%E3%82%8D%E3%82%93%EF%BC%89%E3%81%A8%E3%82%82%E3%81%84%E3%81%86%E3%80%82
0062◆iiDSZQODZc
垢版 |
2021/02/17(水) 21:23:04.48ID:zLf32ctK

とりあえずこの部分が終わったら次の英文にいくかな

Among the earliest studies of grammar are descriptions of Sanskrit, called vykara�a. The Indian grammarian P�ini wrote the A�dhy�y, a descriptive grammar of Sanskrit, sometime between the 4th and the 2nd century BCE.[7][8] This work, along with some grammars of Sanskrit produced around the same time, is often considered the beginning of linguistics as a descriptive science,[8] and consequently wouldn't be considered "traditional grammar" despite its aniquity. Although P�ini's work was not known in Europe until many centuries later, it is thought to have greatly influenced other grammars produced in Asia, such as the Tolk�ppiyam, a Tamil grammar generally dated between the 2nd and 1st century BCE.[9]

The formal study of grammar became popular in Europe during the Renaissance. Descriptive grammars were rarely used in Classical Greece or in Latin through the Medieval period.[10] During the Renaissance, Latin and Classical Greek were broadly studied along with the literature and philosophy written in those languages.
[11] With the invention of the printing press and the use of Vulgate Latin as a lingua franca throughout Europe, the study of grammar became part of language teaching and learning.[10]

Although complete grammars were rare, Ancient Greek philologists and Latin teachers of rhetoric produced some descriptions of the structure of language.
[12] The descriptions produced by classical grammarians (teachers of philology and rhetoric) provided a model for traditional grammars in Europe. According to linguist William Harris, "Just as the Renaissance confirmed Greco-Roman tastes in poetry, rhetoric and architecture, it established ancient Grammar, especially that which the Roman school-grammarians had developed by the 4th [century CE], as an inviolate system of logical expression."[8]
The earliest descriptions of other European languages were modeled on grammars of Latin. The primacy of Latin in traditional grammar persisted until the beginning of the 20th century.[8]

The use of grammar descriptions in the teaching of language, including foreign language teaching and the study of language arts, has gone in and out of fashion.[10] As education increasingly took place in vernacular languages at the close of the Renaissance, grammars of these languages were produced for teaching.
Between 1801 and 1900 there were more than 850 grammars of English published specifically for use in schools.[13] Mastering grammar rules like those derived from the study of Latin has at times been a specific goal of English-language education.[14] This approach to teaching has, however, long competed with approaches that downplay the importance of grammar instruction.[15]
Similarly in foreign or second language teaching, the grammar-translation method based on traditional Latin teaching, in which the grammar of the language being learned is described in the student's native language, has competed with approaches such as the direct method or the communicative approach, in which grammar instruction is minimized.[10]
0063◆iiDSZQODZc
垢版 |
2021/02/17(水) 21:51:24.05ID:zLf32ctK
Among the earliest studies of grammar are descriptions ofSanskrit, called�vy�kara�a. The Indian grammarian�P�ini�wrote theA�dhy�y, a�descriptive grammar�of Sanskrit, sometime between the 4th and the 2nd century BCE.[7][8]�This work, along with some grammars of Sanskrit produced around the same time, is often considered the beginning of�linguistics�as a�descriptive science,[8]�and consequently wouldn't be considered "traditional grammar" despite its aniquity. Although P�ini's work was not known in Europe until many centuries later, it is thought to have greatly influenced other grammars produced in Asia, such as the�Tolk�ppiyam, a�Tamil�grammar generally dated between the 2nd and 1st century BCE.[9]
文頭は[M]Among-[V]are[S]descriptions
というMVS倒置の典型
サンスクリット語の横で文字化けしているのはラトビア語
grammarian文法家
the NP, NPという非制限の同格(NPは名詞句のこと)
sometime between A and B
これは見慣れないとしても簡単
sometime around 1920:1920年頃に
ludwigで調べると42件出てくるためよくある表現であることがわかる
descriptive science記述科学
antiquity:the distant past (= a long time ago), especially before the sixth century:
along with X:in addition to someone or something else
grammarsが複数形になっていることに注意
文法書の意味
date Oの年代を突き止める
倒置という用語は広く使われすぎているため注意が必要 
例えばOSVやCSVというのは目的語Oや補語Cの前置という現象である。
逆に後置という現象も存在する
分離した同格のthat節などもそれ
倒置とはこれらの現象などによりVSという形になっているものを指す

The formal study of grammar became popular in Europe during the�Renaissance. Descriptive grammars were rarely used inClassical Greece�or in�Latin�through the�Medieval period.[10]During the Renaissance, Latin andClassical Greek�were broadly studied along with the literature and philosophy written in those languages.[11]�With the invention of the�printing press�and the use of Vulgate Latin as a�lingua franca�throughout Europe, the study of grammar became part of�language teaching and learning.[10]
through 〜で「〜まで」の意味(時間的)
これはアメリカの用法のはず
broadly広く
printing pressで印刷機のこと
throughout 〜中で
vulgate
ウルガタ(聖書) ((カトリック教会で一般に用いるラテン語訳聖書))
lingua franca
リング・フランカ ((異なる母国語の話し手どうしで用いられる混成共通語))
part of Oの一環
0064◆iiDSZQODZc
垢版 |
2021/02/17(水) 21:52:28.64ID:zLf32ctK
文字化けしすぎ
すまん
変なコピペした
0065◆iiDSZQODZc
垢版 |
2021/02/17(水) 21:58:09.23ID:zLf32ctK
疲れたし次のやつ
面白いの探す
動物がいい
0066◆iiDSZQODZc
垢版 |
2021/02/17(水) 22:26:51.50ID:zLf32ctK
https://www.timeforkids.com/g56/prehistoric-paintings/

British and Colombian archaeologists have discovered a large display of ancient rock art. It’s on cliff faces in the Colombian Amazon rainforest. The drawings were probably created 12,500 years ago. They provide further proof that the rainforest’s earliest inhabitants lived alongside Ice Age animals. The archaeologists made this discovery in 2019. But they kept it secret until now, ahead of the rock art’s debut in an upcoming documentary.
archaeologist考古学者
cliff face:the vertical face of a cliff
proof that SVということの証拠
theyはdrawingsのこと
もちろん研究者のことじゃないよ(たぶん)
inhabitant住民
alongside Oとともに
ahead of Oに先立って
0067◆iiDSZQODZc
垢版 |
2021/02/17(水) 23:49:54.24ID:zLf32ctK
ちなみに最後の文は
今まで秘密してきた→今になってやっと公開した(〜に先立って)
と解釈
0068◆iiDSZQODZc
垢版 |
2021/02/18(木) 00:53:09.86ID:fB1uOaWq
クジラ構文の由来となった言葉
New-York is a point into which much information centers. Men departing from this point circumnavigate the globe, voyaging from the arctic to the antarctic regions. From this class of my fellow citizens, much of the information I possess on this subject has been derived; and as a man of science, I can say positively, that a whale is no more a fish than a man; nobody pretends to the contrary now-a-days, but lawyers and politicians.
center集まる
center on Oに集まる をよく見るがcenter intoもあるということ
circumnavigate Oを一周する
circleですね
navigate航海する案内する進む は必須 
the arctic北極地方
the antarctic南極地方
このthis classは部類の意味で「men departing from this point」を受け継いでいるのだろう
derive Oを引き出す推測する
deriveを修飾するfromの前置詞句が文頭に前置されている
ちなみに由来するの意味であれば完了相はおかしい
a man of science科学者
このpositivelyは「キッパリと」の意味
to the contraryはpretendに対する補語
but以降は名詞句しか存在しないがpretended to the contrary (thenなど)が省略されている
クジラ構文とは飽くまでも「A⇔B」という2つの異なる命題が同値、つまり真偽が一致することを主張する文
Aの真偽に説得力を持たせるためにBという真理値の明らかな命題と同値と伝えるのである
そして多くの場合は互いが偽の場合に用いる
0069◆iiDSZQODZc
垢版 |
2021/02/18(木) 00:57:24.02ID:sCSpsMuH
ごめん間違えた
これ全部セリフなんだから
最後のbutは単なる前置詞ですね
0071◆iiDSZQODZc
垢版 |
2021/02/18(木) 02:38:14.81ID:sCSpsMuH
conjureは2年前の早稲田でも出ていましたね
ではここの読解を明日から

The mention of Cupid typically conjures up images of a cherubic infant wielding a bow and arrow, but this wasn’t always the case. Long before the Romans adopted and renamed himand way before his association with Valentine's DayCupid was known to the Greeks as Eros, the handsome god of love.

One of the first authors to mention Eros (circa 700 B.C.) was Hesiod, who described him in “Theogony” as one of the primeval cosmogonic deities born of the world egg. But later accounts of the lineage of Eros vary, describing him as the son of Nyx and Erebus; or Aphrodite and Ares; or Iris and Zephyrus; or even Aphrodite and Zeus�who would have been both his father and grandfather.

Armed with a bow and a quiver filled with both golden arrows to arouse desire and leaden arrows to ignite aversion, Eros struck at the hearts of gods and mortals and played with their emotions. In one story from ancient Greek mythology, which was later retold by Roman authors, Cupid (Eros) shot a golden arrow at Apollo, who fell madly in love with the nymph Daphne, but then launched a leaden arrow at Daphne so she would be repulsed by him.


In another allegory, Cupid’s mother, Venus (Aphrodite), became so jealous of the beautiful mortal Psyche that she told her son to induce Psyche to fall in love with a monster. Instead, Cupid became so enamored with Psyche that he married her�with the condition that she could never see his face. Eventually, Psyche’s curiosity got the better of her and she stole a glance, causing Cupid to flee in anger. After roaming the known world in search of her lover, Psyche was eventually reunited with Cupid and granted the gift of immortality.

In the poetry of the Archaic period, Eros was represented as a studly immortal who was irresistible to both man and gods. But by the Hellenistic period, he was increasingly portrayed as a playful, mischievous child. Because of his associations with love, 19th-century Victorians�credited with popularizing Valentine's Day and giving the holiday its romantic spin�began depicting this cherubic version of Cupid on Valentine’s Day cards in a trend that has persisted until this day.
0072◆iiDSZQODZc
垢版 |
2021/02/18(木) 02:45:25.37ID:sCSpsMuH
また文字化けすまん
明日読むときには直しとく
調べてわかったけどHistory.comは広島大学とか同志社大学で問題文の出典になったりしてるんだね
長くないし、単語とか構文も勉強になるレベルだからここ気に入るかも
0073◆iiDSZQODZc
垢版 |
2021/02/18(木) 03:10:35.41ID:sCSpsMuH
>>14
訂正
タイポしてた
x be attribute to
o be attributed to
0076名無しなのに合格
垢版 |
2021/02/18(木) 13:52:43.27ID:4vOhagib
こういう事例があるから、文法には例外がつきものなんだなと思った
以下は「一音節」の形容詞にmoreがつく事例


321 名無しさん@英語勉強中 (ササクッテロ Spc7-buwc)[] 2021/02/16(火) 03:50:08.29 ID:tdVTA0QRp
moreを辞書で引くと色々な発見があって面白い
形容詞・名詞・副詞とあって、言われればそうだなと気づくことばかりだ
今までの人生で辞書とかほとんど引いたことないから新鮮な驚きだ

ところで、家庭教師で準1を目指してる女の子を何ヶ月か生徒として受け持ったことがあるが、
英作文で"Studying English is more fun than playing the piano."みたいな文をその生徒が書いてきたことがある

自分は規範文法に沿って「funは一音節だからmoreは前に来ないんだよ。funnierも少し微妙だからmore enjoyableにしてみよう!!」と答案を訂正したことがある

しかし、自宅に帰って調べてみたところ"A is more fun than B"という形が、結構Twitterや日常会話で使われてることがわかった

規範文法を杓子定規的に当てはめてしまった自分を恥じて、翌週にその生徒には「ごめんね、やっぱりmore funで良かったんだ」と謝った思い出もある
moreという単語にはとても思い入れがある
0077◆iiDSZQODZc
垢版 |
2021/02/18(木) 14:32:10.16ID:OI/7B5/9
>>76
それはまた違うよ
funは名詞だから
まあmore A than B「BというよりA」のときもBが原級だからそこに対応してAを原級にするのが普通とかもあるね
0078◆iiDSZQODZc
垢版 |
2021/02/18(木) 14:44:11.21ID:OI/7B5/9
I had alot of�fun on the wedding party.
は可能だけどa lot ofの代わりにveryにすることはできない。funが名詞だから
0079◆iiDSZQODZc
垢版 |
2021/02/18(木) 14:44:52.44ID:OI/7B5/9
I had a lot of fun on the wedding party.
0080◆iiDSZQODZc
垢版 |
2021/02/18(木) 15:39:57.58ID:OI/7B5/9
まぁそれでも規範的には間違いだが
ないわけではない
ludwigで調べると
a lot of funとvery funの比率は81:9で
very funも存在している
It's very fun.(The Guardian)
It's been very fun.(The New York Times)
ただ圧倒的にa lot of funが多く、規範的に教えるならばmore funだろう
more funとfunnerの比率は更に差が開き、98:2である。
ちなみにfunnierはfunnyの比較級であるから何重にもミスをしてしまっている
0081名無しなのに合格
垢版 |
2021/02/18(木) 15:45:24.56ID:4vOhagib
funには形容詞もあるよ。主にアメリカ英語の方になるけど
0082◆iiDSZQODZc
垢版 |
2021/02/18(木) 15:59:46.01ID:OI/7B5/9
>>81
形容詞の例は上に書いてあるよ
ただIt's fun.のfunを形容詞だって主張するのは無理がある
辞書でも形容詞的用法として載っているのは名詞の形容詞的用法と捉えることもできそうな限定修飾が基本だし
0083◆iiDSZQODZc
垢版 |
2021/02/18(木) 16:11:27.44ID:OI/7B5/9
英文は出勤の電車で読みます
0085名無しなのに合格
垢版 |
2021/02/18(木) 18:41:25.53ID:tHWjViID
>>76見て思い出したけど
よく「英語の勉強」って言う場合はstudyよりlearnの方が正しい、studyだと研究してるみたい、って話があるけどどうなの?
0086名無しなのに合格
垢版 |
2021/02/18(木) 19:00:02.64ID:GKDqx+5t
「学ぶ」はlearnで訳すしな。
でも「知る」もlearnで訳すよな。
0088◆iiDSZQODZc
垢版 |
2021/02/18(木) 22:41:42.34ID:6/mLpg0z
learnは習得までも含意するってのは有名な話
まぁこれに対する反論も見たことあるけどね
0089◆iiDSZQODZc
垢版 |
2021/02/18(木) 23:03:21.28ID:6/mLpg0z
では各英英辞書を見てみよう
まずはLongman
・Learn
to gain knowledge of a subject or skill, by experience, by studying it, or by being taught
・Study
to learn about a subject at school, university etc
(勉強するに対応するような意味のみ取り上げています)
次にCambridge
・Learn
to get knowledge or skill in a new subject or activity:
・Study
to learn about a subject, especially in an educational course or by reading books:
次にMerriam-Webster
・Learn
to gain knowledge or understanding of or skill in by study, instruction, or experience
・Study
@to read in detail especially with the intention of learning
A to engage in the study of
次にMacmillan
・Learn
to gain knowledge or experience of something, for example by being taught
・Study
to learn about a subject by going to school, university etc
次にOxford Leaner's Dictionaries
・Learn
to gain knowledge or skill by studying, from experience, from being taught, etc.
・Study
to spend time learning about a subject by reading, going to college, etc.
次にCollins
・Learn
If you learn something, you obtain knowledge or a skill through studying or training.
・Study
If you study, you spend time learning about a particular subject or subjects.
最後にUrban Dictionary
・Learn
To persuade someone to your way of thinking via violent means.
例文
"I'll learn you good!" (Followed by a slap)
・Study
To go over to the opposite sex's house not to study, but to "study." "Studying" usually involves fooling around without any pants on.
例文
"I'm going over to Amanda's house to 'study'"
0091◆iiDSZQODZc
垢版 |
2021/02/18(木) 23:08:46.90ID:6/mLpg0z
Urban Dictionaryは読んでて面白いよ
ちなみに2月11日に「今日の単語」として紹介されてたのがこれ
・smash the guitar
To masturbate vigorously, as if romancing one's beloved guitar. Common among musicians and college freshmen.

Bro: "Man, some nights I love to just lie in bed and smash the guitar."
0092◆iiDSZQODZc
垢版 |
2021/02/18(木) 23:19:06.02ID:6/mLpg0z
ちなみにここで出てきている
as if romancing one's beloved guitar
は受験においても大事な文法事項を孕んでいる
as if は節だけでなくto不定詞、分詞、前置詞なども引き連れることができるってやつですね
もうひとつ、as ifのasとifとの間に存在する省略と、それに関する時制の問題も大事な項目だけどそれはまたいつか多読で出てきたら
0093名無しなのに合格
垢版 |
2021/02/18(木) 23:20:05.62ID:tHWjViID
studyによってlearnするってのがめっちゃ分かりやすいな
learnは経験や勉強を通して学ぶこと、studyは勉強することって感じか
0094名無しなのに合格
垢版 |
2021/02/18(木) 23:21:51.75ID:0kccqPrb
>>92
これってif necessaryとかのit isの省略って習うやつとは別物なの?
0095◆iiDSZQODZc
垢版 |
2021/02/18(木) 23:28:24.54ID:6/mLpg0z
>>94
もちろんなんかしらが省略されていると考えていいけど、いろんなものがくるって認識しとけば問題はない
0096◆iiDSZQODZc
垢版 |
2021/02/18(木) 23:31:56.65ID:6/mLpg0z
あ、でも省略と見なすのは無理がある場合もありそう
0097名無しなのに合格
垢版 |
2021/02/18(木) 23:45:07.60ID:eET4mekb
わっしょいわっしょい
0098◆iiDSZQODZc
垢版 |
2021/02/19(金) 00:12:19.93ID:L0nzyHmq
The mention of Cupid typically conjures up images of a cherubic infant wielding a bow and arrow, but this wasn’t always the case. Long before the Romans adopted and renamed him--and way before his association with Valentine's Day--Cupid was known to the Greeks as Eros, the handsome god of love.

One of the first authors to mention Eros (circa 700 B.C.) was Hesiod, who described him in “Theogony” as one of the primeval cosmogonic deities born of the world egg. But later accounts of the lineage of Eros vary, describing him as the son of Nyx and Erebus; or Aphrodite and Ares; or Iris and Zephyrus; or even Aphrodite and Zeus--who would have been both his father and grandfather.

Armed with a bow and a quiver filled with both golden arrows to arouse desire and leaden arrows to ignite aversion, Eros struck at the hearts of gods and mortals and played with their emotions. In one story from ancient Greek mythology, which was later retold by Roman authors, Cupid (Eros) shot a golden arrow at Apollo, who fell madly in love with the nymph Daphne, but then launched a leaden arrow at Daphne so she would be repulsed by him.

In another allegory, Cupid’s mother, Venus (Aphrodite), became so jealous of the beautiful mortal Psyche that she told her son to induce Psyche to fall in love with a monster. Instead, Cupid became so enamored with Psyche that he married her--with the condition that she could never see his face. Eventually, Psyche’s curiosity got the better of her and she stole a glance, causing Cupid to flee in anger. After roaming the known world in search of her lover, Psyche was eventually reunited with Cupid and granted the gift of immortality.


In the poetry of the Archaic period, Eros was represented as a studly immortal who was irresistible to both man and gods. But by the Hellenistic period, he was increasingly portrayed as a playful, mischievous child. Because of his associations with love, 19th-century Victorians--credited with popularizing Valentine's Day and giving the holiday its romantic spin--began depicting this cherubic version of Cupid on Valentine’s Day cards in a trend that has persisted until this day.
0099英文多読者 ◆iiDSZQODZc
垢版 |
2021/02/19(金) 01:07:55.45ID:L0nzyHmq
名前つけた
The mention of Cupid typically conjures up images of a cherubic infant wielding a bow and arrow, but this wasn’t always the case. Long before the Romans adopted and renamed him--and way before his association with Valentine's Day--Cupid was known to the Greeks as Eros, the handsome god of love.
conjure A upを思い起こす
これは早稲田の正誤で出てたのが思い出される
cherubic天使のような、ケルビムの(ような)ケルビム:智天使。天使の一種。偽ディオニシウス・アレオパギタに由来する「天使の階級」では第二位に位置づけられる
wield Oを扱う
the case事実、実情、真相
That's not the case. 事実はそうではない
wayは注意が必要な単語。wayは「はるかに」の意味で強調のために用いられることがあるHis coat is way too big. 彼のコートはちょっと大きすぎる.とかでtoo 〜についたりもする
associate with Oと交際する結びつく
associate A with B連想する
このbeforeは前置詞であるが名詞構文のため
before he associated with Valentine's Dayとパラフレーズできるようにしたい
ただここは悩みどころ
Soon a lifeguard came to my resctue.
のように名詞構文で所有格が目的語を表すこともあるためhe was associated with Valentine's Dayとパラフレーズすることも可能(wasでいいのかな)
まぁどっちでもそんなに変わらないし前者で取るかな
Eros 《ギリシャ神話》エロス
(◇愛の神;ローマ神話の Cupid に当たる)


One of the first authors to mention Eros (circa 700 B.C.) was Hesiod, who described him in “Theogony” as one of the primeval cosmogonic deities born of the world egg. But later accounts of the lineage of Eros vary, describing him as the son of Nyx and Erebus; or Aphrodite and Ares; or Iris and Zephyrus; or even Aphrodite and Zeus--who would have been both his father and grandfather.
to mentionはfirstに呼応
Theogony神統記 著書の名前ですね
theoは神を表す
神話を表すことmythやmythologyのth
primeval原始の
「medieval中世の」と似てますね
deity神
born of Oから生まれる、Oのもとに生まれる
world eggは初めて聞いた
https://en.m.wikipedia.org/wiki/World_egg
暇なときちゃんと読みます
account of Oに対する説明 
linage血筋
varyは「異なる」や「様々である」という意味の状態動詞である「変化する」の動作動詞ではない
(Aresも)AphroditeがZeusの子供だから次の英文が意味を持つ
who would have been both his father and grandfather
ここらへんをしっかり掴むためには教養が必要(俺にはないので調べながら)

Armed with a bow and a quiver filled with both golden arrows to arouse desire and leaden arrows to ignite aversion, Eros struck at the hearts of gods and mortals and played with their emotions. In one story from ancient Greek mythology, which was later retold by Roman authors, Cupid (Eros) shot a golden arrow at Apollo, who fell madly in love with the nymph Daphne, but then launched a leaden arrow at Daphne so she would be repulsed by him.
be armed with Oで武装している
quiver震える
今回はこの意味ではなく矢筒を意味する名詞
arouse Oを刺激する
leaden鉛製の
aversion嫌悪
ignite Oに火を付ける
strike at Oに向けて打つ
mortals人間(死の運命ある)
play with Oをもてあそぶ
retell Oを(平易に)書き直す
madly気が狂ったように
repulse Oに拒絶反応を起こさせる
be repulsed by Oに強い嫌悪感を抱く
ここのsoは「なので」と訳さないようにしたい
なぜwouldなのかを考えたい
so thatのthatの省略であり、目的を表している。ちなみにsoが省略されることもある
0101名無しなのに合格
垢版 |
2021/02/19(金) 01:14:27.60ID:XH44CAf3
英文読み漁ってるとso thatなんてちゃんと書いてある方が珍しいぐらい大体soだけのイメージ
0106英文多読者 ◆iiDSZQODZc
垢版 |
2021/02/19(金) 02:13:38.11ID:L0nzyHmq
さっきのigniteだけど
イグニット系の名前の炎のキャラクターいるよね

In another allegory, Cupid’s mother, Venus (Aphrodite), became so jealous of the beautiful mortal Psyche that she told her son to induce Psyche to fall in love with a monster. Instead, Cupid became so enamored with Psyche that he married her--with the condition that she could never see his face. Eventually, Psyche’s curiosity got the better of her and she stole a glance, causing Cupid to flee in anger. After roaming the known world in search of her lover, Psyche was eventually reunited with Cupid and granted the gift of immortality.
allegory寓意、寓喩
こんな言葉知らないから日本語を勉強しないと
be jealous of Oを妬んでいる
so...that構文
induce O to Vするように誘導する
instead:その代わりに、それをしないで、それどころか
be enamored with Oに夢中になっている
再びso...that
二度あることは三度あるということ
英文解釈教室を持っている人は1.4 例題(1)を読もう
on (the) condition that SVという条件で
このonがwithにできるのかな?
それともwith OCって解釈すればいいかな
curiosity好奇心
get the better of Oに勝る
steal a glance at Oをちらりと盗み見る
causingは主節を主語とする分詞構文
このfleeは」逃げる」というより「消え失せる」の意味
in 感情 で 「〜という気持ちで」 の意味
in anger怒って
roam Oを当てもなく放浪する
in search of Oを探し求めて
eventually最終的には
be reunited with Oと再開する
immortality不死
ここのgrantは注意が必要
一見、謝罪としてプレゼントとして与えた
と誤読しかねない
ただ王女が不死をプレゼントするとは考えにくい
grantはwasではなくreunitedと並列されどちらも受動態

In the poetry of the Archaic period, Eros was represented as a studly immortal who was irresistible to both man and gods. But by the Hellenistic period, he was increasingly portrayed as a playful, mischievous child. Because of his associations with love, 19th-century Victorians--credited with popularizing Valentine's Day and giving the holiday its romantic spin--began depicting this cherubic version of Cupid on Valentine’s Day cards in a trend that has persisted until this day.
the Archaic period古代ギリシャのアルカイック期
represent A as Bとして表現する描く
studly体つきのよい、性的に魅力のある
immortal不死の者
be irresistible to Oにとって抵抗できないほど魅力的な 当然ここのmanは人類のこと
Hellenistic period古代ギリシャのヘレニズム期
increasingly次第に
protray A as Bどして描く
playfulふざけた
mischievousいたずら好きな
be credited with Oの功績があると信じられている
popularize Oを広める
spin解釈
depict Oを描写する
persist 生き残る、続く
0109英文多読者 ◆iiDSZQODZc
垢版 |
2021/02/19(金) 02:44:16.97ID:jaeVXUsq
次はこのシリーズ読もうかな
憲法系は難しいだろうな
語彙も構文も
一応イギリスの法典化されていない憲法とかアメリカとイギリスの憲法の違いとかはなんとなくの知識は大学の授業で読んだことがあるけどあまり覚えていない
TFKからレベルが上がってきた
https://www.history.com/history-at-home-us-constitution
0110英文多読者 ◆iiDSZQODZc
垢版 |
2021/02/19(金) 02:53:38.20ID:jaeVXUsq
もし本文とか読んでる素材以外でも質問あったら書き込んで
答えられるかわからないけど
0111英文多読者 ◆iiDSZQODZc
垢版 |
2021/02/19(金) 04:10:01.31ID:jaeVXUsq
Several framers met with untimely deaths.
Was there a curse of the Constitution? Alexander Hamilton was famously killed by Aaron Burr in 1804, but he wasn’t even the first framer of the U.S. Constitution to die in a duel with a political rival. In 1802, North Carolina delegate Richard Spaight was mortally wounded by a dueling pistol fired by sitting congressman John Stanly. Four years later, Virginian George Wythe died of arsenic poisoning, likely at the hand of a debt-riddled grandnephew and heir. Pennsylvania delegate Gouverneur Morris died in 1816 after a ghastly bit of self-surgery in which he unsuccessfully attempted to dislodge a urinary tract blockage with a piece of whale bone, while New York’s John Lansing mysteriously vanished in December 1829 after leaving his Manhattan hotel room to mail a letter.

Rhode Island boycotted the Constitutional Convention.
America’s littlest state had a big independence streak. Rhode Island, distrustful of a powerful federal government, was the only one of the 13 original states to refuse to send delegates to the Constitutional Convention. It was a decision that rankled even the normally temperate George Washington, who wrote in July 1787 that “Rhode Island … still perseveres in that impolitic, unjust, and one might add without much impropriety scandalous conduct, which seems to have marked all her public councils of late.” On the condition that a Bill of Rights be included, Rhode Island became the 13th state to ratify the Constitution on May 29, 1790, more than a year after Washington was sworn in as president.

Some big names were absent from the Constitutional Convention.
When Thomas Jefferson gushingly called the Constitutional Convention delegates “an assembly of demigods,” he wasn’t being full of himself. Jefferson was not among the founding fathers who gathered in Philadelphia; he was in Paris serving as minister to France. John Adams was also abroad, serving as minister to Great Britain. Samuel Adams, John Hancock and Patrick Henrywho turned down an invitation because he “smelt a rat in Philadelphia, tending toward the monarchy”�also did not participate.
Attendance was spotty.
When the Constitutional Convention opened on May 14, 1787, only delegates from Pennsylvania and Virginia were present. It wasn’t until May 25 that a quorum of seven states was achieved. Weather�ever the convenient excuse�was blamed for the tardiness, but the convention was plagued throughout with attendance issues. While James Madison boasted that he never left the proceedings for more than “a casual fraction of an hour,” his fellow delegates were not as fastidious. Nineteen of the 74 delegates to the convention never even attended a single session, and of the 55 delegates who did show up in Philadelphia, no more than 30 stayed for the full four months. New Hampshire’s delegation arrived two months late, by which time two of New York’s three delegates had left in opposition to the proceedings, leaving just Hamilton behind and depriving the state of a quorum to vote. Thus, Washington wrote that the Constitution was signed by “11 states and Colonel Hamilton.”
0112英文多読者 ◆iiDSZQODZc
垢版 |
2021/02/19(金) 04:16:23.24ID:jaeVXUsq
1. Several framers met with untimely deaths.
Was there a curse of the Constitution? Alexander Hamilton was famously killed by Aaron Burr in 1804, but he wasn’t even the first framer of the U.S. Constitution to die in a duel with a political rival. In 1802, North Carolina delegate Richard Spaight was mortally wounded by a dueling pistol fired by sitting congressman John Stanly. Four years later, Virginian George Wythe died of arsenic poisoning, likely at the hand of a debt-riddled grandnephew and heir. Pennsylvania delegate Gouverneur Morris died in 1816 after a ghastly bit of self-surgery in which he unsuccessfully attempted to dislodge a urinary tract blockage with a piece of whale bone, while New York’s John Lansing mysteriously vanished in December 1829 after leaving his Manhattan hotel room to mail a letter.

2.Rhode Island boycotted the Constitutional Convention.
America’s littlest state had a big independence streak. Rhode Island, distrustful of a powerful federal government, was the only one of the 13 original states to refuse to send delegates to the Constitutional Convention. It was a decision that rankled even the normally temperate George Washington, who wrote in July 1787 that “Rhode Island … still perseveres in that impolitic, unjust, and one might add without much impropriety scandalous conduct, which seems to have marked all her public councils of late.” On the condition that a Bill of Rights be included, Rhode Island became the 13th state to ratify the Constitution on May 29, 1790, more than a year after Washington was sworn in as president.

3.Some big names were absent from the Constitutional Convention.
When Thomas Jefferson gushingly called the Constitutional Convention delegates “an assembly of demigods,” he wasn’t being full of himself. Jefferson was not among the founding fathers who gathered in Philadelphia; he was in Paris serving as minister to France. John Adams was also abroad, serving as minister to Great Britain. Samuel Adams, John Hancock and Patrick Henry--who turned down an invitation because he “smelt a rat in Philadelphia, tending toward the monarchy”--also did not participate.
0113英文多読者 ◆iiDSZQODZc
垢版 |
2021/02/19(金) 04:16:29.24ID:jaeVXUsq
4.Attendance was spotty.
When the Constitutional Convention opened on May 14, 1787, only delegates from Pennsylvania and Virginia were present. It wasn’t until May 25 that a quorum of seven states was achieved. Weather--ever the convenient excuse--was blamed for the tardiness, but the convention was plagued throughout with attendance issues. While James Madison boasted that he never left the proceedings for more than “a casual fraction of an hour,” his fellow delegates were not as fastidious. Nineteen of the 74 delegates to the convention never even attended a single session, and of the 55 delegates who did show up in Philadelphia, no more than 30 stayed for the full four months. New Hampshire’s delegation arrived two months late, by which time two of New York’s three delegates had left in opposition to the proceedings, leaving just Hamilton behind and depriving the state of a quorum to vote. Thus, Washington wrote that the Constitution was signed by “11 states and Colonel Hamilton.”

5.Not all the delegates who attended the convention signed the Constitution.
Although 55 delegates participated in the Constitutional Convention, there are only 39 signatures on the Constitution. Fourteen men, having already left Philadelphia, were not present for the signing, and only Delaware delegate John Dickinson had a proxy sign for him. Three delegates--Elbridge Gerry of Massachusetts and Edmund Randolph and George Mason of Virginia--were dissatisfied with the final document and refused to ink their signatures.
6.“We the People of the United States” was a late change.
The Constitution’s iconic opening line was not included in early drafts of the document. Instead, the preamble started with a much less pithy litany of individual states listed from north to south: “We the people of the States of New Hampshire, Massachusetts…” and so on. Credit for the late alteration goes to a five-person Committee of Style--comprised of Hamilton, Madison, Morris William Samuel Johnson and Rufus King--and Morris is considered to have been responsible for composing much of the final text, including the revised preamble.
7.The man who hand-wrote the Constitution was not a delegate.
While Morris has been nicknamed the “Penman of the Constitution,” the real hand wielding the quill that scrawled the final copy of the Constitution belonged to Jacob Shallus. The assistant clerk of the Pennsylvania State Assembly was paid $30 and given just two days to write most of the document’s 4,543 words on four sheets of vellum parchment. While his script was exquisite, Shallus wasn’t totally flawless. Between the final article and the delegate signatures on the Constitution’s final page is an “errata” paragraph listing some of the minor errors he had made along with their corrections.
0114英文多読者 ◆iiDSZQODZc
垢版 |
2021/02/19(金) 04:17:01.21ID:jaeVXUsq
これで文字化けないね
1.とかのあとにスペースなかったりするけど許して
ダッシュは--で代用
0115英文多読者 ◆iiDSZQODZc
垢版 |
2021/02/19(金) 04:26:37.64ID:jaeVXUsq
文法的に面白いところがあるね
On the condition that a Bill of Rights be included,
詳しいことはそこを読むときに
0116名無しなのに合格
垢版 |
2021/02/19(金) 04:34:45.38ID:h1TMke2k
>>115
仮定法現在?
0117英文多読者 ◆iiDSZQODZc
垢版 |
2021/02/19(金) 05:01:11.51ID:jaeVXUsq
>>116
そそ
格式張った英語ではifとかwhetherの中で原形が使われるってやつですね
仮定法現在でok
同じような意味を表すon condition that SVでも同様
if need beだとほぼ義務的に仮定法現在だけど、それ以外では「まれ」ですね


でもここらへんの英文を毎日読んでたら
「まれ」と呼ばれるやつによく出会う気がする
ジャンルの問題かな
0118英文多読者 ◆iiDSZQODZc
垢版 |
2021/02/19(金) 05:06:24.13ID:jaeVXUsq
仮定法現在に絡んで命令法譲歩構文というものがあるけど
ここらへんは難しいね
0119英文多読者 ◆iiDSZQODZc
垢版 |
2021/02/19(金) 05:30:04.55ID:jaeVXUsq
書くにあたって参照することが多い書籍など
Wisdom英和辞典
オンラインにある英英辞典
英辞郎
英文法総覧
英文解釈教室
英文法詳解
英文法解説
α英文法
英文法の真相
読むための英文法
ぐらいかな

もう少し自分の知識と参照する書籍のレベルを上げたい
まぁじっくり勉強します
0120英文多読者 ◆iiDSZQODZc
垢版 |
2021/02/19(金) 13:40:21.98ID:HdLPz/6u
1. Several framers met with untimely deaths.
Was there a curse of the Constitution? Alexander Hamilton was famously killed by Aaron Burr in 1804, but he wasn’t even the first framer of the U.S. Constitution to die in a duel with a political rival. In 1802, North Carolina delegate Richard Spaight was mortally wounded by a dueling pistol fired by sitting congressman John Stanly. Four years later, Virginian George Wythe died of arsenic poisoning, likely at the hand of a debt-riddled grandnephew and heir. Pennsylvania delegate Gouverneur Morris died in 1816 after a ghastly bit of self-surgery in which he unsuccessfully attempted to dislodge a urinary tract blockage with a piece of whale bone, while New York’s John Lansing mysteriously vanished in December 1829 after leaving his Manhattan hotel room to mail a letter.
frame Oを練る立案する 
framerはframeする(した)人々のことだろう
辞書にはない
meet with Oに出くわす経験する
untimely death早すぎる死
curse呪い、災い
famouslyは文修飾であることに注意
firstにto dieが呼応
the first framer of the U.S. Constitution to die...も名詞構文のようなもの
the first person who framed the U.S. Constitution to die...
とパラフレーズ可能
a duel with Oとの決闘論争対決
in a duel with Oはdieにかかるため、他にも政敵との対決の末に死んだframerが存在するということ
a political rivalもアレクサンダー大王の英文で出てきましたね
ここのdelegateは動詞ではなく人名に掛かる役職を表す言葉
delegate下院議員
mortally致命的に、ひどく
wound Oを傷つける
dueling pistols
(2丁一組の) 決闘用ピストル
もちろんこれは分詞ではなく動名詞
pistolがduelするわけではない
congressman下院議員
sitting現職の(限定修飾のみ)
die of Oで死ぬ
arsenic poisoningヒ素中毒
arsenicヒ素
poisoning中毒
be riddled with Oでいっぱいである
debt-riddled≒debt-ridden借金でいっぱいの
at the hand(s) of Oの手によって
likelyおそらく(副)
grandnephew姪/甥の息子
heir相続人
ここは注意しなければいけない
grandnephewとheirは同一人物である
理由はheirにはaがないため
The novelist and doctor cameto the party. は同一人物
The novelist and the doctorCcime to the party.
は2人
ghastlyぞっとするような
a bit ofのaとbitの間に入るんだね 修飾先の問題かな また後で考える
unsuccessfullyはその結果を示している
dislodge Oを取り除く
urinary尿の泌尿器の
urinary tract尿路
blockage障害物
尿路結石かな?
whileは他方の意味
New York's にはdelegateが省略されている
vanish姿を消す
mail a letter手紙を出す
mail Oを郵送する
0121英文多読者 ◆iiDSZQODZc
垢版 |
2021/02/19(金) 13:57:14.20ID:HdLPz/6u
a bit of ghastly self-surgeryだとa bit ofがghastlyに掛かると解釈されうるから、それを避けようとした結果かな
0122英文多読者 ◆iiDSZQODZc
垢版 |
2021/02/19(金) 16:10:05.56ID:HdLPz/6u
これってどうなんだろう
もしかしたら素晴らしいのかな
https://youtu.be/zraKxwx0KN4
パンッ!これがwith!!ってのは全くもって説明になってと思うのだが、意味をちゃんと書いている
単なる意味に加えて説明になっていない感覚的な説明を加えたらなんとなく分かる気になって意味を分かりやすくなるのかな
意外と教育的で優しいのかもしれない
知らないけど
0125名無しなのに合格
垢版 |
2021/02/19(金) 17:06:18.46ID:juWlOmbF
>>122
個人的にはこんな説明されたら授業切るw
訳し分けと感覚的な捉え方(パン!)の関連もいまいちだし説明になってないと思う
速読するときのwithの感覚的な捉え方で言うと関の「付帯と対立の2パターンに分類できる」って話は納得した
0126英文多読者 ◆iiDSZQODZc
垢版 |
2021/02/19(金) 17:12:45.44ID:EM9BsW9G
まぁどこまで真面目に、どこまで正確にするかってのは難しいけどね
なんとなく分かった気にさせるってのも苦肉の策としてはあるのかもしれない
俺は嫌いだけど
0127英文多読者 ◆iiDSZQODZc
垢版 |
2021/02/19(金) 17:21:55.87ID:EM9BsW9G
まぁ英語ってのは他の科目に比べて
嘘も方便
ってのが許さがちな分野ではある
寧ろ必要な側面もある
0128名無しなのに合格
垢版 |
2021/02/19(金) 17:28:01.32ID:/yd4iWLN
まあ特に学部受験レベルでは言語学的な正確性よりも短い時間で分かりやすく忘れにくい説明が正義なのはそう
0129名無しなのに合格
垢版 |
2021/02/19(金) 18:19:49.94ID:8m9IV6pk
sustainable erection

訳せ
0130名無しなのに合格
垢版 |
2021/02/19(金) 18:37:09.80ID:Fnwm3iwa
Everyone who saw her liked her, but most of all her grandmother, who did not know what to give the child next.


↑これ、別スレにあった「赤ずきん」の英語版(Little Red Riding Hood)の一節らしいんだけど、
but以下の動詞が無いのは省略されてるからなのかな?それとももっと別の理由?
0131英文多読者 ◆iiDSZQODZc
垢版 |
2021/02/19(金) 18:53:23.21ID:EM9BsW9G
>>130
most of allとりわけ

Ugliness is one of the symptoms of disease, beauty of health.が
Ugliness is one of the symptoms of disease and beauty is one of the symptoms of health.
の共通部分を省略したものであるのと同じで
liked herが省略されてるんじゃないかな
(英文解釈教室14.4.10より)

, whoに関しては連結用法と捉えて良さそう
0133名無しなのに合格
垢版 |
2021/02/19(金) 18:59:51.01ID:63SEvWeW
>>130
everyone〜butでbutは前置詞じゃね?
0135英文多読者 ◆iiDSZQODZc
垢版 |
2021/02/19(金) 19:03:49.87ID:EM9BsW9G
紋切り型公式主義に拠る敗北として多田正行に斬られそう
次の段落わからない
ごめんなさい
"〜"の部分はとりあえず抜いてます
今日は頭が回らない


2.Rhode Island boycotted the Constitutional Convention.
America’s littlest state had a big independence streak. Rhode Island, distrustful of a powerful federal government, was the only one of the 13 original states to refuse to send delegates to the Constitutional Convention. It was a decision that rankled even the normally temperate George Washington, who wrote in July 1787 that “Rhode Island … still perseveres in that impolitic, unjust, and one might add without much impropriety scandalous conduct, which seems to have marked all her public councils of late.” On the condition that a Bill of Rights be included, Rhode Island became the 13th state to ratify the Constitution on May 29, 1790, more than a year after Washington was sworn in as president.
Constitutional Convention憲法制定会議
大会、協議会の意味のconventionですね
ただ憲法関連の文では「慣例法」という言葉があるからその意味にも注意
boycott Oへの参加を拒否する
have a independence streakは
have a streak of independenceの意味かな
それならたくさん類例がある
どういう意味かな
また後で考える
be distrustful Oを信用しない
ここは分詞構文のBeing省略
onlyにto refuseが呼応している
13 original states
アメリカ合衆国は最初13州で国旗の横線が13本なのは有名な話
50の星は現在の州の数を表している
It was a decision
ここで倒置と同様に"誤読"のポイントになりやすい強調構文(it-分裂文)を考慮するのはあまりセンスがない
it-分裂文の焦点にa[an] 名詞が来るのは稀なためitが代名詞あるいはit=thatやit=toを予想する
今回はthat節の主語が一見存在しないように見える
そのためthatが接続詞かつ主語の代名詞
つまり関係代名詞と判断
itは前の内容を受けた代名詞である
ここは文構造的には分裂文と取れそうなところであるが、焦点部にaが基本的に来ないことを知っていると悩まずに済む
ここらへんは難関大入試の和訳で出しても苦しむ人かなり出てくるのでは?(舐めすぎ?)
rankle Oを苛立たせる、苦しめる
temperate穏やかな
impolitic浅はかな
unjust不当な不法な
on the conditionについては>>117の通り
ratify Oを承認する
to ratify はthe 13thに呼応している
more than a yearは塊でafter に掛かる
be sworn in就任の宣誓をする
0136英文多読者 ◆iiDSZQODZc
垢版 |
2021/02/19(金) 19:07:26.94ID:EM9BsW9G
""のところを解説できる人いたら代わりに頼む
頭が回らないから僕は明日考えます
0137名無しなのに合格
垢版 |
2021/02/19(金) 19:11:18.54ID:ZaK6a/Hu
>>134
いや、みんな彼女のこと好きだけど特におばあちゃんは次に何をあげれば良いか分からない(=なんでもあげてしまう)ほど大好きってこと
most of allもちゃんとハマる
0138名無しなのに合格
垢版 |
2021/02/19(金) 19:11:40.01ID:ZaK6a/Hu
なんかID変わったけど
0139英文多読者 ◆iiDSZQODZc
垢版 |
2021/02/19(金) 19:13:23.14ID:EM9BsW9G
>>137
なら前置詞じゃなくない?
あれ、俺の前置詞のbutの認識おかしいかな
しらべるかぁ
0140名無しなのに合格
垢版 |
2021/02/19(金) 19:15:28.18ID:Fnwm3iwa
>>131
なるほど、ありがとうございます!
ちなみに、その省略されたliked herはどの部分に補えば良いと思いますか?
0141英文多読者 ◆iiDSZQODZc
垢版 |
2021/02/19(金) 19:17:06.79ID:EM9BsW9G
やっぱり前置詞だとしたら
おばあちゃんは好きじゃないってことにならないか?

>>140
grandmother liked her
0143名無しなのに合格
垢版 |
2021/02/19(金) 19:20:12.22ID:Fnwm3iwa
>>141
お答えいただきありがとうございます!
文末ではなくてgrandmotherの後ろの方が適切なのですね

grandmotherの後ろに補うと先行詞の候補が2つ考えられてしまうので、少し不安を感じていました
しかし、大変参考になりました
0144名無しなのに合格
垢版 |
2021/02/19(金) 19:23:00.35ID:Fnwm3iwa
ちなみに、先ほど申し上げた「別スレ」は↓のスレのことでした
>>130の一文の文法的解釈をめぐって、大論争になってしまっていたようなので

英文解釈参考書スレッドpart27
https://lavender.5ch.net/test/read.cgi/english/1611282871/
0146名無しなのに合格
垢版 |
2021/02/19(金) 19:34:35.76ID:aBiR6N1Y
自分のレス論外で草
おばあちゃんはただ好きなだけじゃないっていう強調かなと思ったんだけどなぁ
every〜butのパッと見の前置詞感すごい
まあ省略って考えるのが普通か…
0147英文多読者 ◆iiDSZQODZc
垢版 |
2021/02/19(金) 19:38:39.74ID:EM9BsW9G
>>146
たぶんね
まぁそういう修辞技法があるって言われたら
そうなのかもって信じてしまうけどね
0149名無しなのに合格
垢版 |
2021/02/19(金) 19:50:27.67ID:O6X5GKTD
伊藤和夫にドヤされるのが目に浮かぶわ
0150名無しなのに合格
垢版 |
2021/02/19(金) 19:50:39.30ID:O6X5GKTD
またID変わった
0152英文多読者 ◆iiDSZQODZc
垢版 |
2021/02/19(金) 19:59:14.00ID:EM9BsW9G
西きょうじのポレポレにも越前敏弥の日本人なら必ず誤訳する英文にも誤訳はある
それだけ精読は難しい
0153英文多読者 ◆iiDSZQODZc
垢版 |
2021/02/19(金) 20:34:08.35ID:EM9BsW9G
streakに関しては
have a streak性質を持つ

bigとindependenceっていう単語で説明が加わっているって感じっぽいね
0155英文多読者 ◆iiDSZQODZc
垢版 |
2021/02/19(金) 22:53:26.73ID:XpycbFxp
""に関してコメントもらいました
解決
そこらの子供向けの文法書だけじゃ足らないね でも英文解釈教室には書いてた
読めていなかった
0157英文多読者 ◆iiDSZQODZc
垢版 |
2021/02/19(金) 22:59:38.40ID:XpycbFxp
難しいよぉ
It is a X thatでItが代名詞のパターン
on condition that S be C
破格の挿入
1パラグラフでこんなに難点が含まれてるのかよ
0158英文多読者 ◆iiDSZQODZc
垢版 |
2021/02/19(金) 23:06:59.94ID:XpycbFxp
形容詞をSVが前後どちらからでも就職できるのかぁ
破格は難しいなぁ
0160英文多読者 ◆iiDSZQODZc
垢版 |
2021/02/20(土) 00:28:22.70ID:Q+2N9Dax
来期、もし英語専門の先生の授業があったら履修して質問しようかな
0164英文多読者 ◆iiDSZQODZc
垢版 |
2021/02/20(土) 01:10:35.96ID:Q+2N9Dax
これ読みます
https://www.timeforkids.com/g56/blast-past-2/


A conch shell discovered 90 years ago in France is now thought to be the oldest instrument of its kind. Researchers say it's about 18,000 years old.

The shell was found in a cave in France's Pyrenees region in 1931. Archaeologists thought it was an ancient cup. For decades, it sat in a natural history museum in the city of Toulouse. But recently, researchers took a fresh look at the shell. They found that the tip had been broken off to make a mouthpiece and sound holes had been added. When you blow into it, the shell can play three clear notes.

Rasoul Morteza, a composer in Canada, has studied the sounds produced by shells. "It's amazing when there's an object forgotten somewhere," he says, "and suddenly, it comes again into the light."
0165英文多読者 ◆iiDSZQODZc
垢版 |
2021/02/20(土) 01:41:40.77ID:Q+2N9Dax
A conch shell discovered 90 years ago in France is now thought to be the oldest instrument of its kind. Researchers say it's about 18,000 years old.
a conch shellホラ貝
この場合のinstrumentは単なる道具ではなく楽器
The shell was found in a cave in France's Pyrenees region in 1931. Archaeologists thought it was an ancient cup. For decades, it sat in a natural history museum in the city of Toulouse. But recently, researchers took a fresh look at the shell. They found that the tip had been broken off to make a mouthpiece and sound holes had been added. When you blow into it, the shell can play three clear notes.
cave洞穴
位置するの意味のsit
take a fresh look at Oを新たな目で見直す
break A offをちぎり取る
この節のandの並列はhadとhadで単純
note音色
Rasoul Morteza, a composer in Canada, has studied the sounds produced by shells. "It's amazing when there's an object forgotten somewhere," he says, "and suddenly, it comes again into the light."
composer作曲家 ちなみにconductor指揮者
study Oをじっくり調べる、観察する、研究する
観察するの意味のstudyは要注意
設計図をじっくりと観察している様子を描写するのにも使える
come to light明るみに出る
このitはwhen節を指すとも漠然とした状況を指してるともいえるやつかな
itを漠然とした状況表すitかつwhenを「〜を考えると」という根拠を表すwhenと解釈することは厳しそう。amazingなのはwhenの内容だろう

江川の英文法解説より
下のような文のit は実際的にはifたはwhen 以下の内容を指しているが、普通のit と見るか,状況のitとするかの分類は紛らわしい。見方によっては形式主語の it に含めることもできる。しかし,あまり分類にこだわるのは賢明ではあるまい。要するに,英語のitにはさまざま
な用法があることを認め、文の意味がわかればよいのである。
It was a bitter blow when we found out they had been cheating us all along. (彼らが私たちをずっとだまし続けていたことがわかったときは、ひどいショックだった)
0166英文多読者 ◆iiDSZQODZc
垢版 |
2021/02/20(土) 01:43:46.15ID:Q+2N9Dax
癒やしを求めてtfkなのに
少し悩んだ
今日は意味さえわかれば良いという姿勢でいく
that SVと同じ意味を表すhow SVと同じようにこのwhen SVを解釈させてくれ
0170英文多読者 ◆iiDSZQODZc
垢版 |
2021/02/20(土) 02:27:38.85ID:Q+2N9Dax
大昔のやつだから晒す
本心とか関係なく、とりあえずその場で書くためだから内容に関してはかなりてきとー

Should Japan rethink its relationship with the United States?

What Japan is to the United States has been like what children are to their parents. Now that Japan is one of the most developed countries, it is time for Japan to rethink its relationship with the United States and change it to some extent.
First of all, Japan should stop constructing the base of the army of the U.S. It is true that the American force is strong enough to protect us from other powers, but we can't stand the destruction of nature. To build it, the sea of Okinawa is being polluted, because of which coral in Okinawa is being endangered. We should conserve coral. If we lose it, we will never regain it.
Second, Japan should resolve the situation that American soldiers who have committed a crime in japan are difficult for the Japanese court to judge. When some American soldiers raped a little girl in Japan, who was an elementary school student then, it took a lot of time to bring them to the court of Japan. Japan should keep people from being injured in Japan.
Third, Japan should refrain from buying arms from the U.S. Japan has many problems to spend a lot of money on. We should improve the lives of the old who suffer from poverty and loneliness. We should invest in schools to help students study hard in a good setting.
Since Japan is no longer a small country, Japan doesn't have to try to please the U.S. and should behave as an independent country.
0175英文多読者 ◆iiDSZQODZc
垢版 |
2021/02/20(土) 17:06:08.70ID:3iE1CyJf
3.Some big names were absent from the Constitutional Convention.
When Thomas Jefferson gushingly called the Constitutional Convention delegates “an assembly of demigods,” he wasn’t being full of himself. Jefferson was not among the founding fathers who gathered in Philadelphia; he was in Paris serving as minister to France. John Adams was also abroad, serving as minister to Great Britain. Samuel Adams, John Hancock and Patrick Henry--who turned down an invitation because he “smelt a rat in Philadelphia, tending toward the monarchy”--also did not participate.
gushinglyおおげさに
be full of oneself傲慢である
ここでbeの進行形が使われていることに注意が必要
be being 〜で〜か言動をする、〜な振る舞いをするの意味である
すなわちうぬぼれた発言をしたわけでないという意味
founding father建国の父
service as 〜として働く
これは進行形ととってもいいけど、間にin Parisがあるし、大事なのは「Philadelphiaにいなかったこと」だから分詞構文で取るのが自然
ここのministerは少し悩むところ
名詞と取るのが自然
役職を表す場合は可算名詞の単数形ですら冠詞が不要になる
関係代名詞のasであれば時制がおかしい
ministerはここでは断定できないがおそらく大臣の意味 牧師などの意味もあることに注意
ただminister toとはなんなのか
動詞ならばminister to Oに仕えるの意味がある
ここから類推すると
a minister to Oに仕える大臣
などと解すればよさそう
turn A downを断る
smell a rat胡散臭いと思う、不審を抱く
tend toward Oにつながる
monarchy君主制
つまり、君主制につながるのではないかという不審を抱いたということ
0176英文多読者 ◆iiDSZQODZc
垢版 |
2021/02/20(土) 17:07:35.11ID:3iE1CyJf
Johe Adams was also abroad, serving as minister to Great Britain.
を見て先の分詞構文であるという読みの正しさを確認する
0177英文多読者 ◆iiDSZQODZc
垢版 |
2021/02/20(土) 17:47:39.48ID:3iE1CyJf
4.Attendance was spotty.
When the Constitutional Convention opened on May 14, 1787, only delegates from Pennsylvania and Virginia were present. It wasn’t until May 25 that a quorum of seven states was achieved. Weather--ever the convenient excuse--was blamed for the tardiness, but the convention was plagued throughout with attendance issues. While James Madison boasted that he never left the proceedings for more than “a casual fraction of an hour,” his fellow delegates were not as fastidious. Nineteen of the 74 delegates to the convention never even attended a single session, and of the 55 delegates who did show up in Philadelphia, no more than 30 stayed for the full four months. New Hampshire’s delegation arrived two months late, by which time two of New York’s three delegates had left in opposition to the proceedings, leaving just Hamilton behind and depriving the state of a quorum to vote. Thus, Washington wrote that the Constitution was signed by “11 states and Colonel Hamilton.”
spottyむらのある
It wasn't until X that SV.
この種のuntilを使う構文は複数存在するが、否定の範囲を把握するのはかなり難しい
伝えたいことは否定ではなく肯定の意味である
次のブログが参考になる
@抜く h@ttps://tmrowing.haten@ablog.co@m/entry/20130329?a@mp=1&__twitt@er_impression=true
俺もちゃんと覚えてないので読み直します
quorum議決に必要な定足数
ever the romanticは「いつもロマンチックな人」のいみ
つまり天気がいつも便利な言い訳
ということ
be blamed for Oの原因とされている
tardiness遅刻
plague Oを悩ませる
be plagued with Oに悩まされる苦しめられる
throughout初めから終わりまで
attendance issuesは出席に関する問題ということ
boast that SVを自慢する
ボースティングって言うよね
a fraction of Xのわずか
にcasualが掛かっている
casualは偶然の、時々の、のどちらかだろう
ほぼ同じか
fastidious非常に注意深い、潔癖な
この同等比較では比較対象が省略されているが、as James Madisonと補えばよい
leave the proceedingsは前後から推測すれば
一連の会議にほとんど遅刻したことがないという意味だろ
欠席に関することかとも思えるが、であればhourではないだろう
後で意味を調べる
次の文のnever even...のところは
一階すら出席しなかったの意味
session集まり
of the 55 delegatesはどういう意味だろう
残りの55人ならばthe rest of〜ではないのか?この構文だと19 of the 55 delegatesとしか構文が取れないのではないだろうか
no more thanは少なさを強調する表現
delegation代表団代議員団
by which timeは関係形容詞
関係詞には関係代名詞、関係副詞、関係形容詞があることに注意
in opposition to Oに反対で
leave A behindを置き去りにする

やーめた
次の英文
0178英文多読者 ◆iiDSZQODZc
垢版 |
2021/02/20(土) 17:50:00.23ID:3iE1CyJf
意味の把握はできても
構文や辞書内の単語の意味を指摘できないところがでてくる
0180英文多読者 ◆iiDSZQODZc
垢版 |
2021/02/20(土) 20:07:16.01ID:3iE1CyJf
イソップ物語読もうかな
英文解釈クラシックでも取り上げられてた

http://read.gov/aesop/106.html

An old Lion, whose teeth and claws were so worn that it was not so easy for him to get food as in his younger days, pretended that he was sick. He took care to let all his neighbors know about it, and then lay down in his cave to wait for visitors. And when they came to offer him their sympathy, he ate them up one by one.

The Fox came too, but he was very cautious about it. Standing at a safe distance from the cave, he inquired politely after the Lion's health. The Lion replied that he was very ill indeed, and asked the Fox to step in for a moment. But Master Fox very wisely stayed outside, thanking the Lion very kindly for the invitation.

"I should be glad to do as you ask," he added, "but I have noticed that there are many footprints leading into your cave and none coming out. Pray tell me, how do your visitors find their way out again?"

Take warning from the misfortunes of others.
0181英文多読者 ◆iiDSZQODZc
垢版 |
2021/02/20(土) 20:23:51.74ID:3iE1CyJf
精読の指南

An old Lion, whose teeth and claws were so worn that it was not so easy for him to get food as in his younger days, pretended that he was sick. He took care to let all his neighbors know about it, and then lay down in his cave to wait for visitors. And when they came to offer him their sympathy, he ate them up one by one.
・were so worn:この時点でso... thatの類を予測し、thatでその正しさを確認する
・so easy: so... thatでも同じ話であるがso...「それほど...」という意味であり、その比較対象がなければ「とても…」と解釈する。ここでもthatやasなどの「どれ程か」を説明する部分を探しながら読んでいけばasで「キタこれ!」と気付けるわけである。asにはit isが省略されていると考える。クラシックではこの比較はわざわざ指摘されてなかったのが気になる。
The Fox came too, but he was very cautious about it. Standing at a safe distance from the cave, he inquired politely after the Lion's health. The Lion replied that he was very ill indeed, and asked the Fox to step in for a moment. But Master Fox very wisely stayed outside, thanking the Lion very kindly for the invitation.
あまり言うことはなさそう
"I should be glad to do as you ask," he added, "but I have noticed that there are many footprints leading into your cave and none coming out. Pray tell me, how do your visitors find their way out again?"
Prayはやや古い表現でpleaseの意味
クラシックではpleaseに修正されていた
find one's wayは大事な項目
one's wayはかなり生産的な表現であり 
cheat[trick] one's way into A
騙してAに入り込む
beat one's way困難を乗り越えて突き進む
thread one's way縫うように進む
elbow one's way肘でかき分けて進む
pick one's way用心して進む
feel one's way手探りで進む
のようにかなりいろいろな表現がある
Take warning from the misfortunes of others.

終わり
0182英文多読者 ◆iiDSZQODZc
垢版 |
2021/02/20(土) 20:26:20.09ID:3iE1CyJf
クラシックに書いてあることは書かずに自分で気づいたことのみ書きました
クラシックは勉強なるっていうよりも、ビジュアル英文解釈みたいにちゃんと一つのストーリーになってるからいいね
ただ英文解釈教室と比べると遥かに軽いし全然タイプが違う参考書。クラシックであれば数時間で本編は読み切れるが英文解釈教室はそうはいかない
0183英文多読者 ◆iiDSZQODZc
垢版 |
2021/02/20(土) 20:27:49.80ID:3iE1CyJf
次はクラシックに載ってないから配慮せずにどんどんコメントする
A Lion and a Man chanced to travel in company through the forest. They soon began to quarrel, for each of them boasted that he and his kind were far superior to the other both in strength and mind.

Now they reached a clearing in the forest and there stood a statue. It was a representation of Heracles in the act of tearing the jaws of the Nemean Lion.

"See," said the man, "that's how strong we are! The King of Beasts is like wax in our hands!"

"Ho!" laughed the Lion, "a Man made that statue. It would have been quite a different scene had a Lion made it!"

It all depends on the point of view, and who tells the story.
0184英文多読者 ◆iiDSZQODZc
垢版 |
2021/02/20(土) 20:47:09.51ID:3iE1CyJf
A Lion and a Man chanced to travel in company through the forest. They soon began to quarrel, for each of them boasted that he and his kind were far superior to the other both in strength and mind.
[かたく、文語]chance to Vb偶然Vbする
in company with Oと一緒に
quarrel口論する
forは前置詞ではなくandなどと同じ等位接続詞
等位接続詞はFanboys接続詞とも呼ばれる(等位接続詞の頭文字)
mindをどう取るか
知性で良いのか
mindには精神や知性の意味がある
知性はemotionと対比されるものであり、精神はbodyと対比されるものである。
精神、勇敢さなどのことではないだろうか
kindは種族の意味
Now they reached a clearing in the forest and there stood a statue. It was a representation of Heracles in the act of tearing the jaws of the Nemean Lion.
clearing空き地
there stood a statueはThere be Xでbe動詞の仮にstandが使われている。このように自動詞であれば様々なものが使われる。
in the act of Ving最中の現場の
tear Oを引き裂く
"See," said the man, "that's how strong we are! The King of Beasts is like wax in our hands!"
辞書に見当たらないが、ここは
我々人類の手にかかれば百獣の王も蝋のよう(に容易く切り裂ける)。の意味だろう
"Ho!" laughed the Lion, "a Man made that statue. It would have been quite a different scene had a Lion made it!"
" "VSという引用符の後によく起こる倒置
would の時点で仮定法の可能性が浮かび
had a Lion made itが倒置によるifの省略であると即座にわかる
It all depends on the point of view, and who tells the story.
0185英文多読者 ◆iiDSZQODZc
垢版 |
2021/02/20(土) 20:47:43.87ID:3iE1CyJf
次はこれ読みます
A Milkmaid had been out to milk the cows and was returning from the field with the shining milk pail balanced nicely on her head. As she walked along, her pretty head was busy with plans for the days to come.

"This good, rich milk," she mused, "will give me plenty of cream to churn. The butter I make I will take to market, and with the money I get for it I will buy a lot of eggs for hatching. How nice it will be when they are all hatched and the yard is full of fine young chicks. Then when May day comes I will sell them, and with the money I'll buy a lovely new dress to wear to the fair. All the young men will look at me. They will come and try to make love to me,but I shall very quickly send them about their business!"

As she thought of how she would settle that matter, she tossed her head scornfully, and down fell the pail of milk to the ground. And all the milk flowed out, and with it vanished butter and eggs and chicks and new dress and all the milkmaid's pride.

Do not count your chickens before they are hatched.
0186英文多読者 ◆iiDSZQODZc
垢版 |
2021/02/20(土) 21:23:13.18ID:3iE1CyJf
A Milkmaid had been out to milk the cows and was returning from the field with the shining milk pail balanced nicely on her head. As she walked along, her pretty head was busy with plans for the days to come.
milkが動詞として用いられている
搾乳することであることはすぐにわかるだろう
pailバケツ
balance Oのバランスを保つ
pretty
be busy witk Oで忙しい
plan for Oの計画
days to come はSV関係であり名詞を不定詞が修飾する型であるが、当然for the daysは意味上の主語を表すforではない
X to comeはよくある表現で既に↑にもたくさんある
prettyはどのような意味だろう
単純に「かわいい」でよいのだろうか、「ずる賢い」の意味もあるようである
後ほどわかるかもしれない
"This good, rich milk," she mused, "will give me plenty of cream to churn. The butter I make I will take to market, and with the money I get for it I will buy a lot of eggs for hatching. How nice it will be when they are all hatched and the yard is full of fine young chicks. Then when May day comes I will sell them, and with the money I'll buy a lovely new dress to wear to the fair. All the young men will look at me. They will come and try to make love to me,but I shall very quickly send them about their business!"
形容詞の並列に関しては英文解釈教室の改訂版にも関わった柴田耕太郎のサイトが参考になる 気になったら調べてみて
muse考え込む
churn Oを攪乳器でかきまわす(バターを作ったりする)
creamとto churnはVO関係でto不定詞は形容詞として機能
普段の読解で不定詞を単に形容詞的用法と解釈するだけでは心許ない
SV,VO,同格 いずれであるか判断したい
The butter I make I will take to market
OMSVの目的語の前置詞
関係詞がついてるためなかなかに面白い構造
with the money I get for itも前置詞句の前置であり、前置という点で同じである
英語において同じ構文が繰り返されることは往々にして見られる
get A for Bのforは交換を表す
hatchは「孵る」あるいは「孵す」の意味
need ingのようにeggsがhatchの意味上の目的語になってるのではないだろうか、もちろんこれは名詞的動名詞としてである。
ちなみに、S need VingのVingは名詞的動名詞であり、形容詞修飾は可能だが、副詞による修飾は不可能なようである
chickひよこ、可愛子ちゃん
後者の意味を最初に知った
chickenのenが消えた形だろう
fineはおそらく「見た目のいい(かわいいとか)」の意味かな?元気の意味であればで叙述用法がふつう
fairは品評会、博覧会、移動遊園地などさまざま
make loveは性行為をするの意味であるがここでは不自然
make love to Oに言い寄る、口説く
という古い表現が存在する
shallはおそらく「するでしょう」の意味でよいだろう
send A about A's business:Aを追い返す
0187英文多読者 ◆iiDSZQODZc
垢版 |
2021/02/20(土) 21:35:33.81ID:3iE1CyJf
As she thought of how she would settle that matter, she tossed her head scornfully, and down fell the pail of milk to the ground. And all the milk flowed out, and with it vanished butter and eggs and chicks and new dress and all the milkmaid's pride.
asは理由を表す
toss Oを揺さぶる
settle Oを解決する
scornfully軽蔑したように
このようにfall downのdownを前置できるのだろうか
初めて見たかも
and with...以降はMVSの倒置
このwithは原因を表す
Her face was red with anger.と同じでよいのだろうか
Do not count your chickens before they are hatched.
0190名無しなのに合格
垢版 |
2021/02/20(土) 22:13:21.42ID:PlNnfbeW
透視図読んだことある?
0191英文多読者 ◆iiDSZQODZc
垢版 |
2021/02/20(土) 22:25:02.70ID:3iE1CyJf
>>190
ないや

大学受験のときは基本はここだを8割ぐらい身につけた段階で関関同立とかセンターではほとんど構文で困らなくなって(と勘違いしてたんだろうな)ストップした
受かってから教習所の待ち時間でビジュアル英文解釈読んで
あとは英文解釈教室を全部じゃないけど軽く読んだぐらい
ビジュアルは基本はここだとか基礎の英文法がだいたい身についてたからサラーと読めて簡単だったけど英文解釈教室は時間がかかる

透視図とポレポレ、〜の技術、熟考は読んでない
最後の2冊は持ってるから少し目を通したことはあるけどね
0193英文多読者 ◆iiDSZQODZc
垢版 |
2021/02/20(土) 22:51:23.34ID:3iE1CyJf
先行詞明示のthatといえば合衆国独立宣言の冒頭の次の英文
"We hold these truths to be self-evident, that all men are created equal, that they are endowed by their Creator with certain unalienable Rights, that among these are Life, Liberty and the pursuit of Happiness."
これ読んだときは先行詞明示とか知らず
these truths「これらのってどれなの。わからんから後ろに書いてないとおかしい。」という発想で同格のthatなどを予測して読んだはず。
まぁこれは関係詞じゃなくて同格だけど、要は同じこと。
0194名無しなのに合格
垢版 |
2021/02/20(土) 22:52:42.55ID:PlNnfbeW
そっか
受験生時代に時間余って読み込んでたけどネットでよく言われてるほど別に難しくないと思うんだよね
むしろ薄いながら省略とか倒置とかめっちゃ丁寧に解説してて英語に時間かけられない受験生におすすめできる良書って雰囲気すらある
京大志望以外やる必要ないとか言ってる人は噂だけで言ってんのかなって思って
読んだことあったら感想聞いてみたかった
0196英文多読者 ◆iiDSZQODZc
垢版 |
2021/02/20(土) 23:05:13.25ID:3iE1CyJf
英文解釈の参考書って超上級の物を除けば弱者のための裏ワザ感強いよな
0197名無しなのに合格
垢版 |
2021/02/20(土) 23:07:24.18ID:YCUAz/1v
2週間でTOEIC500点から600点に上げる方法教えてよ
0198英文多読者 ◆iiDSZQODZc
垢版 |
2021/02/20(土) 23:08:36.23ID:3iE1CyJf
>>197
余裕
単語詰める
形式を把握して時間配分を決める
Part 7の最後の方は塗り絵前提で
Part 5,6でしっかり取る

これで100〜200はすぐあがる
0199名無しなのに合格
垢版 |
2021/02/20(土) 23:10:27.63ID:YCUAz/1v
>>198
あざーす 意外と簡単にいけるのか
でも単語の勉強はしないとダメなのか
TOIECシス単とか使ってもいいか
0200名無しなのに合格
垢版 |
2021/02/20(土) 23:13:18.85ID:YCUAz/1v
ミス
TOIEC→TOEIC
スペル覚えれねーわ
0201英文多読者 ◆iiDSZQODZc
垢版 |
2021/02/20(土) 23:14:24.85ID:3iE1CyJf
>>199
見たことがない
どういう経緯での500なのか
LRの配分はどうなのか
高校文法はできるのか

いろいろ関係するけどTOEICは割とみんなすぐ100〜200はあがる
上がらない人もいるけど
TOEIC対策ばかりしてきての500ならば上がりにくいし
高校英語しっかりやってきてTOEIC500ならば上がりやすい

しっかり本気でやれば大丈夫
まぁしっかり本気でやらなくとも形式になれるだけでもあがったりする
解き方とかも決めちゃいな
形式決まりきってるんだから
(ちまたの参考書に載ってるPart 5の解き方は大嫌いだからおすすめしないけど)
0203英文多読者 ◆iiDSZQODZc
垢版 |
2021/02/20(土) 23:16:27.28ID:3iE1CyJf
大学入試であれば形式も変わったりするし、複数大学受けるから形式に合わせていくの嫌われるけど
TOEICは形式絶対に勝手に大幅変更はないから短期間で点数だけ上げたいなら完璧に形式に合わせていくべき
0204名無しなのに合格
垢版 |
2021/02/20(土) 23:18:28.36ID:YCUAz/1v
>>201
センターは9割取れたし
国立二次も8割くらい取れたのに
大学入ってサボりまくってこのザマよ

LとRの配分はわかんねーわ
成績もらったらポイ捨てしてたからな
そういやTOEIC対策やったことねーな
形式慣れればいいんか
0205英文多読者 ◆iiDSZQODZc
垢版 |
2021/02/20(土) 23:24:04.03ID:3iE1CyJf
>>204
ま?そんなことあるんか
受験生かと思ったわ 
文法できるならTOEICで悩むことは一切なくて単純な知識と速さの問題だけだろうから
公式問題集ひたすらやるのでもいいだろうし
「3週間で攻略TOEIC L&Rテスト900点!」みたいな本で軽く解き方とか勉強するのもあり
文法忘れてるなら勉強したほうがいいけどセンターで9割取れたなら受験参考書を軽く見直すだけでいいんじゃないかな
Lはしらん
でも公式問題集は本番とナレーターが同じだから使えるはず
公式問題集買うなら最新版な
俺自身、Rは485だけどLはそこまでよくないからなんともいえん
0206名無しなのに合格
垢版 |
2021/02/20(土) 23:30:06.54ID:YCUAz/1v
>>205
お前オススメの「3週間で攻略TOEIC L&Rテスト900点!」買ってみるわ
あと公式問題集は古いのしか持ってないからそれ使うわ
親切に教えてくれてありがとうな
0207英文多読者 ◆iiDSZQODZc
垢版 |
2021/02/20(土) 23:30:39.61ID:3iE1CyJf
>>206
古いのは形式少し違ったりナレーターが古い可能性あるから注意な まぁそこまで古くないだろうから心配ないか
0208英文多読者 ◆iiDSZQODZc
垢版 |
2021/02/20(土) 23:37:29.84ID:3iE1CyJf
イソップ物語ってかなりたくさんあるな
いつか時間を作って読み切りたいな
0211英文多読者 ◆iiDSZQODZc
垢版 |
2021/02/21(日) 00:09:41.31ID:KT7lDIxU
これ読みます

https://greatist.com/live/science-backed-ways-to-make-your-partner-orgasm

The Wizard of Os: 3 Proven Ways to Give Your Female Partner an Orgasm

Three things make your toes curl:

bloodcurdling embarrassment
wearing pointy jester shoes
Oh my goodness, how are you doing that with your tongue?
Despite having the only organ in the human body completely dedicated to pleasure, people with clitorises can find orgasms to be mysteriously out of reach.

Some of those looking to rock their partner’s entire world claim that the clitoral orgasm is a mystery that ranks alongside the Bermuda Triangle or where that missing sock goes on laundry day.

When mystery calls, though, science invariably answers. And this time, it’s wearing the sexiest lab coat imaginable.

Sex researchers have been asking some smart questions. With their help, we parted the legs of truth and revealed practical tips to help your partner come (to some positive conclusions about your technique, obviously).

And don’t worry, we’re not leaving folk with penises out -- they tend to have an easier time of it, but that’s not true for everyone.

One study from 2019 suggested based on data that over 90 percent of people with penises regularly orgasm during sex, but that the figure is 50 percent for people with clitorises.Trusted Source

Well, that just won’t do, now, will it? Sex should be fun, sensual, and safe for everyone who tries it. And that includes whoever’s in your bed right now! (Okay, tip number one: Put the articles away and get to it.)

We’ve got a few tips that will have your partner soundproofing their bedroom as a matter of urgency. You’ll graduate from our walkthrough magna cum loudly.

While we’re on this sticky subject, we explain the awesome health benefits of a good, old-fashioned orgasm, what could be blocking the road to O-town, and how to get there.
0212英文多読者 ◆iiDSZQODZc
垢版 |
2021/02/21(日) 00:19:34.26ID:KT7lDIxU
The Wizard of Os: 3 Proven Ways to Give Your Female Partner an Orgasm
オズの魔法使い(The Wizard of Oz)のパロディー
オーガズムの魔法使いということ
a proven way実証された方法
wayは同格のto不定詞を使える名詞

Three things make your toes curl:
make somebody's toes curl:〜をいかせる
つまさきをカールさせる→オーガズムに達するということ
下の写真のようなことだろう
https://i.imgur.com/Fg6ol4B.png
0215英文多読者 ◆iiDSZQODZc
垢版 |
2021/02/21(日) 00:30:50.14ID:KT7lDIxU
日本の新聞ではなさそうたけど
割とTIMEやThe New York Timesにはセックスに関する記事があったりする
0216名無しなのに合格
垢版 |
2021/02/21(日) 01:26:46.22ID:blSE4oCl
英語の雑誌とか新聞とか定期購読してるものあるの?ネット記事メイン?
0217名無しなのに合格
垢版 |
2021/02/21(日) 01:29:44.69ID:blSE4oCl
英検取りたいけど2次が面倒臭すぎる
自宅からオンラインで出来れば良いのに
0220英文多読者 ◆iiDSZQODZc
垢版 |
2021/02/21(日) 01:54:30.29ID:KT7lDIxU
次は歌をいきます
https://youtu.be/vhn2Thi3dRw
Miyachi BAD & ブジ

Ninja, Rapper
Medicine smoke weed
Welcome to my house
The walls are bloody red
Asian people left and right
I write my feelings down and sing it
the way I speak is Wasabi
Kill em and leave the body sideways
You and I breathe the same air
But, you are stupid, pussy
Can you walk in my shoes?
Can you find your courage?
Have you lost your courage?
the glow of a god around my body
Born in America, but around Kyushu
Usually I don't try this
But in my spare time I change the language
But in my spare time change the language
I started in high school
Then I practiced in college
Up until now in Philly and New York
Soon after I'll be in Tokyo
6 years from now globally broadcasted
Im going global
Leave the hood straight to the airport
My dreams are huge
But first want to lessen my enemies
Every day talking with my people about schemes to get money
voicemail because I don't got time to talk
Check the time, stupid
People dying around the world
But I will live long
my fever won't go down
"So please go to the hospital, not worth the pain"
I haven't eaten, but have drugs
Squad, squad around me, ill never ever change my brothers
0221英文多読者 ◆iiDSZQODZc
垢版 |
2021/02/21(日) 01:56:45.93ID:KT7lDIxU
yo man that shit was hot right?
I'm fucking korean
they call me a tub but i done my lines
they call me the son but the sun will rise
coz i can body any beat
i want and colonize the country from the other side pockets heavy swollen ill be packing drow and duffs
pops i apologize
was never meant for police i asked the revolution not be televised Miyachi im popping and you know that your mans'll be takin them cold baths coz he never payed for them peacoat my profit, economy on track you look at my face you look at my eyes you probably get used to this young jack coz was probably serving ramen at the spot you stopped with your thot to get lunch at huh put me a rose they think that asian a lane of its own
i told the jew that the platinum and gold i told them throw me with jay and the cole both of me wanted the foreign and native the king and the core and the devil ignored
hold up I'm catching bodies with nobody watching me
autopsy call it the morgue
0222英文多読者 ◆iiDSZQODZc
垢版 |
2021/02/21(日) 02:17:58.93ID:KT7lDIxU
Ninja, Rapper
Medicine smoke weed
Welcome to my house
The walls are bloody red
壁が可算名詞ってのは盲点かもね
Asian people left and rightこれは問題ないが日本語と英語で順番が異なることもよくある
左右もright and leftのほうが多いみたい。ちなみに実質的に全方位の意味
需要と供給はSupply and Demand
I write my feelings down and sing it
感情の意味ではfeelingsと複数形であることに注意
the way I speak is Wasabi
the way (that) SVのように省略できることに注意。省略されることが基本で、thatやin whichそして稀にhowが入る 
ちなみに
I think it's a shame the way the old man died alone. その老人が1人ぼっちで亡くなったのは大変遺憾だと思う
のようにthat節と変わらない機能も持つ
これはhowにも言えることであり共通点である
Kill em and leave the body sideways
'emはthemのこと
sidewaysの語末のsは複数形ではなく、副詞属格である
upstairsやnowadaysのsも複数形ではなく副詞であることのマークと捉えて良い
You and I breathe the same air
But, you are stupid, pussy
Can you walk in my shoes?
in one's shoes:〜の立場に立って
という修辞的表現
俺の靴で歩けるか→俺の立場でやっていけるか
Can you find your courage?
Have you lost your courage?
the glow of a god around my body
glow輝き
Born in America, but around Kyushu
Usually I don't try this
But in my spare time I change the language
I started in high school
Then I practiced in college
Up until now in Philly and New York
Soon after I'll be in Tokyo
6 years from now globally broadcasted
Im going global
Leave the hood straight to the airport
My dreams are huge
But first want to lessen my enemies
Every day talking with my people about schemes to get money
このtalkingは通常の動名詞ではない
every dayが掛かっていることから動詞的動名詞ではなく名詞的動名詞であることがわかる
scheme企み陰謀計画
voicemail because I don't got time to talk
Check the time, stupid
People dying around the world
But I will live long
副詞のlongも存在することに注意
my fever won't go down
擬人法のwon't
"So please go to the hospital, not worth the pain"
I haven't eaten, but have drugs
Squad, squad around me, ill never ever change my brothers


おもしろところはコメントしきったかな
終わり
0225英文多読者 ◆iiDSZQODZc
垢版 |
2021/02/21(日) 15:16:44.17ID:+tK7+w+x
東京外国語大学の自由英作の模範解答を読もうかな

When learning a second language, having an understanding of the culture in which
that language is used is as important as mastering its vocabulary and grammatical rules.
While listening, speaking, reading, and writing are all essential in second language learning,
successful communication with a native speaker is not possible without an understanding
of the everyday “Little c” culture.
Common English expressions can be a source of confusion and anxiety for non-native
speakers. Casual greetings like, “What’s up?”, “What’s happening?”, “What’s
new?” are essentially identical in their purpose, to politely inquire about someone’s state
of being, like “How are you?”, but require a different response. While most appropriate
response to any of these questions is “Not much,” an English language learner who has
only studied the more formal “How are you?” greeting, may be confused and incorrectly
respond with, “I’m fine, thanks.” This common mistake exemplifies the importance of
understanding “Little c” culture in language learning.
A lack of understanding of the ways in which people actually communicate in a
foreign language can result in confusion and embarrassment. For this reason it is essential
that we as English language learners expand our learning beyond our textbooks and gain
a deeper understanding of the cultural context of English communication.
0228英文多読者 ◆iiDSZQODZc
垢版 |
2021/02/21(日) 20:47:20.03ID:9Ot/le2b
An Ox came down to a reedy pool to drink. As he splashed heavily into the water, he crushed a young Frog into the mud.

The old Frog soon missed the little one and asked his brothers and sisters what had become of him.

"A great big monster," said one of them, "stepped on little brother with one of his huge feet!"

"Big, was he!" said the old Frog, puffing herself up. "Was he as big as this?"

"Oh, much bigger!" they cried.

The Frog puffed up still more.


"He could not have been bigger than this," she said. But the little Frogs all declared that the monster was much, much bigger and the old Frog kept puffing herself out more and more until, all at once, she burst.

Do not attempt the impossible.
0229英文多読者 ◆iiDSZQODZc
垢版 |
2021/02/21(日) 21:01:37.51ID:9Ot/le2b
この前と同じサイトから
http://read.gov/aesop/002.html

An Ox came down to a reedy pool to drink. As he splashed heavily into the water, he crushed a young Frog into the mud.
Ox雄牛
reedy葦で生い茂る
reed葦は有名かな
splash into Oへとバシャンと飛び込む
heavily激しく

The old Frog soon missed the little one and asked his brothers and sisters what had become of him.
oneはFrogのことであるが、もう少し具体的には、the little oneは前述のa young Frogのこと
What BECOME of X?:Xはどうなるのか
この場合は彼はどうなったのかの意

"A great big monster," said one of them, "stepped on little brother with one of his huge feet!"

"Big, was he!" said the old Frog, puffing herself up. "Was he as big as this?"
Big, was he!はCVSの倒置
語順の変化とは主に右方に新情報ないし重要情報を置くために発生するが、今回はBigを強調するためだろうか。そのために, があるのだろうか
でもSheは見てないはずだから強調というよりは別の表現を使うべきかな
puff A upをふくらませる

"Oh, much bigger!" they cried.

The Frog puffed up still more.
Sheのこと
"He could not have been bigger than this," she said. But the little Frogs all declared that the monster was much, much bigger and the old Frog kept puffing herself out more and more until, all at once, she burst.
彼はこれ以上デカかったはずがないの意味
all at once突然、一斉に
Do not attempt the impossible.
the 形容詞 〜な物、人
0231英文多読者 ◆iiDSZQODZc
垢版 |
2021/02/22(月) 00:09:23.71ID:uhyGespQ
大阪公立高校C問題解きます
北野高校行きたい中学生はこれぐらい解きます
0233英文多読者 ◆iiDSZQODZc
垢版 |
2021/02/22(月) 00:26:47.17ID:uhyGespQ
1
(1)The story (my uncle told to his children) was very interesting.
was very interestingから主語がstoryだと確定してイ・エから絞る 易
(2)I don't know (which way is better to solve the problem).
これは難易度高いのでは?tough構文のように見えてtough構文ではない
これは同格のto不定詞が分離して修飾している状況か? 
(3)The fact (discovered by the scientist was useful) to solve many problems.
同格かと思いきや違う
SVがかなり離れているため少し難しいか
(4)He stopped exchanging e-mails with (me without explaining the reason).
without Vingに習熟していればよい
(5)Follow the (advice from the people who know) the area well.
areaの意味はなんとなくで中学生でも分かるかな
(6)I asked my aunt (growing vegetables how to choose )fresh ones.
これは相当いじわるなのでは? 
悩んだ 難 英語力というより賢さ
0234英文多読者 ◆iiDSZQODZc
垢版 |
2021/02/22(月) 00:33:26.71ID:uhyGespQ
In 2018, a survey of views on studying abroad was carried out on high school students in four countries about 2,000 students in Japan, about 1,500 students in America, about 3,000 students in China, and about 1,200 students in Korea. For this survey, the high school students answered some questions by choosing one answer from several alternatives.
The following table shows the percentages of their answers to the question: "Do you want to study abroad if it is possible?" To this question, five alternatives were given to the students. Different times to go abroad to study were included in the four alternatives 2 to 5.
First, look at the percentages of the students who chose "No, I do not want to study abroad." We see that?1@?among the percentages of the four countries. And, the percentage of the students in Korea is the lowest.
Next, look at the percentages about the times to go abroad in the four alternatives 2 to 5. In ?2A?, the percentages of the students who chose "while I'm in college" are the highest and they are over 30%. When we look at the percentages about the time to go abroad in ?3B?. we can find a situation that is not soon in the othor three countries: both the percentage of the students who chose "while I'm in high school and the percentage of the students who chose "as soon as I graduate from high school" are more than 15%.
https://i.imgur.com/ZcQUXaQ.jpg
0236英文多読者 ◆iiDSZQODZc
垢版 |
2021/02/22(月) 00:43:50.57ID:uhyGespQ
Proverbs are short sentences passed down from generation to generation and they usually contain advice to people.
There are two proverbs that give us different advice. The first one is "Nothing ventured, nothing gained." This proverb says that we cannot get anything if we don't take a risk. The second one is "Better safe than sorry." This one says that a person should be careful and act safely to avoid regretting later.
Which proverb is more persuasive for you? Write your opinion and reason. After that, write about your experience to support your reason. When you writo your answer, you can express the first proverb as Proverb A, and the second proverb as Proverb B.
これが英作文の問題
0237名無しなのに合格
垢版 |
2021/02/22(月) 01:20:30.35ID:6OnPSUQF
>>236
中学生にここまでやらせるのか…
0238英文多読者 ◆iiDSZQODZc
垢版 |
2021/02/22(月) 01:23:33.44ID:uhyGespQ
Proverb A sounds more persuasive for me. My mother always tells me to give it a try. I know it is the key to success.
まずfor meってあるしI thinkは不要かな
give it a tryでひとつのフレーズ
I make a point of making full use of every chance I have to speak English, thanks to which I can improve my fluency in English. I often use wrong expressions, of course, but I don't care how many mistakes I make. They give me a chance to enhance my English ability.
make a point of Vingでひとつのフレーズ
chance I have to speakは見せかけのhave toという少し難し目の英文解釈のポイント

受験的なところを盛り込んでみた
ゴミだけど許して
0239名無しなのに合格
垢版 |
2021/02/22(月) 01:33:57.68ID:6w+adh3i
>>238
大学受験的な目線で見ると間違った表現を使うことがリスクなのかとか言われそうではある
文法語彙でミスなければほぼ満点が普通だろうけど
0244英文多読者 ◆iiDSZQODZc
垢版 |
2021/02/22(月) 19:05:19.89ID:uyHBgMfn
細江の英文法汎論を読むか
名詞相當文句
Where do you come from?
whereが疑問代名詞として機能
これは現在も使われるし、関係代名詞のwhereも存在する。もちろん前置詞はその目的語に名詞のみならず形容詞や前置詞戻れると読んだことがあるので、そう解すればなんでもよくなるけど。
How far is it from here to there?
hereとwhereが名詞相當文句
He has had ups and downs.  
upsとdownsが名詞相當文句
wisdomではups and downsで「浮き沈み」名詞として記載。ただまぁ形からわかるように副詞が元なのでしょう。
Thou losest here a better where to find.
--Shakespeare.
hereとwhereが名詞相當文句
Every why hath a wherefore.--ibid.


古い二人称単数thou
thou thy(thine) thee thine
thouを主語とする動詞は語尾が-stもしくは-est
今回はlosest←lose

少し調べてみた
here→this place
a better where to find→a better place to find→to find a better place
より良き場所を見つけるために現在の場所を失う の意味のようです
この形はよくあるのかな。現代でも使われうる可能性があるならこれも覚えとかないとやばいかもね
hath: haveの三人称単数直説法現在形

二人称単数という言葉に違和感を抱くかもしれない。Youは「あなた/あなたたち」と習うが本来は複数の意味しかなく、ye(you)とthouがまとめて"you"で表されるようになった。そのためyouに対応するbe動詞は複数に対応するare。元の意味にbe動詞が対応するのは他の代名詞でも見られる。3人称単数のthey(性別を明示しない)もareで対応するし、1人称単数のwe「朕」もareで対応する。
0245英文多読者 ◆iiDSZQODZc
垢版 |
2021/02/22(月) 19:06:55.55ID:uyHBgMfn
一人称単数weはイギリスの法典化されていない憲法について読むときに初めてであった
もちろん王のセリフ
0246英文多読者 ◆iiDSZQODZc
垢版 |
2021/02/22(月) 19:11:28.39ID:uyHBgMfn
mentionとuseについても書いてある
What does marry mean?-Shakespeare.
"marry"は何を意味するのか。
Was is not is.-ibid.
"was"は"is"ではない。
かつてあったということはそれが今も存在するということを意味しないということかな?
0248英文多読者 ◆iiDSZQODZc
垢版 |
2021/02/22(月) 19:42:06.50ID:uyHBgMfn
英語専門なら習うんだろうな
学部選択を後悔してるので、受験生はしっかり決めましょう
0249英文多読者 ◆iiDSZQODZc
垢版 |
2021/02/22(月) 19:45:54.68ID:uyHBgMfn
まず最初に述べられているのが基本事項として
It be 焦点 that 前提.
What 前提 be 焦点.
という構造
これらは文を2つに分けたような形をしているので、前者をit-cleft(it分裂文),後者をwh-cleft(wh分裂文)と呼ぶ

次にprinceが提唱した情報の分類
prince以前から旧情報と新情報の区分けは存在したが、princeは旧情報を更に分類したという話だったはず
0250英文多読者 ◆iiDSZQODZc
垢版 |
2021/02/22(月) 19:52:27.93ID:uyHBgMfn
受験英語の場でも情報構造について触れられることも多いがそれは満足いかない
学校や予備校ではOld information旧情報のうち所与情報のGiven informationばかりが取り上げられて既知情報Known Informationはあまり取り上げられない。というより教師の側があまり把握できていないといったところだろう。そういってよい場面を見たことがある。
Givenは談話の中で既に与えられた情報
Knownは話者が(その実際に拘らず)相手も知っているだろう、世間も知っているだろうと想定する事実のことである
そして
英語では
Old Information→New Informationの順で情報が流れる
0251英文多読者 ◆iiDSZQODZc
垢版 |
2021/02/22(月) 19:56:11.06ID:uyHBgMfn
例えば
The window was broken by Ken.
であればby Kenは新情報である。
He broke the window.
であればHeは旧情報givenである。
クジラ構文の後ろの部分「馬が魚である。という命題の真偽」はKnown Informationと読んでいいと思う。

a. Given information: Information which the cooperative speaker may assume is appropriately in the hearer's consciousness.
b. Known information: Information which the speaker represents as being factual and as already known to certain persons (often not including the hearer)

さっきの説明と微妙に違うけどまぁ同じようなことでしょう
0252名無しなのに合格
垢版 |
2021/02/22(月) 19:59:34.75ID:S3vUum7q
主が読むものに困ってるならダロウェイ夫人で使われてる文法(どこが描出話法だとか)最初の部分だけでいいから解説してほしい、けど、主が自分で勉強してここに載せてくれてる知識もすごくためになるなるんですよね
小説ってやたらと現在分詞で区切るのに、thinking that〜みたいな普通分詞構文なんだからカンマつけるんじゃないのってところにカンマつけないし、とりあえず付帯状況のwithを並列してると捉えてるけど文法がよくわからないです
0253英文多読者 ◆iiDSZQODZc
垢版 |
2021/02/22(月) 20:01:30.96ID:uyHBgMfn
It be 焦点 that 前提
という形は英語のルールからしたら例外的である
新情報→旧情報の順に情報が流れている 
しかし、だからこそ聞き手の注意を引く事ができ、聞き手を引きつけるための技術といえる
It-cleftでは前提はKnown Information
What 前提 be 焦点
の情報の流れは旧情報→新情報であり、また、前提はGiven Informationであるから聞き手にかかる負担が少ないといえる
0255英文多読者 ◆iiDSZQODZc
垢版 |
2021/02/22(月) 20:17:33.58ID:sisp27Yw
次に
聞き手が前提知識を有してない例のit-cleftとかあるけどあとは読んでみて
0256名無しなのに合格
垢版 |
2021/02/22(月) 22:07:31.14ID:S3vUum7q
すいません英語の質問なのですが、模試で、All of this became much less of a joke when,
〜という文があって、このようなことが冗談では済まされなくなったのは〜ときのことだった。と訳されていたのですが、
less +ofってどういう時に使われるのか教えて欲しいです。muchはlessを強めているんだと思います。
0257英文多読者 ◆iiDSZQODZc
垢版 |
2021/02/22(月) 23:37:43.53ID:sisp27Yw
>>256
名詞が含む程度を修飾するためじゃないっけ?

形容詞とか副詞ならそのままでいいけど
名詞だったらより少ないの意味になるからそれを避けるため?
wisdomより
less of an Aの形で; Aは程度性を持つ名詞
The party was less of a success than we had hoped.
0258英文多読者 ◆iiDSZQODZc
垢版 |
2021/02/23(火) 01:00:02.16ID:UTwjDVHf
merriamだとconjunctionと明記されてるね
much less
conjunction
not to mention used especially in negative contexts to add to one item another denoting something less likely
He had trouble paying for a car, much less a high-definition TV.
0260名無しなのに合格
垢版 |
2021/02/23(火) 09:18:52.43ID:VaYMF1F6
>>259
ありがとうございます
0262名無しなのに合格
垢版 |
2021/02/23(火) 23:04:38.39ID:cdwglEf5
わっしょい
0263英文多読者 ◆iiDSZQODZc
垢版 |
2021/02/24(水) 00:34:40.78ID:vQdUTfqP
誰もやってくれなさそうだけど問題

As a result以降の英文を和訳せよ

構造が分かってることが分かればいいから直訳でok 俺もそうする

As a result, U.S. negotiators focused on a bilateral deal with Mexico, the contours of which were announced last week.

Japantimesより

※前後読んでないけど語彙の意味はとりあえず次の意味とする。
a bilateral deal二国間協定
contour輪郭
難しいのこの2つぐらいかな
0264英文多読者 ◆iiDSZQODZc
垢版 |
2021/02/24(水) 01:00:11.90ID:vQdUTfqP
こちらもJapantimesより

This is to do withから和訳せよ

I want to open a free school for drug users and dropouts. Then, I want to do agriculture and farming. I don't mean commercial agriculture, but growing some vegetables and milking cows in a field near my house.
ここから
This is to do with my own experiences. I think I must consider why I needed glue-sniffing and pills and why it was that I was able to quit them.

glue-sniffing:シンナーの吸引
pills錠剤
0267名無しなのに合格
垢版 |
2021/02/24(水) 07:45:46.59ID:snBrjj6X
>>263
結果として、アメリカ政府側の担当官はメキシコとの2国間協定との協議に集中的に入っていたが、先週その協定の大枠が発表された。
いつも見てます!
0268英文多読者 ◆iiDSZQODZc
垢版 |
2021/02/24(水) 08:51:27.25ID:Lb53M5uB
>>263
As a result, U.S. negotiators focused on a bilateral deal with Mexico, the contours of which were announced last week.
ポイントは関係代名詞
前置詞+関係代名詞だけじゃなくて名詞+前置詞+関係代名詞という形もあって主語や目的語として機能する
much of which とかはよく習うかな
今回はカンマがあるからわかりやすいけど
the contoursのところ、カンマのあとから関係詞節が始まっている
the contours of the deal were announced last weekということ
>>267
関係詞のところ完璧
0269名無しなのに合格
垢版 |
2021/02/24(水) 09:49:24.50ID:W9YdVzIi
>>264
これは私自身の経験と関係がある。私はなぜシンナーの吸引と錠剤が必要だったのか、私がそれらを止められたのは一体なぜなのか、考えなければならないと思う。
0270英文多読者 ◆iiDSZQODZc
垢版 |
2021/02/24(水) 13:43:10.16ID:mr3VHFQs
>>264
I want to open a free school for drug users and dropouts. Then, I want to do agriculture and farming. I don't mean commercial agriculture, but growing some vegetables and milking cows in a field near my house.
This is to do with my own experiences. I think I must consider why I needed glue-sniffing and pills and why it was that I was able to quit them.

why it was that I was able to quit them.
がポイント
why was I able to quit them?
これはもちろんI was able to quit them because--.に対応するwh疑問文
wh疑問文は疑問詞を焦点化してit-cleftにできる
Why was it that I was able to quit them?
これは
It was because-- that I was able to quit them.に対応する疑問文
そしてそこから間接疑問にすることも可能
"why it was that I was able to quit them"

>>269
強調構文のところ完璧
0272英文多読者 ◆iiDSZQODZc
垢版 |
2021/02/24(水) 18:11:18.59ID:mr3VHFQs
ビジュアル英文解釈の現代版みたいな本ないのかな
英文解釈の本で一番素晴らしいのはあれでしょ
0273英文多読者 ◆iiDSZQODZc
垢版 |
2021/02/24(水) 20:24:47.39ID:I9owHZUz
have yet to Vbって
完了のhaveに後続するVppをto Vbに変えたものであり、これが可能な理由は、現在完了の完了の否定文はまだやっていないことに加えて、これから行われることまで含意する。そのため未来志向的なto不定詞と相性がよい。
まだhave to Vb「(これから)〜しないといけない」の意味とも矛盾しない。

って説明を考えた。
分かりやすい(筋が通っている)と思うけど妥当だろうか。
生徒に説明するのに「僕の考えた最強の英文法」的なことはしたくないので文献を漁ります。
とりあえずこれ
https://core.ac.uk/reader/147579510
0279英文多読者 ◆iiDSZQODZc
垢版 |
2021/02/25(木) 07:44:56.72ID:hiiYgeO6
関係詞をしっかり理解させるには中学で習う疑問詞を身につけていてほしいが
どれだけの人が特殊疑問文の作り方や疑問詞の品詞を覚えているか
0280英文多読者 ◆iiDSZQODZc
垢版 |
2021/02/25(木) 08:02:56.39ID:hiiYgeO6
英語の適性とは別に英文法の適性ってあるよな
「なんとなくでいいじゃん。分かりにくい」とか「文法用語とか多すぎて意味わからん」って思う人もいれば
「なんとなくじゃわからない」とか「文法用語のおかげで、現象に名前が与えられ、ラベリングすることができ、整理され、分かりやすい」ってなる人もいる 
まぁこれもgradableだけど
0281英文多読者 ◆iiDSZQODZc
垢版 |
2021/02/25(木) 08:51:25.19ID:7vUwc20T
juken 7の動画では築館先生の数学の授業が超わかりやすかった
もうないかもしれないけど
0282英文多読者 ◆iiDSZQODZc
垢版 |
2021/02/25(木) 09:17:25.62ID:7vUwc20T
英語の授業の方法を考えるけど
ちゃんと英文法を授業で完結する形で網羅的に教えるって無理よな
+αで総合英語読んでもらうか、資料を配るとかだよな
0283名無しなのに合格
垢版 |
2021/02/25(木) 09:52:40.36ID:s/jcm+nW
逆で
いやってほど毎日毎日短文や長文を音読させて 日々日々これ 
(同じ文法項目も単語を入れ変えて複数用意したり疑問文にしたり否定文にしたり時制変えたりいろいろする)
電車の中とかシャワーの時とか寝てるときとか、ぼーとしてる時に、つい口に出てくるまでやる


そのあとで、これにはな、こういうルールや名前があったんだぜ
ってやった方が効果的だと思う

総合英語なんか読みにくいし、そもそもあれを何十分もじっと読む子なんか1000人い1人もいない
最後まで読みとおす子も1万人に1人もいない
あれは辞書的に調べものにつかうやつ 
総合英語読め、って言うのは英語好きが普通の人間ことを何もわかってない発想
0284英文多読者 ◆iiDSZQODZc
垢版 |
2021/02/25(木) 14:45:25.03ID:hiiYgeO6
>>283
総合英語以外で自分で英文法学ぶとしたらどうするの?(あ、さっきの総合英語って英文法の核とかも含んでるつもりだった)
0285英文多読者 ◆iiDSZQODZc
垢版 |
2021/02/25(木) 14:52:55.34ID:hiiYgeO6
まあ英文法総覧を読み通せって言ったら厳しい子いるだろうなとは思うけど
授業と並行してエバーグリーンとかを読み通せないって最初から想定して勉強させるってのは無理
そのレベルだったら網羅的に教えようと思わないし、網羅的に教える必要があるって思う時点である程度の読解力は要求していいだろうし
0286英文多読者 ◆iiDSZQODZc
垢版 |
2021/02/25(木) 15:02:57.31ID:hiiYgeO6
正直あれぐらい読めないなら大学入って専門書読めないでしょって感じ
例えばまともに大学で勉強しようと思ったら大学入って大学の授業を受けながら「ミクロ経済学の力」とか、せめて「ファンダメンタルミクロ経済学」レベルを読んだりするわけで

それが「網羅的に英文法を勉強する必要のある、上位校を目指す受験生に」要求してはいけないとしたら相当に厳しい
俺の意識が乖離しすぎて変えないといけないことになる
もちろん国立志望で高3です。もう英語に時間さけませんとかの時期なら分からんでもないが
0287英文多読者 ◆iiDSZQODZc
垢版 |
2021/02/25(木) 15:10:20.26ID:hiiYgeO6
数学だと総合的研究2bとかと比べたら大抵の総合英語なんて超薄いし
0288名無しなのに合格
垢版 |
2021/02/25(木) 16:01:44.70ID:s/jcm+nW
そういう事じゃなくて
英語って所詮言語だから、楽器やスポーツのように考えて
「いいから、これを何百回もやれよ」をまずやって
そのあとで理論を言う方が体が納得するって言うか腑に落ちると思わない?
英語も所詮語学だから論理がきっちりしてるようで例外に次ぐ例外が嫌って程あるじゃない
そこが数学とは違う マクロ経済学とも(こいつらも現実と乖離してる面があるにせよ)また違う

冠詞の使い方とか、加算不可算とか、
総合英語的な奴で、これはこうであれはこうで、でもこういうのはこうで
だがしかしこの場合はこうで、なぜならこうこうこういう理由で
とかやっても、そん時納得しても
実際、英語の会話の時に(試験の英作文でも)それ思い出して使えるかって言ったら微妙じゃない
あれ暗記用じゃないし、かりに暗記してても素早い会話で「ええとこっちだ、いやあっちだ」とかできない

そんなだったら、「文句言わずに冠詞が違うケース500個を暗唱するくらい唱えろ」
を、まずやってから
「ほーらこういうやつはTheだろ、こういうのは無冠詞だろ」とか言うと
「あ、ほんとだ」ってなる
モチほかの文法もね 網羅的学習の前に訓練が先じゃないかと

そう思っただけです
0290英文多読者 ◆iiDSZQODZc
垢版 |
2021/02/25(木) 17:37:14.01ID:5jvSEiUW
帰ってきたらこれ読みます

https://www.timeforkids.com/g56/winter-storm-woes-2/?rl=en-1050

A series of winter storms began a sweep across the United States on February 11, slowing distribution of the COVID-19 vaccine in many parts of the country.

Severe weather hit everywhere from Washington State to the southern U.S. Snow and ice forced airports to close, made roads dangerously slick, and caused millions of people to lose electricity, while record-low temperatures made it dangerous to be outdoors.

The U.S. Centers for Disease Control and Prevention expected major delays in vaccine shipments. Several states rescheduled vaccination appointments. “We want to protect the safety of everyone involved,” Missouri governor Mike Parson said.

In Washington, officials restricted appointments for a first dose of the vaccine and focused on second doses. In Texas, a health center had no power on February 15. Without refrigeration, 8,400 doses were at risk of spoiling, which sent officials scrambling to administer the shots.

The storm was just the latest roadblock to vaccine rollout, with limited supply and other problems slowing the process. But distribution has improved in recent weeks. Each day, 1.7 million Americans get a shot, according to the New York Times.

Stop and Think! What information does the author provide in the lead, or first, paragraph? How does the rest of the article build on that paragraph?
0291名無しなのに合格
垢版 |
2021/02/25(木) 20:44:40.91ID:fpMFNEfy
共通テスト後モチベ下がってたときにこのスレ見て気分転換してたから
お礼と言ってはなんだけど今日の医科歯科の英文和訳置いとく

(1) Perhaps more intriguing than exactly when or where dogs became domesticated is the question of how.

(2) Though the origins of the dog/human partnership remain unknown, it's becoming increasingly clear that each species has changed during our long years together.
0293英文多読者 ◆iiDSZQODZc
垢版 |
2021/02/25(木) 22:10:11.08ID:5jvSEiUW
exactlyといえば数年前の阪大の
how exactlyは悩んだ受験生多いみたいだね
今回はwh- exactlyではなくexactly wh-だから素直に読めばいいけど
あとでそれも読み直す
0294英文多読者 ◆iiDSZQODZc
垢版 |
2021/02/25(木) 22:15:12.77ID:5jvSEiUW
阪大のはこれ
We can read the text of great speeches that predate the phonograph, like Abraham Lincoln's Gettysburg Address, but how exactly the president delivered the lines is lost forever.
0295英文多読者 ◆iiDSZQODZc
垢版 |
2021/02/25(木) 23:32:41.73ID:5jvSEiUW
(1) Perhaps more intriguing than exactly when or where dogs became domesticated is the question of how.
形容詞句more intriguingから始まる時点でCVSかCSVを予測
is the questionでCVSで有ることを確認

恐らく、いつ、どこで犬が飼いならされるようになったのかということよりも興味深いのはどのように犬が飼いならされるようになったのかという問題である。

exactlyは訳さなくてもいいかな
訳すとしたら「正確には」とか「という正確な情報」とかでいいかな
thanのあとは別にthe question ofを補うまでもない
受験英語で構文把握に困ることはあまりないけどきれいに日本語にするの得意じゃないから勘弁な それも日本語力及び英語力の不足ゆえ
CVSをわざわざSVCで訳すよりも擬似分裂文っぽく「〜なのは‥」と訳した

(2) Though the origins of the dog/human partnership remain unknown, it's becoming increasingly clear that each species has changed during our long years together.
犬と人間との協力関係の起源は不明なままであるが、両種族がその長い年月の間一緒に変化してきたことはどんどん明らかになってきている。

こっちは言及できる点が一つもないかな
0296名無しなのに合格
垢版 |
2021/02/25(木) 23:40:44.55ID:fpMFNEfy
これでも過去問と比べたら難しい方な気はする
差付かないだろうと思うんだけど多分細かい訳出で減点されるのかな
もしくは和訳は全学科共通問題だからただのサービス問題ってだけかも
0297英文多読者 ◆iiDSZQODZc
垢版 |
2021/02/25(木) 23:46:35.47ID:5jvSEiUW
阪大は

We can read the text of great speeches that predate the phonograph, like Abraham Lincoln's Gettysburg Address, but how exactly the president delivered the lines is lost forever.

exactlyには疑問詞の強調の用法があることに注意
「どれほど正確に」では意味が通らない
what ever, what the hell, what on earth, what in the world, what the fuckなどの疑問詞の強調は有名
ロングマンより
what/�where/�when etc. exactly
いったい何を[どこに,いつ]…か
Where exactly are we going?
いったいどこへ行こうっていうんだい.
What exactly are you implying?
いったい君は何を言いたいんだい

つまりhowのところは
いったい大統領がどのように述べたのか
ということ
0299英文多読者 ◆iiDSZQODZc
垢版 |
2021/02/26(金) 00:00:55.96ID:L4LVoUW4
さっきの
恐らく、いつ、どこで犬が飼いならされるようになったのかということ
をわざわざしっかりと
〜という疑問とか〜という問題
にしたほうがいいのかな
0300英文多読者 ◆iiDSZQODZc
垢版 |
2021/02/26(金) 06:23:58.85ID:1BSqMxMa
阪大2019を教えるときに使った英文

Behind every film are dozens and dozens of craftspeople. We might not see them, but every single one is vital to the finished film.(Drop Boxのツイートより)
場所句倒置構文

I wonder what it is that is making the noise upstairs.(2004センター試験本試)

X is making the noise upstairs.
it分裂文(t-that強調構文)にするためにit is X thatのようにit isとthatで挟む It is X that is making the noise upstairs. 何が~しているのかという間接疑問の表現を作るためにXをwhatに変えて文頭に移動させる what it is that is making the noise upstairs.
名詞:一体なにが上の階でこの騒音をたてているのか。
wonderの目的語にすると
I wonder what it is that is making the noise upstairs.

名詞:what it is like to stay at home
名詞:外出しないとはどのようなものか
※強調構文ではなく単なるit-to構文

couldn't be further from the reality
ポイント:couldn't 比較級  (仮定法)
(仮定:何があっても)より〜になれないということからmaxであるという意味になる couldn't be further from the reality 現実とは全く異なる 現実とは程遠い (なにがあってもこれ以上現実から遠くなれない→現実とmax離れている)
類) I couldn't agree more. 大賛成
It couldn't be better.   最高
0302英文多読者 ◆iiDSZQODZc
垢版 |
2021/02/26(金) 19:52:32.90ID:d7gOHQzQ
焦点1読んだ
ここで既に予測と修正が触れられてる
超大事
メモ
think highly of Oを高く評価する
three hundred feet ahead: 300フィート前方に
We learn English 3 hours a week.
perの意味で、前置詞と解してもよいa

My house stands off the main road.
off〜から離れて
前置詞のoffといえば次を思い出す
off the coast of X: X沖に
coast(=海岸)から離れているということ
a fishing boat (7 miles) off the coast of England 英海岸(7マイル)沖にいる漁船

When we arrived in New York, it was past nine o'clock.
It was past nine o'clock when we arrived in New York.
のようにSV[SV]は[SV]SVのように順序を交換可能と書いてある([SV]は従属節のこと)
正しいのが違いに注意を払うことも必要
情報構造の違いが存在する
複文や重文で節が複数ある場合、後ろに置かれるものが新情報となる
そして次の文では新情報の内容に関連して記述が続く可能性が高い
0303名無しなのに合格
垢版 |
2021/02/26(金) 20:14:12.47ID:vaNPeDOv
>>297
howをどのように訳されていらっしゃいますが、howをどのようにと訳すならbe動詞の補語が欠落しているときじゃないでしょうか?
文を見ても特に欠落ないし、大統領による言葉の伝え方とかって訳になりませんか? howをthe wayみたいに解釈するパターンじゃーないでしょうか?
(自分は正直、howが程度なのか、th wayなのかの見分け方があまりよくわかってないのもありますが、疑問に思ったので答えて欲しいです)
0304英文多読者 ◆iiDSZQODZc
垢版 |
2021/02/26(金) 20:22:36.66ID:d7gOHQzQ
焦点2 SV[SV[SV]]
learnの使い方は大事
(良い結果として)「身につける」や「to doできるようになる」の意味を持つ
Our son learned to be patient. 息子は我慢強くなった.
焦点3
Light is very important because it helps us see things.
このようにbecauseの前は基本的にカンマを置かないようにしたい
もちろんカンマがあっても良い場合や必要とする場合も存在するが基本的には不要
ちなみにカンマの有無は関係詞と同じく制限と非制限
pictures and movies of distant people and place
は典型的な因数分解型
(picture+movie)×(distant(people+plance))
これに対して意味の上での釣り合いだけじゃなく伊藤和夫は釣り合い、長さのバランスにも指摘しているのが印象的
0305英文多読者 ◆iiDSZQODZc
垢版 |
2021/02/26(金) 20:27:41.85ID:d7gOHQzQ
>>303
もちthe wayと同じ方法とか仕方、様態の意味でとってるよ
あれ
日本語の問題かな
「どのようにSがVしたのか」、「どのようにしてSがVしたのか」、「SがVした方法」ここらへんってほとんど同意じゃない?
正直日本語の能力も低いので日本語を間違えてたらすまん
0306英文多読者 ◆iiDSZQODZc
垢版 |
2021/02/26(金) 20:30:05.55ID:d7gOHQzQ
wisdomより
How did he get [find] the money? 彼はどのようにしてそのお金を手に入れた[見つけた]のか
0307英文多読者 ◆iiDSZQODZc
垢版 |
2021/02/26(金) 21:12:14.07ID:d7gOHQzQ
焦点4
チャーチル可愛くて笑う
take A with one: Aを携帯する
このようにしたら所有だけでなく手元に持っている、携帯していることも表す

焦点5
to one's mind
Aの考えでは(今回は意見のようなものではないので訳は工夫)
to one's 感情やto the 感情 of A
Aが[感情]したことには
To his great delight, she accepted his offer of marriage. 彼が大いに喜んだことに, 彼女は結婚の申し込みを承諾した.

By this time he is very busy
これ少しむずいのでは
byは瞬間動詞と共に用いる
〜までに…するの意味
untilは継続
〜まで…するの意味
そのため今回は「この時まで忙しい」のではなく、「この時までに忙しくなる」の意味
0309名無しなのに合格
垢版 |
2021/02/26(金) 21:28:06.39ID:vaNPeDOv
https://eikaiwa.dmm.com/uknow/questions/1437/
英語の質問?なんですが、with a short sleepにはaがついてて、with little sleepにはaがついてないんですがそういうもんなんでしょうか?
littleとかがつくときはいらない…まあa littleだと少しっていう意味になるし要らないのかなぁ 
0312英文多読者 ◆iiDSZQODZc
垢版 |
2021/02/26(金) 22:13:19.39ID:d7gOHQzQ
many aとかはどちらも限定詞だと思うのだけど、どう理屈で説明されるのだろう
疑問詞ではないwhat a 〜のwhatは限定詞じゃないと説明されるけど
0313英文多読者 ◆iiDSZQODZc
垢版 |
2021/02/26(金) 22:14:29.11ID:d7gOHQzQ
determinerは限定詞とか決定詞とか複数の呼称が広く使われてるから高校生の時いろいろと混乱した
0314英文多読者 ◆iiDSZQODZc
垢版 |
2021/02/26(金) 22:40:27.45ID:ihW6CWEr
焦点6
sledgeそり
try A outがちゃんと機能するかテストする
it snowed a lot
非人称のitが出てきた
最近、東進のクマがit seems thatのitを形式主語として解説しているのが取り上げられ、デタラメだと一部では話題になっている。

ここでit seems thatに関する各文法書の記述を確認しよう
0315英文多読者 ◆iiDSZQODZc
垢版 |
2021/02/26(金) 23:03:03.80ID:ihW6CWEr
・英語の構文150(美誠社)
形式主語itと非人称it、どちらとも考えられる
・英文法解説
非人称のitと解説
It seems certain that the diamond is real.を並べて形式主語の例も紹介
・ゆっくりとしっかり学ぶ英文法
状況を表すitとして解説
・English EX
意味を紹介し、itが何かという話には立ち入らない(見た限り)

これぐらいでいいや

「意味さえ分かればいい。そんな細かいこと…」って思うとしてもEnglish EXみたいに説明すればいいし
そもそもit=thatって分かりやすい分かりにくいどころか何も説明になってない気がする
まぁ昔の話だからきっと今は
0318名無しなのに合格
垢版 |
2021/02/27(土) 08:26:07.23ID:BZl5JiaW
おはようございます 
0319英文多読者 ◆iiDSZQODZc
垢版 |
2021/02/27(土) 20:41:47.16ID:OchSfMBS
焦点6
His young son Bobby
ここでなぜカンマがないのだろうか
関係詞のカンマの有無と同じ
他の息子の存在が示唆されている
まぁどこまでこのルールが適用されるかわからんけど
指摘にあるように the Smiths' houseは重要な文法ポイント
take Bobby there in the car.
wisdomより
私たちは彼の車であちこち移動した.
× We drove around by his car.
○ We drove around in his car.
注「車で」はby carだが, one'sがあると 前置詞 はin.
They put the sledge in and went off.
go offは出発するの意味
降りるはget off

焦点7
teach oneself XでXを独学で学ぶの意味
He learned to write so well
非常にすぐれた文章が書けるようになった…
のように訳しているのは>>304で述べたのと同じ理由
leave〜を辞める退職する
Franklin was only a boy at the time
ここはwas a only boyの場合と意味がかなり異なることに注意
副詞のjustや副詞のbutと同じ
he set out to make his own way in the world
set out for Oに向けて出発する
進む以外のmake one's wayに注意
世界に進むでは意味不明
成功するや認められるの意味
he went on to Philadelphia.
引き続きの意味のためonがついている
arrived in Philadelphia dirty, muddy, and very hungry.
準補語という大事な文法事項
0320英文多読者 ◆iiDSZQODZc
垢版 |
2021/02/28(日) 00:14:45.22ID:IK256J8n
チャプター10
in his old country彼が以前(かつて)住んでいた国
the town of her birth 彼女の生まれた町
チャプター11
crumbかけら
groom花婿
bun甘く丸いパン
lean over Oの上に身を乗り出す
チャプター12
his wife had to tell him about his promises to meet persons, his classes -- even his meals.
his promises, his classes, his mealsの並列
sent A offを見送る
be good friends
これは交換複数と呼ばれるもの
change trainsやshake handsの複数と同じ(だったはず)
Our school is within a ten-minute ride of my house.
ビジュアルでもこれあったのね
>>14と同じ
0321英文多読者 ◆iiDSZQODZc
垢版 |
2021/02/28(日) 00:26:12.01ID:IK256J8n
チャプター13
Allow enough time for discussion.
与える もしくは 充てるや見ておく
の意味のallow
ん?ビジュアル英文解釈2に誤訳がある気がする
ちょい考える
0322英文多読者 ◆iiDSZQODZc
垢版 |
2021/02/28(日) 00:41:45.90ID:IK256J8n
教授法の話
try to explain it to them in a way that everyone will understand.(thatはyour studentsのこと)
伊藤和夫はこのthatを関係代名詞と説明している
understand a wayということであるが
生徒が方法するのか?
まぁそれがありえないとは言い難いが
itを理解できるような方法では?
understandが自動詞で使われているのは気になるが
0323英文多読者 ◆iiDSZQODZc
垢版 |
2021/02/28(日) 01:21:57.99ID:IK256J8n
英文解釈教室ではこのような疑問はあまり発生しない。(というよりすぐ解決する)

http://www.wayaku.jp/study/study03.html
http://sawai-kohsuke.com/kaisetsu02/mokuji.html
http://www.kenkyusha.co.jp/uploads/lingua/prt/19/eibunkai1910.html
http://www.wayaku.jp/kaishaku/index01.html
などなど様々な批評や解説が簡単に見つかるから。

有名であればあるほど他の人のコメントが見つけやすい
0325英文多読者 ◆iiDSZQODZc
垢版 |
2021/02/28(日) 02:59:07.04ID:IK256J8n
チャプター13続き
意外
wisdomはworthを前置詞としている
worth 〜の価値がある
注意点はといえば次の項目
The show is worth seeing.
のように主語がseeingの目的語となり、seeingの直後には空所が存在する。
その点でneed ingと似ていますね
worth ingのingも名詞的動名詞かな
・Still, the pledge was worth a careful reading.
(the New York Times)
・The Speed Camera Radar application is worth regular updating in order to keep you up to date with the latest radar information.
(よくわからんサイトだから信用は?かな)
とかがあるから名詞的動名詞っぽいけど
conversion変換
repetition反復
review復習
0327英文多読者 ◆iiDSZQODZc
垢版 |
2021/02/28(日) 05:06:26.45ID:GxLWIWkG
チャプター15
be true to Oに忠実である
Carol is true to her word .
be true to Oに一致している
The movie is true to the title.
injureはアクセントが第一音節
disciplineしつけ、訓練、規律、懲罰、学科
grief悲しみ
yel /yel/ 叫ぶ これなら発音記号と違って文字化けしないかな
例えば
furiously/fyoor-ee-uhs-lee/怒り狂って
「I can remember always losing to my elder brothers at our little football gamesのalwaysはlosingに掛かる」 たしかに
stillそれでも
with all one's strength全力で
sometimes時々、たまに、時々の
roundは〈英〉でよく使われるんだ
そのことは知らなかった
with all 〜があるので(理由)あるものの(譲歩)
He'll find a job easily with all the work experience he has.
Even with all the problems, I love my job.
チャプター16
catch Oを理解する
hintほのめかし
set Oを作る
set A for BにAを示す
set a good example for Oへ良い例を示す
hypocritical偽善的
0328英文多読者 ◆iiDSZQODZc
垢版 |
2021/02/28(日) 05:33:11.79ID:GxLWIWkG
チャプター17
overpopulated人口過剰の
ここのlearned to doも「できるようになった」の意
先のチャプター16で仮定法と指摘されていた箇所に違和感を抱いたが(if S V現在形, S 助動詞の過去形. で仮定法と解説)、このチャプター17でやっと解説。仮定法の対応関係が守られていない例であり、現在形は過去形であるのが正しいと解説されている。
調べてみたところ仮定法と直説法の混用はしばしばあるよう
後でまた調べます
0329英文多読者 ◆iiDSZQODZc
垢版 |
2021/02/28(日) 06:38:24.05ID:GxLWIWkG
チャプター18
fit A for Bに適するようにする
as to Oに関して
fully少なくとも、たっぷり、優に
Fully 85% of the students were women. 学生の少なくとも85%は女性だった
give A credit for Bの点でAを認める
credit名誉称賛功績
チャプター19
socialize with Oと交際する
able有能な(限定用法)
be in A's interest(s): Aのためになる
interests利益(通例複数)
in the name of Oの大義名分のもとに


more often than not≒ as often as not≒ usually
たいていの場合は
それぞれ
more often than not often
as often as not often
の略
oftenであることの強調表現であるからusuallyと同意
似た表現に
as likely as not≒ more likely than not≒ more than likely≒ probably
最初の2つもnot likely のlikely が省略されており、どの表現もlikelyの強調のためprobablyと同意
参考:
https://gamp.ameblo.jp/bakumatsutaiyoudenn/entry-11783246637.html
0330名無しなのに合格
垢版 |
2021/02/28(日) 07:06:54.43ID:YtIHyq5+
保守
0332英文多読者 ◆iiDSZQODZc
垢版 |
2021/02/28(日) 07:08:09.27ID:GxLWIWkG
てか最近参考書勉強日記になってるな
英文も載せれないし微妙
ちゃんとそこらの英文読むってのもするか
0334英文多読者 ◆iiDSZQODZc
垢版 |
2021/02/28(日) 07:10:56.70ID:GxLWIWkG
前と同じサイトから
http://read.gov/aesop/003.html

Belling the Cat

The Mice once called a meeting to decide on a plan to free themselves of their enemy, the Cat. At least they wished to find some way of knowing when she was coming, so they might have time to run away. Indeed, something had to be done, for they lived in such constant fear of her claws that they hardly dared stir from their dens by night or day.

Many plans were discussed, but none of them was thought good enough. At last a very young Mouse got up and said:

"I have a plan that seems very simple, but I know it will be successful.

All we have to do is to hang a bell about the Cat's neck. When we hear the bell ringing we will know immediately that our enemy is coming."

All the Mice were much surprised that they had not thought of such a plan before. But in the midst of the rejoicing over their good fortune, an old Mouse arose and said:

"I will say that the plan of the young Mouse is very good. But let me ask one question: Who will bell the Cat?"

It is one thing to say that something should be done, but quite a different matter to do it.
0335英文多読者 ◆iiDSZQODZc
垢版 |
2021/02/28(日) 07:55:34.06ID:GxLWIWkG
今回からdeep l翻訳利用して日本語も載せようかな
おかしすぎるところだけ手を加えればいいし
一部しか手を入れないし、変なところがあるけど勘弁な(明らかな誤訳は指摘してくれたら助かる)

Belling the Cat
bell Oに鈴をつける

The Mice once called a meeting to decide on a plan to free themselves of their enemy, the Cat. At least they wished to find some way of knowing when she was coming, so they might have time to run away. Indeed, something had to be done, for they lived in such constant fear of her claws that they hardly dared stir from their dens by night or day.
ーーーーーーーーーーーーーーーーー
Deep L翻訳(一部修正)
ネズミはかつて敵である猫から自由になる計画を決めるために会議を開いた。少なくとも、彼らは逃げる時間を確保できるように、彼女がいつ来るかを知る方法を見つけたいと思っていた。実際、何かをしなければならなかった。彼らは彼女の爪を常に恐れて生活していたので、夜も昼も巣穴から身動きをとる勇気がなかった。
ーーーーーーーーーーーーーーーーー
mouthの複数形がmiceなのは中学で習った人も多いかな
覚えにくいかもしれないけどサイコロの例はみんな知っている
diceはもともと単数形dieの複数形
でも現在はdiceが単数形としても複数形としても用いられる
「運命によって与えられたもの」が語源らしいです
die自体もThe die is cast.「賽は投げられた。」を聞いたことがある人は多そう
call Oを招集する
decide on A:複数の選択肢からAに決める
この場合は既に複数の計画が存在しており、そこから選ぶということ
free A of BからAを解放する
some wayを「いくつかの方法」と解釈しないようにしたい。「何らかの方法」
so they might...
のsoは>>99と同じくso thatのthatが省略されたものであるため「なので」と解釈しないようにしたい。目印はso以下に存在する助動詞
so thatは目的ならカンマを用いず、結果であればカンマを用いるのが普通であるが、今回は目的と解するといいかな
調べてみるとこんなサイトが見つかった
@抜いて
https://jay0123.hate@nablog.com/entry/20100210/1265793567
また、この論文に少し参考になることが載ってそう
https://www.google.com/url?sa=t&;source=web&rct=j&url=https://www.jstage.jst.go.jp/article/japeronso1991/1995/3/1995_3_21/_pdf&;ved=2ahUKEwjutpXtkYvvAhULEqYKHXK5Cy84ChAWMAV6BAgGEAI&usg=AOvVaw1NblWNew8fTQT-WMpMSdIa
こちらはまだ読んでいないので暇なとき読みます
wisdomから例を
They are climbing higher so that they may get a better view. 彼らはもっと眺めがよくなるようにさらに高く登っているところだ.
等位接続詞のfor
thatはsuchに呼応
constant fear絶え間ない恐怖
dare (to) Vbする勇気がある、あえて…する
stirわずかに身動きをする
den住処の穴≒lair
by night夜間に
今回はby night or dayなので昼夜問わずの意味
0336名無しなのに合格
垢版 |
2021/02/28(日) 08:06:07.18ID:N3o/4AuX
あ、この話前にリスニングで聴いたことある
簡単な英語に書き換えてあったから原文初めて見た
0337英文多読者 ◆iiDSZQODZc
垢版 |
2021/02/28(日) 08:25:11.85ID:GxLWIWkG
Many plans were discussed, but none of them was thought good enough. At last a very young Mouse got up and said:
多くの計画が議論されましたが、どれも十分に良いとは思われませんでした。最後にとても若いネズミが立ち上がって言いました。

"I have a plan that seems very simple, but I know it will be successful.
非常にシンプルに思えるが、必ず成功すると確信している計画がある。

All we have to do is to hang a bell about the Cat's neck. When we hear the bell ringing we will know immediately that our enemy is coming."
私たちがすべきことは、猫の首に鈴をかけることです。鐘が鳴るのを聞けば敵が接近しているのをすぐに分かる。"
ーーーーーーーーーーーーーーー
aboutは〜の周りの意味
例wear a scarf about one's neck 首にスカーフを巻く.

All the Mice were much surprised that they had not thought of such a plan before. But in the midst of the rejoicing over their good fortune, an old Mouse arose and said:
ネズミたちは皆、今までそのような計画を考えていなかったので、とても驚いていました。しかし、自分たちの幸運を喜んでいる最中に、一匹の年老いたネズミが起きて言いました。
ーーーーーーーーーーーーーーー
that節はsurprisedに呼応して理由を表す
midst最中 middleのイキった版
rejoice over Oを喜ぶ
もちろんこれは通常の動名詞ではなく名詞的動名詞(だってtheがついている)
good fortune幸運
※これは「天才的な発想によってもたらされるであろう今後の幸せな生活(運命)」のことでよいだろうか?それとも単にラッキー的な?
arise立ち上がる

"I will say that the plan of the young Mouse is very good. But let me ask one question: Who will bell the Cat?"
私は若いマウスの計画は非常に良いと言うでしょう。しかし、一つ質問させてください。"誰が猫の鈴を鳴らすのか?"
ーーーーーーーーーーーーーーー
このwill sayはどうやって処理したところだろうか「〜と言えよう」的なことかな

It is one thing to say that something should be done, but quite a different matter to do it.
何かをするべきだと言うのは一つのことですが、それをするのは全く別の問題です。
ーーーーーーーーーーーーーーー
one thingは
Living together is one thing, getting married is another. 一緒に住むことと結婚することは全然違う
と同じような意味だろう
0338英文多読者 ◆iiDSZQODZc
垢版 |
2021/02/28(日) 08:27:27.23ID:GxLWIWkG
>>336
俺も読んだことあると思ったら英文解釈クラシックにあった
でもこれはこれでかなり、相当に英文が違う
0340名無しなのに合格
垢版 |
2021/02/28(日) 08:29:04.42ID:y3+a8ogU
e
0342名無しなのに合格
垢版 |
2021/02/28(日) 08:31:17.28ID:N3o/4AuX
>>339
なるほど
0344英文多読者 ◆iiDSZQODZc
垢版 |
2021/02/28(日) 08:42:25.77ID:GxLWIWkG
翻訳にあたって生々しい表現を消したり
かなり変えられてるみたいね
もともと口頭伝承だからそもそもあれだけとま
0346英文多読者 ◆iiDSZQODZc
垢版 |
2021/02/28(日) 08:58:39.00ID:GxLWIWkG
英文解釈クラシックの
ネズミはEliot/Jacobs Version
年老いたライオンはAesop For Children
0350英文多読者 ◆iiDSZQODZc
垢版 |
2021/02/28(日) 09:33:50.70ID:GxLWIWkG
現代だとありえない文法とか出てきそう
日本語に比べると変化が少ないとはいえ
0351英文多読者 ◆iiDSZQODZc
垢版 |
2021/02/28(日) 09:42:08.13ID:GxLWIWkG
まずは導入部分
写真から読み込むところもあるから誤認識されていてスペルとかおかしいところあったらごめんね


EDITOR'S PREFACE.

Twas the Golden Age, when every brute
Had voice articulate, in speech was skilled,
And tongues of rock and pine-leaf then were free;
To ship and sailor then would speak the sea;
Sparrows with farmers would shrewd talk maintain;
Earth gave all fruits, nor asked for toil again.
Mortals and gods were wont to mix as friends.
To which conclusion all the teaching tends
Of sage old Aesop.”
Babrius. Proem I.
0352英文多読者 ◆iiDSZQODZc
垢版 |
2021/02/28(日) 10:00:17.32ID:GxLWIWkG
EDITOR'S PREFACE
preface前書き

“ 'Twas the Golden Age, when every brute
Had voice articulate, in speech was skilled,
And tongues of rock and pine-leaf then were free;
'Twasなんて初めて見たけど推測は容易
調べてみるとwisdomでも'tは載っていた
it wasのこと
brute野蛮な者
articulate明瞭な
なんで後置修飾?
in speechはwas skilledに対する主語になりえない
every bruteでandがカンマで代用されたかな
tongues of rock and pine-leafは比喩でしょう 何を表してるかは知らない

To ship and sailor then would speak the sea;
Sparrows with farmers would shrewd talk maintain;
これはMVSかな
船と船乗りにとって海はspeakしていたと
wouldは助動詞ではない
shrewdは形容詞であるから
この場合のwouldは本動詞で
would (that)SVを望む の意味
shrewd talk的確(?)な会話が

maintainの自動詞用法は「冷静でいる」しか見つからない
これでいいのか?



もういいや
本編いきます
0353英文多読者 ◆iiDSZQODZc
垢版 |
2021/02/28(日) 10:08:53.28ID:GxLWIWkG
The Two Frogs
One hot summer, the lake in which two Frogs lived was completely dried up, and they were obliged to set off in search of water elsewhere. Coming to a deep and deliciously cool well, one of the Frogs proposed that they should jump in at once. “Wait a bit,” cried the other: “if that should dry up, how could we get out again?
be obliged to Vbせざるをえない
ちなみに
be obligedで感謝しているの意味にもなるようです
set offは先にも出た「出発する」の意味
in search of Oを探して
elsewhereどこか他の場所で
deliciouslyとてもおいしく、愉快に
well 井戸
at once同時に
if that should dry upは仮定法現在のためshouldになっているが直説法として捉えて問題ないだろう
dry up干上がる
0354英文多読者 ◆iiDSZQODZc
垢版 |
2021/02/28(日) 10:11:31.57ID:GxLWIWkG
あ、仮定法現在って呼ばないな
普通に仮定法でいいか
すまん
それに対応するcouldね
0356英文多読者 ◆iiDSZQODZc
垢版 |
2021/02/28(日) 10:59:32.34ID:GxLWIWkG
JUPITER AND THE CAMEL.
jupiter木星あるいは神ユピテル
camelラクダ
THE Camel once upon a time complained to Jupiter that he was not as well served as he ought to be in the means of defence and offence.
この場合のserveは与えるに近いかな
つまり攻守の手段という点において十分に与えられていないということ
" The bull," said he, "has horns, the boar, tusks, and the lion and tiger, formidable claws and fangs that make them feared and respected on all sides.
bull雄牛
boreイノシシ
tusk牙(象やイノシシ系の)
formidable恐るべき
最初はそれぞれの武器を列挙している
thatはそれらの武器全てに掛かる関係代名詞
fear Oを畏れるOに畏怖の念を抱く
on all sides至るところで
I, on the other hand, have to put up with the abuse of all who choose to insult me." Jupiter angrily told him that if he would take the trouble to think, he would see that he was endowed with qualities shared by no other beast; but that, as a punishment for his unreasonable importunity, henceforward his ears should be shortened.
abuse罵詈雑言
ofはabuseの意味上の主語
angrily怒った様子で
if he wouldのwouldは意志のwillが時制の一致を受けたもの
take the trouble to VわざわざVする
think 熟考する
see that SVと分かる
endow A with Bを与える
henceforth≒henceforward=今後は
importunity執拗な要求
take the trouble to VbわざわざVする
ここのbut thatについて悩んでたけど
普通にtalk A but Bの形としてもいいのかな
与えられているけどバツとして耳を短くしちゃうぞっていう
but thatの塊でもいろんな意味はあるみたいだけど
ここのshouldはあまり悩みすぎないようにしたいshouldの意味を考えても無駄
wouldと同じく時制の一致をうけ、shallの過去形として使われているのも
shall Vすることとなろう の意味
0359英文多読者 ◆iiDSZQODZc
垢版 |
2021/02/28(日) 11:17:13.71ID:GxLWIWkG
THE LION HUNTING WITH OTHER BEASTS.

A LION, a Heifer, a Goat, and a Sheep once agreed to share whatever each might catch in hunting. A fine fat stag fell into a snare set by the Goat, who thereupon called the rest together. The Lion divided the stag into four parts.
heifer若い雌牛
agree to Vするのに同意する
fine立派な
stag雄鹿
snareわな(足を縄で締めるタイプ)
thereuponそのすぐあとに
Taking the best piece for himself, he said, "This is mine of course, as I am the Lion;" taking another portion, he added, “ This too is mine by right-the right, if you must know, of the strongest." Further, putting aside the third piece, " That's for the most valiant," said he; "and as for the remaining part, touch it if you dare."
by right権利によって
the right of the strongestの間に挿入が存在している
valiant勇敢な
as for Oに関して
dare Vb大胆にもVする
結局ライオンは全部食べるのだろう

if you must know のifはどういう機能だろう
後で調べてみる
0360英文多読者 ◆iiDSZQODZc
垢版 |
2021/02/28(日) 11:20:39.02ID:GxLWIWkG
if you must know でフレーズとして辞書に載っているね
そんなに知りたければ教えるが
used when you answer a question that you think someone should not have asked, because it is slightly impolite
0361英文多読者 ◆iiDSZQODZc
垢版 |
2021/02/28(日) 11:49:03.35ID:GxLWIWkG
次はAesop for Childrenで同じやつを読んでみよう
http://read.gov/aesop/141.html
The Lion's Share

A long time ago, the Lion, the Fox, the Jackal, and the Wolf agreed to go hunting together, sharing with each other whatever they found.

One day the Wolf ran down a Stag and immediately called his comrades to divide the spoil.

Without being asked, the Lion placed himself at the head of the feast to do the carving, and, with a great show of fairness, began to count the guests.

"One," he said, counting on his claws, "that is myself the Lion. Two, that's the Wolf, three, is the Jackal, and the Fox makes four."

He then very carefully divided the Stag into four equal parts.

"I am King Lion," he said, when he had finished, "so of course I get the first part. This next part falls to me because I am the strongest; and this is mine because I am the bravest."

He now began to glare at the others very savagely. "If any of you have any claim to the part that is left," he growled, stretching his claws meaningly, "now is the time to speak up."

Might makes right.
0363英文多読者 ◆iiDSZQODZc
垢版 |
2021/02/28(日) 12:05:54.42ID:GxLWIWkG
A long time ago, the Lion, the Fox, the Jackal, and the Wolf agreed to go hunting together, sharing with each other whatever they found.
登場人物(動物)が変わってるのね
One day the Wolf ran down a Stag and immediately called his comrades to divide the spoil.
comrade仲間
spoil成果戦利品
Without being asked, the Lion placed himself at the head of the feast to do the carving, and, with a great show of fairness, began to count the guests.
carve Oを切り分ける
fairness公平公正
"One," he said, counting on his claws, "that is myself the Lion. Two, that's the Wolf, three, is the Jackal, and the Fox makes four."
make Cになる、相当する
He then very carefully divided the Stag into four equal parts.

"I am King Lion," he said, when he had finished, "so of course I get the first part. This next part falls to me because I am the strongest; and this is mine because I am the bravest."
fall to Oに帰属する
He now began to glare at the others very savagely. "If any of you have any claim to the part that is left," he growled, stretching his claws meaningly, "now is the time to speak up."
glare at Oをじっと睨む
claim権利(の主張)
growlうなる
meaningly意味ありげに

Might makes right.
最後に1文でまとめられてるのいいね
0364英文多読者 ◆iiDSZQODZc
垢版 |
2021/02/28(日) 12:45:50.20ID:GxLWIWkG
では次はJBR 1874 Editor's
Collectionのp.3

THE STAG LOOKING INTO THE POOL.

A STAG drinking at a clear pool, admired the handsome look of his spreading antlers, but was much displeased at the slim and ungainly appearance of his legs. "What a glorious pair of branching horns !" said he. “ How grace- fully they hang over my forehead! What an agreeable air they give my face! But as for my spindle-shanks of legs, I am heartily ashamed of them." The words were scarcely out of his mouth, when he saw some huntsmen and a pack of hounds making towards him. His despised legs soon placed him at a distance from his followers, but, on entering the forest, his horns got entangled at every turn, so that the dogs soon reached him and made an end of him. " Mistaken fool that I was !" he exclaimed ; " had it not been for these wretched horns my legs would have saved my life."
0365英文多読者 ◆iiDSZQODZc
垢版 |
2021/02/28(日) 13:12:50.53ID:GxLWIWkG
THE STAG LOOKING INTO THE POOL.
stagは先程同様雄鹿
A STAG drinking at a clear pool, admired the handsome look of his spreading antlers, but was much displeased at the slim and ungainly appearance of his legs.
antler枝角
his spreading antlers look handsomeということ
ungainly不格好な
→これは簡単にしたやつを塾バイトの教材として使ったことある。今気づいた
"What a glorious pair of branching horns !" said he. “ How gracefully they hang over my forehead! What an agreeable air they give my face! But as for my spindle-shanks of legs, I am heartily ashamed of them."
glorious華麗な
branch枝分かれする
gracefully美しく優美に
hang over Oから突き出ている
agreeable心地よい
air雰囲気
shanks脚、脛
spindle-shanks細く長い脚
heartily心から
The words were scarcely out of his mouth, when he saw some huntsmen and a pack of hounds making towards him. His despised legs soon placed him at a distance from his followers, but, on entering the forest, his horns got entangled at every turn, so that the dogs soon reached him and made an end of him. " Mistaken fool that I was !" he exclaimed ; " had it not been for these wretched horns my legs would have saved my life."
(scarcely・hardly)…(when・before・than)---
…するとすぐ---
…するかしないかのうちに---
thanはno sooner...than---との混交
huntsman猟師
pack群れ
make towards Oへと進む
on ingするとすぐに、同時に
be entangledからまる
turn曲線
今回のso thatは結果
make an end of Oを殺す
mistaken 〜:誤った〜
fool馬鹿もの
exclaim叫ぶ
最後は倒置によるif省略が起こった普通の仮定法
wretched 〜:むかつく〜
0366英文多読者 ◆iiDSZQODZc
垢版 |
2021/02/28(日) 13:16:25.16ID:GxLWIWkG
次はこれのAesop for Children
A Stag, drinking from a crystal spring, saw himself mirrored in the clear water. He greatly admired the graceful arch of his antlers, but he was very much ashamed of his spindling legs.
“How can it be,” he sighed, “that I should be cursed with such legs when I have so magnificent a crown.”
At that moment he scented a panther and in an instant was bounding away through the forest. But as he ran his wide-spreading antlers caught in the branches of the trees, and soon the Panther overtook him. Then the Stag perceived that the legs of which he was so ashamed would have saved him had it not been for the useless ornaments on his head.

We often make much of the ornamental and despise the useful.

また今度読みます
0368名無しなのに合格
垢版 |
2021/02/28(日) 21:07:23.70ID:HY7KQ1qz
多読がんばれー 今何読んでるの?
0369英文多読者 ◆iiDSZQODZc
垢版 |
2021/02/28(日) 21:13:33.57ID:+T7tPqtC
>>369
さっきまではビジュアル英文解釈の誤訳に悩んでたけど
基本ずっとイソップ物語に苦戦してると思う

>>349のJBR Collection(1874年)のバージョンを読んで、それが>>348のAesop for Children(1919年)のバージョンにも含まれてる話ならそっちも読んでいくつもり
0370英文多読者 ◆iiDSZQODZc
垢版 |
2021/03/01(月) 10:46:46.74ID:zqQdFHVZ
ではAesop for Childrenの
The Stag & His Reflection

http://www.read.gov/aesop/017.html

A Stag, drinking from a crystal spring, saw himself mirrored in the clear water. He greatly admired the graceful arch of his antlers, but he was very much ashamed of his spindling legs.
crystal透き通った
mirror Oを映す
greatly非常に
graceful優美な
archアーチ
antler枝角
spindlingひょろながい
“How can it be,” he sighed, “that I should be cursed with such legs when I have so magnificent a crown.”
how can SV:どうしてSVなんだ,よくもSがVする
sighため息をつく
shouldは具体的な意味を持たず感情を表す表現(推量だとおかしいよね)
Althoughの意味のwhen
wisdomより類例
She said she was underage when I knew she was over 20. 彼女は20歳を過ぎていることを私は知っているが, 自分は未成年だと言った
At that moment he scented a panther and in an instant was bounding away through the forest. But as he ran his wide-spreading antlers caught in the branches of the trees, and soon the Panther overtook him. Then the Stag perceived that the legs of which he was so ashamed would have saved him had it not been for the useless ornaments on his head.
scenet Oを察知する
in an instant一瞬で
bound跳ね返る、飛ぶように走る
ornaments装飾品
of which はashamedに掛かるふつうの前置詞+関係代名詞
had it not…は倒置によるif省略で「〜がなかったら」
wouldも仮定法のwould
We often make much of the ornamental and despise the useful.
make much of Oを重んじる
ornamental観賞用の
despise〜をひどく嫌う
the 形容詞で「〜なもの」を表す
α英文法より類例
It is likely that the urgent is dominating our lives.緊急のものが私たちの生活を支配しがちである
0371英文多読者 ◆iiDSZQODZc
垢版 |
2021/03/01(月) 10:50:49.32ID:zqQdFHVZ
次は
JBR Collection
http://read.gov/books/pageturner/aesops_fables/#page/16/mode/2up

THE COCK AND THE JEWEL.

A BRISK young Cock scratching for something with which to entertain his favourite hens, happened to turn up a jewel. Feeling quite sure that it was something precious, but not knowing well what to do with it, he addressed it with an air of affected wisdom as follows :-" You are a very fine thing, no doubt, but you are not at all to my taste. For my part, I would rather have one grain of dear delicious barley than all the jewels in the world."
0372英文多読者 ◆iiDSZQODZc
垢版 |
2021/03/01(月) 11:10:20.63ID:zqQdFHVZ
THE COCK AND THE JEWEL.

A and Bというとき、AとBは対等とは限らない
例えばニ詞一意のand
wikiの二詞一意より
「「sound and fury(響きと怒り)」は、本来は「furious sound(怒りの響き)」と表現すべきところだが、二詞一意は、形容詞の中に表された元々の従位的概念を名詞に変換するのが一般的で、それにより、さらに印象的なイメージを与えることができる。
『主の祈り』の中にある「The kingdom and the power and the glory(国と力と栄え/栄光)」は、この原則を拡大させたもので、「glorious, powerful kingdom(栄えある力強き国)」を接続詞で繋がれた3つの名詞に変換させている。」
他にも論文?を
https://www.google.com/url?sa=t&;source=web&rct=j&url=https://core.ac.uk/download/pdf/144568464.pdf&;ved=2ahUKEwjCtfONgI7vAhVDeXAKHZp5A54QFjAKegQIDRAC&usg=AOvVaw0dXGOaX0HY3iinYkq-sKoS

この論文のほうが面白いので休憩
0373英文多読者 ◆iiDSZQODZc
垢版 |
2021/03/01(月) 11:30:58.42ID:zqQdFHVZ
andで名詞または形容詞を並べると意味の上では一方が他方を就職する場合がある

二詞一意:意味上主従の関係がある
「名詞+and+名詞」では,意味上「形容詞+名詞」となる。
death and honor→ honorable death
cups and gold→ golden cups
「形容詞+and+形客詞」では,「副詞十形容詞」となる。
nice and warm→ nicely warm

go and seeが意味的にはgo to seeの意味
これは文法でも習うから知ってた

Hendiadys
同義語反復構文(二重表現法):andやorで同じ意味の言葉を並べ、難解な語彙の理解を助ける(, orのことかな)

zeugmaくびき語法
@文中でひとつの動詞または形容詞が二つ以上の名詞と関連して用いられること。 He bought a book, she a notebook and her brother a pencil. boughr はabook, anotgbook,apencil に文法的にも意味的にも正しく関連している。 A中には、一つの動詞または形容詞が、意味的に二つ以上の名詞と関連していながら,文
法的には一つの名詞にしか関連しない場合がある。 The apple was eaten and bananas neglected. は意味的にapple と bananas に関連しているが、文法的にはwas はapple にしか関 連しておらず, bananas のあとにwere が入るのが正しい。
B文法的にニつ以上の名詞と関連していながら, 意味的には一つの名詞としか関連してい
ない場合もある。 He killed the man and the luggage. 文法的にはkilled がthe manと the luggage の両方に関連しているが,意味的にはkilledはthe maeにしか関連し得ないので、andのあとにdestroyedなどが入るのが正しい。
syllepsis兼用法
ーつの文の中で、一つの語を二つまたはそれ以上の異なる意味(文字どおりの意味と比 験的な意味)で用いること。表現を引き締めるのに役立つ。 He took a towel and bath.
fook が「タオルを取る」、「風呂に入る」のニつの意味で使用されている。

syllepsisおもしろいね
0374英文多読者 ◆iiDSZQODZc
垢版 |
2021/03/01(月) 11:33:25.58ID:zqQdFHVZ
http://user.keio.ac.jp/~rhotta/hellog/cat_hendiadys.html

これも面白い
flat adverbなんて表現知らなかった
longとかなんだね

「主にくだけた口語で用いられ,本来の副詞より端的で力強い表現」(小西,p. 507);「単純形副詞がよく用いられるのは,文が比較的短く,幾分感情的色彩がある場合で,口語体ではより力強い表現として好まれる傾向がある: It hurts bad. / Take it easy. / Go slow.」(荒木, p. 522)
 (6) flat adverb と -ly adverb では,分裂文 (cleft sentence) で焦点化された場合の容認可能性が異なる.flat adverb は,動詞句と分断されることにより形態だけでは副詞とわかりにくくなるために,容認度が低くなるのではないか(荒木,p. 522).

(7) A and B と形容詞を等位接続すると,A-ly B ほどの意となる二詞一意 (hendiadys) の例も,形容詞の副詞化という現象の一種としてとらえることができそうだ(大塚,pp. 531, 589).ex. nice and cool (= "nicely cool"), good and tired (="well tired"), rare and hungry (="quite hungry").
0375英文多読者 ◆iiDSZQODZc
垢版 |
2021/03/01(月) 12:09:28.65ID:zqQdFHVZ
THE COCK AND THE JEWEL.

A BRISK young Cock scratching for something with which to entertain his favourite hens, happened to turn up a jewel.
brisbきびきびとした
大文字なのはなにか意味があるのだろうがとりあえず後で考える
cockは男根の意味もあるが今回は雄鶏の意味
scratchひっかく
something with which to entertain his favorite hens
これは少し面白い構文
hen雌鶏(複数いるの?)
happend to Vb:いきなりVする
turn A upを発見する

Feeling quite sure that it was something precious, but not knowing well what to do with it, he addressed it with an air of affected wisdom as follows :-" You are a very fine thing, no doubt, but you are not at all to my taste. For my part, I would rather have one grain of dear delicious barley than all the jewels in the world."
まず分詞構文の並列から
address Oに話しかける
affected気取った
気取った賢者の雰囲気でってと?
fine上等な
no doubt確かに(副詞句)
be to one's taste:〜の好みに合っている
for one's part:〜としては
would rather V1 than V2:V2するよりV1したい
would sooner V1 than V2と同意
ここでのratherはsoonerの意味
そしてsoonはここでは時間的順序ではなく願望の順序、つまり優先順位を表している
thanのあとにはhaveが省略
barley大麦
0376名無しなのに合格
垢版 |
2021/03/01(月) 12:28:56.98ID:PKfxcl7Y
>>373

兼用法とか二詞一意を面白いと感じれるなら、

The Elements of Eloquence

とかおすすめだよー
0377英文多読者 ◆iiDSZQODZc
垢版 |
2021/03/01(月) 12:39:55.31ID:zqQdFHVZ
scratching for something with which to entertain his favorite hens
scratch for somethingに関しては「〜を求めて(探して)(そのへんを)引っ掻く」の意味だろう
with which to に関しては次の通り
まず関係代名詞から。
関係代名詞は不完全文を導くとよく言うがその表現は不思議
This is the office which he works in.(α英文法より)は
he works in a officeが 
he works in whichとなりそれが前に出て
which he works in となったもの 
(だからこそ関係代名詞という名前でありwhichは接続詞兼代名詞)そのため節内では目的語の欠落などはない
もちろんわざわざwhcihなどを無視してそれよりあとだけを見れば不完全だけど
What do you want?のdo you wantだけを見て不完全とは言わないだろう
欠落しているのではなく単に移動しており、痕跡としてφの位置がわかれば良い
節としては完全に要素を備えてる
むしろ不定詞の形容詞的用法のほうが不完全
VO関係での修飾を見る
something to entertain his favorite hens with
ここではwithの目的語が欠けている
そして関係詞節と違い、withの目的語が句の中で存在しているわけではない
目的語はその句の外側に存在しており、目的語であるsomethingはそれだけで別の、scratchの目的語という一つの機能を果たしている
関係詞にもどる
This is the office which he works in.ではwhichを前に移動させていたが前置詞句の塊、つまりin which の状態で前に出すこともできる。
This is the office in which he works.
これは規範文法に則り、前置詞+空所という形で文を終わらせないためというのがメジャーな理由
それと同じようなことを
something to entertain his favorite hens with
でも行おうとしてもsomethingは既にscratchの目的語として機能しているためwith somethingにはできないし、with toも意味不明。そのため関係詞の助けを借りる必要が生じ、whichが突然現れ
something with which to entertain his favorite hensとなる

(こういうふうに自分の中で理屈付けしてるけど正しいかは知らん)
実例解説英文法に不定詞関係節として載ってたはず
まぁ前置詞+関係詞でその後ろの残りのSVがto不定詞になるって考えればおわり


お気に入りの雌鶏を楽しませるためのものを求めて(探して)(そのへんを)引っ掻いてたの意味だろう
0378英文多読者 ◆iiDSZQODZc
垢版 |
2021/03/01(月) 12:42:42.75ID:zqQdFHVZ
>>376
Twitterで紹介されてるの見て調べてここらへんここらへんの項目とかその書籍名も知った
いつか余裕ができたら買います
ありがとう
今調べたら意外と安い
0380かばん屋
垢版 |
2021/03/01(月) 12:51:50.22ID:PKfxcl7Y
>>379
すごい安いから是非
0382英文多読者 ◆iiDSZQODZc
垢版 |
2021/03/01(月) 12:54:58.73ID:zqQdFHVZ
あとで
JBR Collectionの続き読みます
http://read.gov/books/pageturner/aesops_fables/#page/16/mode/2up
 
次は
THE WOLF AND THE LAMB.

A HUNGRY Wolf one day saw a Lamb drinking at a stream, and wished to frame some plausible excuse for making him his prey. "What do you mean by muddling the water I am going to drink ?" fiercely said he to the Lamb. " Pray forgive me," meekly answered the Lamb; "I should be sorry in any way to displease you, but as the stream runs from you towards me, you will see that such cannot be the case." "That's all very well," said the Wolf; "but you know you spoke ill of me behind my back a year ago." " May, believe me," replied the Lamb, "I was not then born." " It must have been your brother then," growled the Wolf. " It cannot have been, for I never had any," answered the Lamb. "I know it was one of your lot," rejoined the Wolf, "so make no more such idle excuses." He then seized the poor Lamb, carried him off to the woods, and ate him.
0383かばん屋
垢版 |
2021/03/01(月) 12:55:36.70ID:PKfxcl7Y
>>381
なんとなくそんな気がしたから名前を変えました😇
0384英文多読者 ◆iiDSZQODZc
垢版 |
2021/03/01(月) 12:59:48.14ID:zqQdFHVZ
>>383
昔からMr.BIG見てて
その中ですげえなって思ってたアカウント
最近見ないなと思ってたけど転生してたからフォローしてます😇
0385英文多読者 ◆iiDSZQODZc
垢版 |
2021/03/01(月) 13:09:57.24ID:zqQdFHVZ
あ、for Childrenでさっきのあったら
こっち優先で読みます

http://read.gov/aesop/058.html

The Cock & the Jewel
A Cock was busily scratching and scraping about to find something to eat for himself and his family, when he happened to turn up a precious jewel that had been lost by its owner.

"Aha!" said the Cock. "No doubt you are very costly and he who lost you would give a great deal to find you. But as for me, I would choose a single grain of barleycorn before all the jewels in the world."

Precious things are without value to those who cannot prize them.
0386英文多読者 ◆iiDSZQODZc
垢版 |
2021/03/01(月) 15:01:58.58ID:zqQdFHVZ
http://read.gov/aesop/058.html

The Cock & the Jewel
A Cock was busily scratching and scraping about to find something to eat for himself and his family, when he happened to turn up a precious jewel that had been lost by its owner.
scrape Oを擦る
aboutは「あちこちで」という意味の副詞でscratchとscrapeの両方に掛かる
for himself...はfindに掛かる
,whenはカンマに注目
,until は非制限的に(この言い方正しいかわからないけど)「ついにVする」と訳す場合が多い
同じように「そのとき/すぐにVした」のように解釈すべきwhenかな
非制限の,becauseとかと同じように認識

scratchとscrapeはalliteration(頭韻法)だろう
これにあたって先程紹介された
elements of eloquenceのalliteration読んでみたけどそこでもすぐに不定詞関係詞節が出てきて笑った
…he of course needed a history book from which to work
これはwisdomの次の例と同じfromの使い方だろう
study from her book 彼女の本をもとに勉強[研究]する.
0387英文多読者 ◆iiDSZQODZc
垢版 |
2021/03/01(月) 15:36:13.51ID:zqQdFHVZ
"Aha!" said the Cock. "No doubt you are very costly and he who lost you would give a great deal to find you. But as for me, I would choose a single grain of barleycorn before all the jewels in the world."
would give...について
wisdomより
I would give nything [my r�ght �rm, the w�rld,a agreat d�al] to do
…するために,どんなことでもやってみせる
これって仮定法として捉えてもいいかな? 
to getできるならwould giveするっていう
before Oに優先して
まぁさっきのwould rather thanのようなもの

cornといったところでトウモロコシのことを指すとは限らないので注意
ここでは大麦の粒のこと
ただmerriam websterだと
barleycornをa grain of barleyと説明してるからgrain of a grain of barleyやんけとは思う
barleycornはbarleyというcorn(穀物)の種類
ということかな
Precious things are without value to those who cannot prize them.
豚に真珠 
貴重なものも、それを評価できない人々とっては、価値を持たないということかな
0388英文多読者 ◆iiDSZQODZc
垢版 |
2021/03/01(月) 17:13:26.21ID:zde7O7Y4
JBR Collectionの続き
http://read.gov/books/pageturner/aesops_fables/#page/16/mode/2up
 
THE WOLF AND THE LAMB.
lamb子羊 ラム肉で親しみのある単語
A HUNGRY Wolf one day saw a Lamb drinking at a stream, and wished to frame some plausible excuse for making him his prey.
frameは「作成する、考え出す」でも「言い述べる」どっちで捉えてもいいかな
まぁ述べる かな
plausibleもっともらしい
preyは餌食のこと
fall prey to Oの犠牲となる という言い方でも使える
How many brothers fell victim to the streets?
は2PacのLife Goes Onにある一節

"What do you mean by muddling the water I am going to drink ?" fiercely said he to the Lamb. " Pray forgive me," meekly answered the Lamb; "I should be sorry in any way to displease you, but as the stream runs from you towards me, you will see that such cannot be the case."
muddleは適切な訳がwisdomにはないね
merriam websterより
muddle : to make turbid or muddy
mudか来てるんだろうね
displease Oを不快にする
to displease youが条件を表し、shouldは仮定法のshouldかな
wisdomより
If I had money, I should [would] be glad to go with you. お金があったら私は喜んで君と一緒に行くのですが
あ、でもそれだとin any wayが浮く?
ちょいあとで考える
the caseは実情の意味
Such was the case with him. 彼についてはそういう事情だった
Prayは古い言い方でpleaseの意味
meeklyおとなくしく、従順に
"That's all very well," said the Wolf; "but you know you spoke ill of me behind my back a year ago." " May, believe me," replied the Lamb, "I was not then born." " It must have been your brother then," growled the Wolf. " It cannot have been, for I never had any," answered the Lamb.
That's all very well, but...「それは大いに結構だが…」
speak ill of Oの悪口を言う は受験でも定番
behind my backが本当に背中の後ろではないことは明らかだが、一応辞書にも載っている
behind A's back:Aのいないところで
mayを辞書で引こうとしたら
Maoism:毛沢東主義が出てきた
祈願文で主語がないのか?
わからん
"I know it was one of your lot," rejoined the Wolf, "so make no more such idle excuses." He then seized the poor Lamb, carried him off to the woods, and ate him.
lot はfolksとかと同じく「仲間」の意味かな
rejoin言い返す
idle無益な
seize Oを掴む
poorは「かわいそうな」
carry A offの命を奪い去るという意味もあるけど今回は
carry A offを攫う to woods森へ
かな
0390英文多読者 ◆iiDSZQODZc
垢版 |
2021/03/01(月) 17:25:05.30ID:zde7O7Y4
I should be sorry to believe half of them.
なんてのがHurry Potterにあるので
その質問でもネイティブがshould = would と解説しているので仮定法で問題なさそう
0391英文多読者 ◆iiDSZQODZc
垢版 |
2021/03/01(月) 18:59:02.10ID:zde7O7Y4
>>322
結局ビジュアル英文解釈は誤訳ではなく俺の誤訳っぽいです
さっそくthe elements of eloquenceが役に立ちました
ありがとうございます
transferred epithet(転移修飾)に近い用法に思われます
try to explain it to them in a way that everyone will understand
は「way(←生徒が理解する) 」としながら実際は「it(←生徒が理解する)」ということ

きっかけはMr. BIGの次の出題です

https://twitter.com/Kazuma_Kitamura/status/993783968804757504?s=19

Amongst the several paths that are in the same road, I choose that to walk in which seems to be the straightest and cleanest.

これは関係詞を含むto不定詞がpied piping(道連れ移動)され、結局関係詞節の内容は
to walk in that seems to be the straightest and cleanest
そして「to walk in that = the straightest and cleanest」であるが実際には「that = the straightest and cleanest」
https://twitter.com/5chan_nel (5ch newer account)
0392英文多読者 ◆iiDSZQODZc
垢版 |
2021/03/01(月) 19:10:00.23ID:zde7O7Y4
ひえーん
ビジュアル英文解釈でこんなに悩むとは思わなかった
4,5年前から見てて、うんうんと悩めばMr. BIGのTweetが浮かぶぐらい読んでた
著書の英文解体新書もおすすめ
(まだちゃんと読んでない)

まぁ副詞節であることは否定しにくいけど、ネイティブも関係代名詞と回答してたみたいなのでtransferred epithetsと解釈することにします
0393英文多読者 ◆iiDSZQODZc
垢版 |
2021/03/01(月) 19:10:18.88ID:zde7O7Y4
厳密にはtransferred epithetsじゃないだけどそれに近い用法として理解します
0396英文多読者 ◆iiDSZQODZc
垢版 |
2021/03/01(月) 19:43:20.87ID:zde7O7Y4
itを修飾してるわけじゃないしな
that節がwayを修飾してるのは確実
形でも意味の上でも
でもunderstandの意味の上での目的語はitなんだよ
それを省略されていると捉えれば
thatは副詞節で済む

でも、省略されておらず前に出ている関係代名詞であるならばthatは関係代名詞
それはそれでおかしい
「(それ自身が)理解されるような方法」
は意味がわからない
understand wayと言いながらも意味は
understand it
これはtransferred epithetsではないけどなにか変なことが起こっているとしか考えられん
だって関係代名詞って言ってもみんなに否定されるし
0399英文多読者 ◆iiDSZQODZc
垢版 |
2021/03/01(月) 20:10:22.30ID:zde7O7Y4
関係代名詞とすると
they will understand the wayといいながら、実際はthey will understand itを表していることになる

transferred epithetsの
He smoked a nervous cigarette.
のnervousが実際は意味の上でHeを修飾しているのと似ているではないかと思った
もちろんepithetじゃないけどね
最初はそう思った

今はわからん
0401英文多読者 ◆iiDSZQODZc
垢版 |
2021/03/01(月) 22:15:19.38ID:zde7O7Y4
戦略的撤退をします

http://read.gov/aesop/063.html
ではさっきのJBRと同じものをAesop for Childrenバージョンで読みます

A stray Lamb stood drinking early one morning on the bank of a woodland stream. That very same morning a hungry Wolf came by farther up the stream, hunting for something to eat. He soon got his eyes on the Lamb. As a rule Mr. Wolf snapped up such delicious morsels without making any bones about it, but this Lamb looked so very helpless and innocent that the Wolf felt he ought to have some kind of an excuse for taking its life.

"How dare you paddle around in my stream and stir up all the mud!" he shouted fiercely. "You deserve to be punished severely for your rashness!"

"But, your highness," replied the trembling Lamb, "do not be angry! I cannot possibly muddy the water you are drinking up there. Remember, you are upstream and I am downstream."

"You do muddy it!" retorted the Wolf savagely. "And besides, I have heard that you told lies about me last year!" "How could I have done so?" pleaded the Lamb. "I wasn't born until this year."

"If it wasn't you, it was your brother!"

"I have no brothers."

"Well, then," snarled the Wolf, "It was someone in your family anyway. But no matter who it was, I do not intend to be talked out of my breakfast."

And without more words the Wolf seized the poor Lamb and carried her off to the forest.

The tyrant can always find an excuse for his tyranny.
The unjust will not listen to the reasoning of the innocent.
0402英文多読者 ◆iiDSZQODZc
垢版 |
2021/03/01(月) 23:22:00.27ID:zde7O7Y4
The Wolf & the Lamb

A stray Lamb stood drinking early one morning on the bank of a woodland stream. That very same morning a hungry Wolf came by farther up the stream, hunting for something to eat. He soon got his eyes on the Lamb. As a rule Mr. Wolf snapped up such delicious morsels without making any bones about it, but this Lamb looked so very helpless and innocent that the Wolf felt he ought to have some kind of an excuse for taking its life.
strayはぐれた、野良の
reservoir dogsという言葉を思い出した
調べてみたら1992年の映画らしい
耳にしたのはRuff Neckの曲のタイトルとしてhttps://youtu.be/t412KYvszsg
bank土手
woodland森林地帯
veryはsameが表す同一性を強調するもの
★以下不安
come by〜に立ち寄る
目的語はthe riverであり、farther upはそれを修飾のようにも考えたが違いそう
come byは「立ち寄る」の自動詞
upは前置詞
up the riverは「川に沿って」ではなく「川の上の方に」
fartherは「より遠く」の意味だが、羊との位置関係を表しており、upによりどこから遠いのかが分かる
羊と比べて河口からより遠い、つまり「羊より上流に」ということ
snap A upに飛びつく
snapと聞いて思い出すのはeminemの"snap back to reality"
morselごちそう
as a rule普通は
make no bones about O:Oについて言い訳をしない
つまり襲うことに言い訳を見繕わないということ
今回はwithoutとanyが相関してnoの意味になっているので問題ない
soの時点で後ろのthatなどを予想
helpless無力な
take one's life:〜を殺す
0403英文多読者 ◆iiDSZQODZc
垢版 |
2021/03/01(月) 23:43:57.85ID:zde7O7Y4
疑問点は確認しやすいように★つけてく

>>135
★破格の挿入らしきもの(ここは来期教授に聞けたら聞く)
>>177
★のof 55(ここはよくわからんけどまぁいいやって感じ)
0404名無しなのに合格
垢版 |
2021/03/02(火) 01:37:54.92ID:uvbcrxEk
>>403
ofはこうじゃないの?

of the 55 delegates who did show up in Philadelphia, no more than 30 stayed

no more than 30 of the 55 delegates who did show up in Philadelphia stayed

参加した74人の代議員の内19人は一度も出席したことがなく、フィラデルフィアに現れた55人の代議員の内4ヶ月間ずっと滞在したのは30人以下だった
0406名無しなのに合格
垢版 |
2021/03/02(火) 02:17:58.56ID:uvbcrxEk
良かった
英多に分からないって言われると自分が間違えてるのかとw
0407英文多読者 ◆iiDSZQODZc
垢版 |
2021/03/02(火) 02:54:33.80ID:YcFhHnNE
"How dare you paddle around in my stream and stir up all the mud!" he shouted fiercely. "You deserve to be punished severely for your rashness!"
How dare SVは「よくも…できるな」という定型表現
puddleはwisdomにないな
merriam-websterより
puddle: to dabble or wade around in a puddle
wisdomより
dabble (in O):(Oに)手を出す、手をバシャつかせる
wade:(水などの中を)歩く 
paddle and stir up: V1 and V2という並列だが"V1 and therefore V2"でありV2はV1の結果
the CGELにはV1 and V2の並列からimplicateする意味として次のものが載っている
V1 and then V2
V1 and therefore V2 (今回)
if V1 then V2
despite V1, V2
V1 while V2
ちなみにtherebyなどのthere+前置詞系の単語は「前置詞 + it/ them」で解釈する。
例えばthereuponは"upon it/them"
対応するものにwhere+前置詞系の単語もある。こちらは「前置詞+which」で解釈する
whereuponは"upon which"だしwhereforeは"for which"
これじゃできなそうなやつもあるけど、例外なのか、俺の勘違いか
deserve to VbはVに値する、当然だ 
で今回はto be Vppなだけだけど
Vingが来ることもあるみたい
John deserves to be punished.
John deserves punishment.
John deserves punishing.
これは名詞的動名詞かなぁ
rashness軽率さ、命知らず、軽はずみ
0409英文多読者 ◆iiDSZQODZc
垢版 |
2021/03/02(火) 03:05:13.55ID:YcFhHnNE
Another issue that deserves careful monitoring is illegal immigration to the United States.
形容詞がついてて名詞的動名詞なのは見つかりますね
0410英文多読者 ◆iiDSZQODZc
垢版 |
2021/03/02(火) 03:31:10.25ID:YcFhHnNE
"But, your highness," replied the trembling Lamb, "do not be angry! I cannot possibly muddy the water you are drinking up there. Remember, you are upstream and I am downstream."
Highness殿下、妃殿下
これはコードギアスで覚えた単語
Majesty陛下も出てきた
tremble身震いする
muddy Oを泥で汚す
possibly: canやcouldと用いて否定文だと「どうしても…できない」の意味を表す肯定文だと「なんとかして/できるかぎりの」
drink upとあると「飲み干す」の意味のupと捉えたくなるけどそうではなく上流ということ
upstream上流で/の
downstream下流で/の
"You do muddy it!" retorted the Wolf savagely. "And besides, I have heard that you told lies about me last year!" "How could I have done so?" pleaded the Lamb. "I wasn't born until this year."
retort言い返す
plead懇願する主張する
wasn't bornというのが状態を表しているためuntilが使用可能
"If it wasn't you, it was your brother!"

"I have no brothers."

"Well, then," snarled the Wolf, "It was someone in your family anyway. But no matter who it was, I do not intend to be talked out of my breakfast."
snarlうなる
このtalkはtalk A out of Vingの変化系かな
my breakfastはme having (you as) breakfast

byの意味ではないよね
And without more words the Wolf seized the poor Lamb and carried her off to the forest.

The tyrant can always find an excuse for his tyranny.
tyrant暴君
tyranny暴力的な行為、暴政
The unjust will not listen to the reasoning of the innocent.
the 形容詞で「〜な人」の意味
reason Oを説得して…させる(into Ving)、やめさせる(out of Ving)
0411英文多読者 ◆iiDSZQODZc
垢版 |
2021/03/02(火) 04:13:45.02ID:YcFhHnNE
グリム童話って英文解釈に最適では?
元から短いから短文で出題するために文脈から切り離す必要もないし
History.comに比べたら構造の難易度も控えめ
0412英文多読者 ◆iiDSZQODZc
垢版 |
2021/03/02(火) 04:16:52.99ID:YcFhHnNE
今まで読んだ英文まとめます
◎は構文的に面白かったポイント
★は決定的な不明点残ってる(それ以外も中途半端に済ませてるのはある)
。は中断
・・・Time For Kids・・・
>>2海のsound pollutionについて
>>6蛇について
>>66古代の壁画
>>164法螺貝
0415英文多読者 ◆iiDSZQODZc
垢版 |
2021/03/02(火) 04:18:50.42ID:YcFhHnNE
・・・Aesop's Fables・・・()は翻訳のバージョン
>>180The Old Lion & the Fox(Aesop for Children)
>>183The Man & the Lion(Aesop for Children)
>>185The Milkmaid & Her Pail(Aesop for Children)
>>229The Frogs & the Ox(Aesop for Children)
>>335Belling the Cat(Aesop for Children)
0416英文多読者 ◆iiDSZQODZc
垢版 |
2021/03/02(火) 04:21:20.69ID:YcFhHnNE
>>351。★EDITOR'S PREFACE(J.B.R. Collection)
>>353THE TWO FROGS(J.B.R. Collection)
>>356JUPITER AND THE CAMEL(J.B.R. Collection)
>>359THE LION HUNTING WITH OTHER BEASTS(J.B.R. Collection)
>>361The lion's share(Aesop for Children)
0417英文多読者 ◆iiDSZQODZc
垢版 |
2021/03/02(火) 04:24:28.40ID:YcFhHnNE
>>364THE STAG LOOKING INTO THE POOL(J.B.R. Collection)
>>370The Stag & His Reflection(Aesop for Children)
>>371THE COOK AND THE JEWEL(J.B.R. Collection)
>>386The Look & the Jewel(Aesop for Children)
>>388THE WOLF AND THE LAMB(J.B.R. Collection)
0419英文多読者 ◆iiDSZQODZc
垢版 |
2021/03/02(火) 04:29:07.01ID:YcFhHnNE
あ、Jack the Ripper以外
なにも◎つけてない
まぁ悩んで解決して驚いたのはこれぐらいか
(途中で◎つけるの忘れてた)
0420英文多読者 ◆iiDSZQODZc
垢版 |
2021/03/02(火) 04:32:15.81ID:YcFhHnNE
イソップ物語は俺の適合食材と見た
食らいついてく
最初の方適当読みしてたけどちゃんと読むようにしよ
0422英文多読者 ◆iiDSZQODZc
垢版 |
2021/03/02(火) 05:40:29.83ID:YcFhHnNE
(おそらく)402話あるJ.B.R. Collection
これに加えて(おそらく)145話あるAesop for Childrenも読む
これをやりきったときの自分の読解力が怖い
(ジャンルも書いた人(翻訳者)も限定的なので一般的な読解力がどこまで上がるか疑問+読み切れるかは微妙)
進捗率16/547(3%)
Belling catから計算すると
12/2(話/日)で1日当たり6話ペースなので
89日間で終わりますね
まぁこの2日間は時間あったから6話読めたってのもあるけどこの間に他の読んだり悩んだりもしてたからAesopに集中すればまた別の話。
9月末までに2冊読み切るのが目標
0423英文多読者 ◆iiDSZQODZc
垢版 |
2021/03/02(火) 05:53:16.77ID:YcFhHnNE
次は
THE PEACOCK'S COMPLAINT.

ということでまた明日(さっきのカウントミスしてるね、prefaceを除くと15話しか読んでない)

http://read.gov/books/pageturner/aesops_fables/#page/18/mode/2up

THE Peacock complained to Juno that while every one laughed at his voice, an insignificant creature like the Nightingale had a note that delighted everybody. Juno, angry at the unreasonableness of her favourite bird, scolded him in the following terms : "Envious bird that you are, I am sure you have no cause to complain. On your neck shine all the colours of the rainbow, and your extended tail shows like a mass of gems. No living being has every good thing to its own share. The falcon is endowed with swiftness, the eagle, strength, the parrot, speech, the raven, the gift of augury, and the nightingale with a melodious note, while you have both size and beauty. Cease then to complain, or the gifts you have shall be taken away."
0424英文多読者 ◆iiDSZQODZc
垢版 |
2021/03/02(火) 06:29:50.90ID:YcFhHnNE
>>404
あ、修飾の話とは別だけど
no more than は「30以下」ではなく「たった30人」で少ないことを強調する表現
no less thanは多さを表す

数学とかで
no more than Xを使うと
not more than Xと同じ「X以下」を表すこともあるみたいだけど
0425英文多読者 ◆iiDSZQODZc
垢版 |
2021/03/02(火) 08:12:19.35ID:YcFhHnNE
次は
THE PEACOCK'S COMPLAINT.
peacockクジャク

THE Peacock complained to Juno that while every one laughed at his voice, an insignificant creature like the Nightingale had a note that delighted everybody.
insignificant取るに足らない
note鳴き声
Junoユノ >>356に登場したJupiterの妻
夫妻揃って文句言われてるのね

Juno, angry at the unreasonableness of her favourite bird, scolded him in the following terms
unreasonable理不尽である
今回は名詞形のため
鳥に対する理不尽な物言いに怒ったということ
terms言い方、言葉遣い

: "Envious bird that you are, I am sure you have no cause to complain. On your neck shine all the colours of the rainbow, and your extended tail shows like a mass of gems.
envious嫉妬深い
Envious bird that you areに関しても気になる点がある
まず左方転移かどうか
左方転移はThe fossil, they found it in Montana.(α英文法より)みたいなやつね
Envious birdがなぜ無冠詞なのか
普通名詞の固有名詞化も浮かぶが、そうするとbirdのbも大文字となるだろう
他にも大雑把にいうと「補語云々のとき」「同格のとき」などに加えて「呼びかけのとき」の場合も無冠詞になるとある
Please, Doctor, save him.(α英文法)
補語だからという理由も考えようとしたが
呼びかけであるとすれば問題ないか
呼びかけで終了 左方転移じゃないっしょ
cause(正当な)理由
On your neck shine all the colours of the rainbowはMVSの倒置
show(〜に)見える の自動詞
a mass of O:大量のO
a gemは「宝石」 ソシャゲとかでよくあるやつ

No living being has every good thing to its own share. The falcon is endowed with swiftness, the eagle, strength, the parrot, speech, the raven, the gift of augury, and the nightingale with a melodious note, while you have both size and beauty. Cease then to complain, or the gifts you have shall be taken away."
★share分け前、割り当
wisdomにあるhave A to oneselfみたいな感じかな
ここ微妙
falcon鷹
swiftnessすばやさ
ここでのspeechは「言語能力」のこと
ravenオオガラス、ワタリガラス
augury予言
giftは才能でいいかな
augury raven ってカードがMTGにあるみたい。
占い鴉と訳されていて能力は予顕
melodious旋律の美しい、心地よい旋律の
whileは対比
cease to Vbするのをやめる
thenは「従って、それゆえ」の意味
shallは「〜することとなろう」のshall
Jupiterと違って優しい
0426英文多読者 ◆iiDSZQODZc
垢版 |
2021/03/02(火) 08:29:18.78ID:YcFhHnNE
>>425
この英文によって>>356のshouldが時制の一致よるものでshallの意味であるとわかる
まぁ分かってたけど
分かってなくても読んでいったら気付けることもあるかもね

ちなみに>>322は提喩とかそこらへんだと考えています
0427名無しなのに合格
垢版 |
2021/03/02(火) 09:27:19.10ID:uvbcrxEk
>>424
ホンマやん
なんかnotだと思ってた
0430英文多読者 ◆iiDSZQODZc
垢版 |
2021/03/02(火) 11:46:59.55ID:YcFhHnNE
次はこちら
urlは先程と同じ(ページも同じだから)

THE CAT AND THE MICE.

A CERTAIN house was much infested by Mice; the owner brought home a Cat, a famous mouser, who soon made such havoc among the little folk, that those who remained resolved they would never leave the upper shelves. The Cat grew hungry and thin in consequence, and, driven to her wit's end, hung by her hind legs to a peg in the wall, and pretended to be dead. An old Mouse came to the edge of the shelf, and, seeing through the deception, cried out, " Ah, ah, Mrs. Pussy! We should not come near you, even if your skin were stuffed with straw."
0431英文多読者 ◆iiDSZQODZc
垢版 |
2021/03/02(火) 12:04:50.53ID:YcFhHnNE
THE CAT AND THE MICE.

A CERTAIN house was much infested by Mice; the owner brought home a Cat, a famous mouser, who soon made such havoc among the little folk, that those who remained resolved they would never leave the upper shelves.
infest Oにはびこる
be infested with Oが群がっている(今回はbyになっている)
a mouserネズミを捕る動物
make Oを生じさせる
havoc大混乱、大損害
suchに呼応するthat
resolve that SVしようと決意する
would:意志のwillが時制の一致でwouldになっている
The Cat grew hungry and thin in consequence, and, driven to her wit's end, hung by her hind legs to a peg in the wall, and pretended to be dead. An old Mouse came to the edge of the shelf, and, seeing through the deception, cried out, " Ah, ah, Mrs. Pussy! We should not come near you, even if your skin were stuffed with straw."
driven...は分詞構文
drive A to Bに追いやる の意味でいいのかな
wit理性かな知恵かな
the hind legs後ろ足
pegフック
pretend to Vするふりをする
see through Oを見抜く
deception欺瞞
pussycat≒pussy≒ネコちゃん(小児)
pussy腰抜け
be stuffed いっぱいである、詰められている
straw麦わら
ここらへんは仮定法
0432英文多読者 ◆iiDSZQODZc
垢版 |
2021/03/02(火) 16:38:33.42ID:/Gk5GsdW
次はhttp://read.gov/aesop/134.html
先程のsop for Childrenバージョン

The Cat & the Old Rat

There was once a Cat who was so watchful, that a Mouse hardly dared show the tip of his whiskers for fear of being eaten alive. That Cat seemed to be everywhere at once with his claws all ready for a pounce. At last the Mice kept so closely to their dens, that the Cat saw he would have to use his wits well to catch one. So one day he climbed up on a shelf and hung from it, head downward, as if he were dead, holding himself up by clinging to some ropes with one paw.

When the Mice peeped out and saw him in that position, they thought he had been hung up there in punishment for some misdeed. Very timidly at first they stuck out their heads and sniffed about carefully. But as nothing stirred, all trooped joyfully out to celebrate the death of the Cat.

Just then the Cat let go his hold, and before the Mice recovered from their surprise, he had made an end of three or four.

Now the Mice kept more strictly at home than ever. But the Cat, who was still hungry for Mice, knew more tricks than one. Rolling himself in flour until he was covered completely, he lay down in the flour bin, with one eye open for the Mice.

Sure enough, the Mice soon began to come out. To the Cat it was almost as if he already had a plump young Mouse under his claws, when an old Rat, who had had much experience with Cats and traps, and had even lost a part of his tail to pay for it, sat up at a safe distance from a hole in the wall where he lived.

"Take care!" he cried. "That may be a heap of meal, but it looks to me very much like the Cat. Whatever it is, it is wisest to keep at a safe distance."

The wise do not let themselves be tricked a second time.
0433英文多読者 ◆iiDSZQODZc
垢版 |
2021/03/02(火) 17:02:01.14ID:/Gk5GsdW
The Cat & the Old Rat
There was once a Cat who was so watchful, that a Mouse hardly dared show the tip of his whiskers for fear of being eaten alive. That Cat seemed to be everywhere at once with his claws all ready for a pounce. At last the Mice kept so closely to their dens, that the Cat saw he would have to use his wits well to catch one. So one day he climbed up on a shelf and hung from it, head downward, as if he were dead, holding himself up by clinging to some ropes with one paw.
so によってthat節を予想して, thatで安心する
カンマがあるのはなぜだろう
結果で非制限的だとこうなる場合があるのかな
watchful油断のない、注意深い
whisker(ネズミなどの)ひげ
pouce襲いかかること
at last とうとう
a witは機知の意味だね
ならさっきのもこれか
あとでdrivenについて考える
keep to Oに留まる
downward(s)下の方へ
as ifは少し注意が必要なので確認する
hold A upを留める
cling to Oにしがみつく
★seeは(can) see that SVと分かる の意味でいいのかな
もう少し辞書が欲しくなってきた
0434英文多読者 ◆iiDSZQODZc
垢版 |
2021/03/02(火) 17:06:26.84ID:/Gk5GsdW
>>431
Cambridge Dictionaryより
be at wit's end: to be so worried, confused, or annoyed that you do not know what to do next
英辞郎より
be at wit's end「途方に暮れて」
bring someone to his wit's end:〜を困らせる手こずらせる、〜が困り果てる
最後のこれが近いかな
bring がdriveになっただけ

調べるのめんどくさがったらだめだね
0435英文多読者 ◆iiDSZQODZc
垢版 |
2021/03/02(火) 17:29:10.43ID:/Gk5GsdW
>>433
as ifの確認
>>92で言ってたやつ
問題は過去の仮定にも拘らずなぜ

as if he were deadと仮定法過去なのかということ
まずは主節が現在形のものを確認
He speaks as if he were ashamed.は(実際に補うことはしないものの)次のような形が根底にある
He speaks as he would speak if he were ashamed. 
(英文法総覧より)
帰結節は省略されており、if以降は前提節であるため、主節は帰結節でも前提節でもなく仮定法と関係がない。(つまり、主節とif節の時制は分断されており、関係がない)
そのため主節の時制はas if節の時制に影響しない
as ifの時制を決めるのは主節の動作の前か前ではないかの順序であり、主節の時制ではない
例えば
She looks/looked as if she had seen a ghost.
(英文法解説より)
ちなみにas thoughのthoughは古い英語でthough=ifであった時代の名残り
またas ifの内容の事実性が高いほど直接法も用いられる
最後に英文法総覧より
「仮定法過去は,通例,「現在の事実に反する仮定」を示すものであり、 これが「過去の事実に反する仮定」を示すのは,きわめてまれなことである」
(参考:α英文法,英文法解説,英文法総覧,英文法詳解)
0436英文多読者 ◆iiDSZQODZc
垢版 |
2021/03/02(火) 18:00:06.98ID:/Gk5GsdW
>>401The Wolf & the Lamb(Aesop for Children)
>>425THE PEACOCKS COMPLAINT.(J.B.R. Collection)
>>431THE CAT AND THE MICE.(J.B.R. Collection)
>>433The Cat & the Old Rat(Aesop for Children)
結局英文多読日記の目次最新版
>>412-417,436
0437かばん屋
垢版 |
2021/03/02(火) 19:04:16.04ID:J9Z+2gpN
>32
suggest O は性質を示しているという意味なので特に「Citizenに対して」など考える必要はない。

>35
deeplの方が正しい
a number of hindrances に"lack of evidence"、"a gamut of misinformation and false testimony"、"tight regulations by the Scotland Yard"が並列で含まれる。

>44
OEDの用例と同様の従位接続詞を避ける用法と考えられるが、archaic and rareとされているし、英文法汎論でも古い用法とされているため、裏付けとしては不十分。
RadfordのColloquial Englishに以下の記載あり。一部のネイティブは容認するが容認しないネイティブも一定数いる用法。
A further context in which some speakers allow that in adverbial clauses is in the second conjunct of coordinate subordinate clauses like those below
(200)
a. There are great deals on offer, if you’ve got a decent credit score and that you behave in the right way (Martin Lewis, BBC Radio 5)
b. If we were sitting here talking about police officers and that they were blatantly breaking the law, you’d be rightly aggrieved
(Peter Kirkham, BBC Radio 5)
ジャックザリッパーの記事は面白い用例と言いたいところだがimpinge onと混同しているのか他動詞のはずのimpedeがimpede onとして使われているので微妙。

>135

that impolitic, unjust, and [ (one might add without much impropriety) scandalous] conduct

構文のブレンド
addの目的語がimpoliticやunjustと並列
疑似等位接続詞も同様の構文から生まれたと推測できる。

>175
minister は辞書を引けば公使のことだと分かったはず

>177
of the 55 delegatesは後ろno more than 30 stayedにつながって、55の代表者のうちわずか30人だけが4か月滞在したということ

>187
down fell the pail of milk (MVS)
方向の副詞が前置されて倒置が起きることがあるというのはフォレストなどの総合英語にも昔から掲載
asは考えているうちに・・・というような同時性を表す
with itは流れたミルクと「一緒に」という普通の用法

>352
wouldは助動詞 
Sparrows with farmers would shrewd talk(O) maintain(V)
SOV語順なのでmaintainは他動詞

>352
>425
Jupiter angrily told him that ... but that
butはtellの補部のthatを繋いでいるだけS said (that) ... and that ...と同様
shouldはshallがbackshiftしているだけ
2人称shallは別に優しくない。

◆2a【話者の意志・命令・強制】((形式・やや古))〔二・三人称の平叙文で〕…させよう,…させてやる,…やってもらう(◆((強))で発音;通例目下の者・子どもに用いる);(穏やかな約束・保証を表して)…させてあげよう,…してよろしい(◆((弱))で発音)◆

>375
affect OにはOを装うという意味がある。
装われた知恵→知恵(があるように)装った雰囲気で

>425
C that SV=C though SV=C as SV
譲歩構文だと気づいていない?
0438英文多読者 ◆iiDSZQODZc
垢版 |
2021/03/02(火) 19:07:10.18ID:/Gk5GsdW
>>437
辻斬りされた
ありがとうございます
本当に本物だったんだ
感謝
またなにかあればよろしくおねがいします
いつも勉強させてもらってます
0440英文多読者 ◆iiDSZQODZc
垢版 |
2021/03/02(火) 19:33:33.13ID:/Gk5GsdW
>32
なるほど

>35
なるほど

>44
なるほど

>135
なるほど?
英文解体新書のブレンドの章を読んでみます

>175
公使がなにか知らなかった
外交使節団の長のことらしい

>177
これは>>404の通りですね
ありがとうございます

>187
なるほど

>352
くそわらう
なるほど

>352
>425
なるほど

>375
なるほど

>425
気づいていませんでした
0441名無しなのに合格
垢版 |
2021/03/02(火) 21:36:39.06ID:cRdRkA0w
>>404
質問なんですが、これって文法でいうとなんていう文法項目なんでしょうか?倒置?なんでしょうか?
0442英文多読者 ◆iiDSZQODZc
垢版 |
2021/03/02(火) 21:48:37.56ID:/Gk5GsdW
>>441
前置詞句の前置と思われる
前置、倒置、後置に関しては>>63に書いた通り
例えばα英文法の特殊構文のところに書いてるよ

> 倒置という用語は広く使われすぎているため注意が必要 
> 例えばOSVやCSVというのは目的語Oや補語Cの前置という現象である。
> 逆に後置という現象も存在する
> 分離した同格のthat節などもそれ
> 倒置とはこれらの現象などによりVSという形になっているものを指す
0443英文多読者 ◆iiDSZQODZc
垢版 |
2021/03/02(火) 22:00:12.57ID:/Gk5GsdW
ちょうどα英文法の前置のところに
X as SVのことが書いてあったので読みます

譲歩のalthoughとthoughのうちthoughは文学的表現として次のようなものになる
Fail though I did, I would not give up the plan. (私は失敗したが、その計画をあきらめない)
as節が譲歩を表す場合もあり、そのときは必ずX as SVとなる。asの代わりにthoughも用いられる。
Freezing as it was, we went out.
=Freezing though it was, we went out. =Though it was freezing, we went out.
理由を表す場合もある
Freezing as it was, we entered the house. =Because it was freezing, we entered the house.
(凍えるような寒さだったので、私たちは家に入った)
米では as C as SVともなりうる
As freezing as it was, we went out.
As greatly as I admire him, his new paper is rather unsatisfactory.
名詞を使う場合は冠詞が省略
Genius as she was, she was quite unassuming.
(彼女は天才だったが, とても控えめだった)

次はMr. BIGのツイートより
being as Cas SV 「現実としてSが、
このようにCである状況で」
doing as 副詞 as 「現実としてS
が、このように副詞にvする状況で」

松井先生によると
although as C as SVというのもあるようです。
0444英文多読者 ◆iiDSZQODZc
垢版 |
2021/03/02(火) 22:10:22.04ID:/Gk5GsdW
thoughは前置
asは(ing as)X as SVの()省略だから発生は別なのね

あ、さっきのMr. BIGのは、それが基となっているよねという話
0445英文多読者 ◆iiDSZQODZc
垢版 |
2021/03/02(火) 22:25:02.75ID:2xU5QUwL
thatの例もwisdomにありました

文)譲歩理由を表す節を導いて1…であ
るけれども;…であるので(補語となる名/形がthatの直前に置かれる;名は単数でも無冠詞
となる

Adventurous spirit that she was, marriage proved to be the ruin of her. 彼女は冒険心のある女性だったが, 結婚は彼女を破滅させるものとなった(≒ Though she was an adventurous spirit, ...)

Terrible liar that she was, she continued to deny everything right up to the end. 彼女はひどいうそつきだったため, 最後の最後まですべてを否定し続けた(≒ As she was a terrible liar, ...).
0446英文多読者 ◆iiDSZQODZc
垢版 |
2021/03/02(火) 22:27:03.41ID:2xU5QUwL
ていうか左方転移云々言ってたけど
that you areを完全に見落としてた
忘れてたというより見てなかった
0447英文多読者 ◆iiDSZQODZc
垢版 |
2021/03/02(火) 22:29:29.73ID:2xU5QUwL
構文ブレンド意外は完璧に理解した
構文ブレンドはとりあえず今から解体新書読みます
0449名無しなのに合格
垢版 |
2021/03/02(火) 23:16:43.40ID:aEG7btNF
>>448
頑張れ
0450英文多読者 ◆iiDSZQODZc
垢版 |
2021/03/02(火) 23:38:44.92ID:2xU5QUwL
>>353
ちなみにロングマンより


GRAMMAR: Patterns with propose
In everyday English, you propose that someone does something:
I propose that Ms Hallam is appointed.
In more formal English, you propose that someone do something, using the base form of the verb (=infinitive without ‘to’):
I propose that Ms Hallam be appointed.
Don’t say: I propose Ms Hallam to be appointed.
You can also use the base form when talking about the past:
I proposed that Ms Hallam be appointed.
You propose that someone should do something:
He proposed that we should have another meeting.
This pattern is used especially in British English.
You say it was proposed that someone (should) do something:
It was proposed that we (should) have another meeting.
0454英文多読者 ◆iiDSZQODZc
垢版 |
2021/03/03(水) 04:15:05.43ID:juVRvDOv
>>433
続き読んでなかった

When the Mice peeped out and saw him in that position, they thought he had been hung up there in punishment for some misdeed. Very timidly at first they stuck out their heads and sniffed about carefully. But as nothing stirred, all trooped joyfully out to celebrate the death of the Cat.
少し二重表現法(repetitive word pairs)を疑った 違うけど
peep out: to appear slowly and not be completely seen
seeはsee OCでCに前置詞句が来ている
misdeed悪行
timidlyびくびくして
sniff around=sniff about:嗅ぎ回る
stirわずかに動く
troop群れをなして進む
Just then the Cat let go his hold, and before the Mice recovered from their surprise, he had made an end of three or four.
hold掴むこと、握っている手
hold手がかり、足がかり 今回はこれかな
let go of Oではないから
let O Vbで目的語が後置されてlet Vb Oになったのかな?
let A go: Aから手を離す
make an end of Oを殺す
Now the Mice kept more strictly at home than ever. But the Cat, who was still hungry for Mice, knew more tricks than one. Rolling himself in flour until he was covered completely, he lay down in the flour bin, with one eye open for the Mice.
be hungry for Oを渇望している、に飢えている
Sure enough, the Mice soon began to come out. To the Cat it was almost as if he already had a plump young Mouse under his claws, when an old Rat, who had had much experience with Cats and traps, and had even lost a part of his tail to pay for it, sat up at a safe distance from a hole in the wall where he lived.
sure enough予想した通り
have OM:OをMに置いておく連れて行く
plumpふっくらとしたまるまるとした
pay for Oの代償を支払う
★このtoはどの用法だろう
at a safe distance安全な距離で
sit up体を起こす
"Take care!" he cried. "That may be a heap of meal, but it looks to me very much like the Cat. Whatever it is, it is wisest to keep at a safe distance."
heap山、堆積
mealごはん
wise賢明な
The wise do not let themselves be tricked a second time.
the first time一回目の時には
一般論だからaになってるのかな
0458名無しなのに合格
垢版 |
2021/03/03(水) 12:10:27.79ID:jojDkN1Q
英語主席の京大医学部生がyoutubeを始めたよ
たしか133/150とかだった記憶
冠模試で2位だったmr.big氏といい勝負してる
0459名無しなのに合格
垢版 |
2021/03/03(水) 13:41:33.63ID:jojDkN1Q
Toeflはibt換算で117とかだった
0461英文多読者 ◆iiDSZQODZc
垢版 |
2021/03/03(水) 14:36:25.28ID:ODrTxpub
有名なやつですね

THE DOG AND HIS SHADOW.

A DoG, bearing in his mouth a piece of meat that he had stolen, was crossing a smooth stream by means of a plank. Looking in, he saw what he took to be another dog carrying another piece of meat. Snapping greedily to get this as well, he let go the meat that he had, and lost it in the stream
0462英文多読者 ◆iiDSZQODZc
垢版 |
2021/03/03(水) 14:53:26.89ID:ODrTxpub
THE DOG AND HIS SHADOW.

A DOG, bearing in his mouth a piece of meat that he had stolen, was crossing a smooth stream by means of a plank. Looking in, he saw what he took to be another dog carrying another piece of meat. Snapping greedily to get this as well, he let go the meat that he had, and lost it in the stream.
by means of Oを用いて
a plank板
look in覗く
he saw what he took to be another dog carrying another piece of meat.

he saw (O:what he took φ to be another dog) (C:carrying another piece of meat)
彼が別の犬と見なしたものが別の肉の欠片を抱えているのを見た
greedily貪欲に
snapかみつくパッと動く
as well…もまた
またlet go ofのofがない
let O Vbの目的語後置かな
0463英文多読者 ◆iiDSZQODZc
垢版 |
2021/03/03(水) 15:00:03.48ID:ODrTxpub
次はこれ
これもJ.B.R. Collection
J.B.R. Collectionだけにする

THE ANT AND THE FLY.

An Ant and a Fly one day disputed as to their respective merits. " Vile creeping insect !" said the Fly to the Ant, " can you for a moment compare yourself with me? I soar on the wing like a bird. I enter the palaces of kings, and alight on the heads of princes, nay, of emperors, and only quit them to adorn the yet more attractive brow of beauty. Besides, I visit the altars of the gods. Not a sacrifice is offered but is first tasted by me. Every feast, too, is open to me.
I eat and drink of the best, instead of living for days on two or three grains of corn as you do." listen to me. "All that's very fine," replied the Ant; " but You boast of your feasting, but you know that your diet is not always so choice, and you are some times forced to eat what nothing should induce me to touch. As for alighting on the heads of kings and emperors, you know very well that whether you pitch on the head of an emperor, or of an ass (and it is as often on the one as the other), you are shaken off from both with impatience.
And, then, the altars of the gods,' indeed! There and everywhere else you are looked upon as nothing but a nuisance. In the winter, too, while I feed at my ease on the fruit of my toil, what more common than to see your friends dying with cold, hunger, and fatigue? I lose my time now in talking to you. Chattering will fill neither my bin nor my cupboard."
0466英文多読者 ◆iiDSZQODZc
垢版 |
2021/03/03(水) 15:35:04.32ID:ODrTxpub
先に置いておく

11話目
THE STAG IN THE OX-STALL.

A STAG, hard pressed by the hounds, ran for shelter into an ox-stall, the door of which was open. One of the Oxen turned round, and asked him why he came to such a place as that, where he would be sure to be taken. The Stag replied that he should do well enough if the Oxen would not tell of him, and, covering himself in a heap of straw, waited for the night.
Several servants, and even the Farm-Bailiff himself, came and looked round, but saw nothing of the Stag, who, as each went away, was ready to jump out of his skin for joy, and warmly thanked the Oxen for their silence. The Ox who had spoken first to him warned him not to be too sure of his escape, and said that glad as they would all be for him to get away, there was a certain person still to come whose eyes were a deal sharper than the eyes of any one who had been there yet.
This was the Master himself, who, having been dining with a neighbour, looked in on his way home to see that all was right. At a glance he saw the tips of the horns coming through the straw, whereupon he raised a hue and . cry, called all his people together, and made a prize of the Stag.

12話目
THE FROG WHO WISHED TO BE AS BIG AS AN OX.

An Ox grazing in a meadow, chanced to set his foot on a young Frog and crushed him to death. His brothers and sisters, who were playing near, at once ran to tell their mother what had happened. " The monster that did it, mother, was such a size!" said they. The mother, who was a vain old thing, thought that she could easily make herself as large.
"Was it as big as this ?" she asked, blowing and puffing herself out. " Oh, much bigger than that," replied the young Frogs. As this then ?" cried she, puffing and blowing again with all her might. Nay, mother," said they; if you were to try till you burst yourself, you would never be so big." The silly old Frog tried to puff herself out still more, and burst herself indeed.

13話目
THE HAWK AND THE NIGHTINGALE.

A NIGHTINGALE once fell into the clutches of a hungry Hawk who had been all day on the look-out for food. Pray let me go," said the Nightingale, " I am such a mite for a stomach like yours. I sing so nicely too. Do let me go, it will do you good to hear me." " Much good it will do to an empty belly," replied the Hawk, "and besides, a little bird that I have is more to me than a great one that has yet to be caught."
0468名無しなのに合格
垢版 |
2021/03/03(水) 21:03:47.43ID:TbQHI8R9
勉強法で質問なんですが,英作文ってどうやって勉強すればいいんでしょうか?
今は自由英作文で、内容をまず日本語で書いて、それを英語で表現を調べているのですが、時間がかかるし、
単語は見つかっても、日本語訳には現れない?使い方が間違っていたりして実際ネイティブは使わないんじゃないかとかって思い
全然勉強が進んでないです… どうやって主さんは英作文を勉強されましたか?
0469英文多読者 ◆iiDSZQODZc
垢版 |
2021/03/03(水) 21:46:43.55ID:mA0wn3AD
こんなことやってみた
でも時間かかるから全部はやらなくていいや
https://i.imgur.com/QzXszjs.jpg

ここで作りました
https://kouzouzu.web.fc2.com/

https://kouzouzu.web.fc2.com/kzzmaker.htm?%5Cn%5BM:%20One%20hot%20summer,%5D%5Cn%5BS:%20the%20lake%20%5Bm:%5BM:%20in%20which%5D%20%5BS:%20two%20Frogs%20%5D%5BV:%20lived%5D%5BM:t%5D%5D%5D%5BV:%20was%20completely%20dried%20up,%5D%20%5Cn
0470英文多読者 ◆iiDSZQODZc
垢版 |
2021/03/03(水) 22:09:23.43ID:nsQXxmV6
第1話 THE TWO FROGS.
One hot summer, the lake in which two Frogs lived was completely dried up, and they were obliged to set off in search of water elsewhere. Coming to a deep and deliciously cool well, one of the Frogs proposed that they should jump in at once. “Wait a bit,” cried the other: “if that should dry up, how could we get out again?
語法・構文 
・One hot summer, the lake in which two Frogs lived was completely dried up,
be Vippで完了を表す用法
・and they were obliged to set off in search of water elsewhere.
be obliged to Vbせざるをえない
be obligedで「感謝している」の意味にもなる
set off出発する
in search of Oを探して
elsewhereどこか他の場所で “in search of water”を修飾 
・Coming to a deep and deliciously cool well, one of the Frogs proposed that they should jump in at once.
deliciouslyとてもおいしく、愉快に、とても心地よく
a well 井戸
at onceすぐに
・one of the Frogs proposed that they should jump in at once.→
proposeに続くthat節内では直説法のほかに、硬い文体で仮定法現在やshouldが用いられる。
・if that should dry up, how could we get out again?
仮定法のためshouldになっており、対応して主節に助動詞の過去形couldがある。
dry up干上がる
和訳
ある暑夏、二匹のカエルの住む湖が完全に枯れ果ててしまい、二匹は他の場所で水場を探すため旅立つことを余儀なくされた。心地よい涼しさの深い井戸に到着すると、一匹のカエルがすぐに飛び込もうと提案した。「ちょっと待て!」もう一匹は叫んだ。「ここが枯れたら俺たちはどうやって再び地上に戻どるのだ。」
0471英文多読者 ◆iiDSZQODZc
垢版 |
2021/03/03(水) 22:24:10.60ID:NR96oYxx
>>468
単語の使い方は辞書見て、表現とかはludwigとかで調べるのもありなんじゃないかな?
わかんね

英作文の勉強をあまりしたことない。実は高校出てなくて予備校も高校一年の頃ちょろっと安河内先生の授業を受けたことがあるだけで、受験でもライティングなかったから和文英訳とかもしたことないし、通常の勉強の仕方もわからん(だから実際和文英訳も自由英作もクソ苦手。英検とTOEFLに向けて勉強しないとね)
高校出てたりちゃんと予備校通ってたら英作文の指導とか受けるのだろうけどね


自由英作に関してはTwitterの英語妹先生にSkype授業してもらったときに英検一級に向けて添削してもらったのと四軒家忍先生にTOEFLに向けて添削してもらったことがそれぞれ数回ある()
でも人に言えるほどのことはない
0472英文多読者 ◆iiDSZQODZc
垢版 |
2021/03/03(水) 23:25:16.56ID:NR96oYxx
parts of eloquenceのzeugmaのチャプターを読んでた
通常のをprozeugmaと呼んで、As you on him, Demetrius dote on youみたいなのをhypozeugmaって呼ぶのね
なるほ

>>304にある並列でのバランスってisocolonって名前あるらしい
おもろい
読んでみます
0473英文多読者 ◆iiDSZQODZc
垢版 |
2021/03/04(木) 00:45:39.11ID:qEVfK/Ab
第2話 JUPITER AND THE CAMEL.
THE Camel once upon a time complained to Jupiter that he was not as well served as he ought to be in the means of defence and offence.
" The bull," said he, "has horns, the boar, tusks, and the lion and tiger, formidable claws and fangs that make them feared and respected on all sides.
I, on the other hand, have to put up with the abuse of all who choose to insult me." Jupiter angrily told him that if he would take the trouble to think, he would see that he was endowed with qualities shared by no other beast; but that, as a punishment for his unreasonable importunity, henceforward his ears should be sh
語法・構文 
Jupiter木星あるいは神ユピテル
camelラクダ
・he was not as well served as he ought to be in the means of defence and offence.
★serve Oを与える “to furnish or supply with something needed or desired”の意味でよいのだろうか
★in …の点で の意味でよいのだろうか
defenceとoffenceはそれぞれ英での綴り
・" The bull," said he, "has horns, the boar, tusks, and the lion and tiger, formidable claws and fangs that make them feared and respected on all sides.
ここは所謂くびき語法(zeugma)により空所化が起こっている。適切な場所にhasを補えばよい。“lion and tiger, formidable claws”ではhaveでなくてはならないが、zeugmaではよくあることらしい。
bull雄牛
boreイノシシ
tusk牙(象やイノシシ系の)
formidable恐るべき
関係代名詞thatの先行詞はそれらの武器全て
on all sides至るところで
・I, on the other hand, have to put up with the abuse of all who choose to insult me.
abuse罵り
ofはabuseの意味上の主語を表している
・Jupiter angrily told him that if he would take the trouble to think, he would see that he was endowed with qualities shared by no other beast;
angrily怒った様子で
if he wouldのwouldは意志を表すwillが時制の一致を受けたもの
未来を表す副詞節内であれ、意志の意味であればwillは用いられる。
take the trouble: to make an effort
think頭を働かせる
he would seeは話者の推量を表す。
endow A with Bを与える
a quality特性
・but that, as a punishment for his unreasonable importunity, henceforward his ears should be shortened.
thatはtoldの目的語であり、told that… but that…という並列。
前言と対比する逆接であるためbutが使用されており、区切りを明確にするためbutの前にセミコロンが使用されている。
henceforth ≒ henceforward = この時点より
unreasonable道理をわきまえない
importunity執拗な要求
ここでのshouldはshallがback shiftしたもの
shallはwisdomだと(平叙文で; 二人称・三人称主語で)
(命令・指図)「…するものとする」
(約束) 「…させよう」、「…してやろう」
(予言)「…することとなろう」※話し手が意志を神などに託し, 予言を表す用法
真ん中のやつかな
wisdomでは(約束)の例文として
They shall be dismissed. (≒ I will dismiss them.)
和訳(やりにくい、くそなのは勘弁)
昔々、ラクダがユピテルに次のように不満を述べた。「攻撃と防御の手段という点において、自分は十分に与えられていません。もっと与えられるべきです。雄牛には角、イノシシには牙、ライオンとトラには恐るべき爪牙が与えられています。そのおかげで彼らはあらゆる場所で畏怖されています。それに比べて私はどのようなものからの罵りにも耐えなければなりません。」ユピテルは怒った様子で彼に言いました。「深く考えようとすれば、自分が他の獣にはない特質を持っていることに気づけるだろう。しかし、道理をわきまえない執拗な要求の罰として、以降お前の耳は短くしてやろう。」
0474英文多読者 ◆iiDSZQODZc
垢版 |
2021/03/04(木) 01:00:22.15ID:0aRM6rj1
★は不安なやつ
次からdeepLを使って、それを微調整します
存在を忘れてた
0475英文多読者 ◆iiDSZQODZc
垢版 |
2021/03/04(木) 02:59:30.26ID:0aRM6rj1
THE LION HUNTING WITH OTHER BEASTS.

A LION, a Heifer, a Goat, and a Sheep once agreed to share whatever each might catch in hunting. A fine fat stag fell into a snare set by the Goat, who thereupon called the rest together. The Lion divided the stag into four parts. Taking the best piece for himself, he said, "This is mine of course, as I am the Lion;" taking another portion, he added, “ This too is mine by right-the right, if you must know, of the strongest." Further, putting aside the third piece, " That's for the most valiant," said he; "and as for the remaining part, touch it if you dare."

・A LION, a Heifer, a Goat, and a Sheep once agreed to share whatever each might catch in hunting.
a heifer若い雌牛
agree to Vするのに同意する
whatever SV:SVしたどんなものでも
mightは譲歩のmayがback shiftしたもの
(back shiftって言い方かっこよかったから真似します)
in Vingするときに
・A fine fat stag fell into a snare set by the Goat, who thereupon called the rest together.
fine立派な
a stag雄鹿
fall into a trap罠に嵌まる
a snareわな(足を縄で締めるタイプ)
set a trap罠を仕掛ける
thereuponそのすぐあとに≒upon it
therebyなどのthere+前置詞の単語は「前置詞+it/them」で解釈すればよい
call A togetherを召集する、呼び集める
・Taking the best piece for himself, he said, "This is mine of course, as I am the Lion;"
divide A into B:Aを分割してBにする
★take A for B
これはAをBと認識する
というよりAをBに持っていくの意味かな
・taking another portion, he added, “ This too is mine by right-the right, if you must know, of the strongest."
a portion一人分
by right of Oの権利によって
the right of the strongestの間に挿入が存在している
if you must knowそんなに知りたければ教えるが
if you must know:used when you answer a question that you think someone should not have asked, because it is slightly impolite
must V:どうしてもVする(と言って聞かない)
・Further, putting aside the third piece,"That's for the most valiant," said he; "and as for the remaining part, touch it if you dare."
furtherさらにその上→文頭においてカンマを伴う"Further,"の形が多い
put A asideを脇にやる
valiant勇敢な
as for Oに関して
remaining残りの
dare (to) Vbする勇気がある
本文ではtouch itが省略されている

和訳
昔、ライオン、雌牛、ヤギ、ヒツジは、狩りで獲れるものは何でも分け合うことに合意しました。立派な太った雄鹿がヤギの仕掛けた罠に落ちてしまい、ヤギはすぐに残りの仲間を呼び集めました。ライオンは雄鹿を4つの部分に分けました。一番良い部分を自分のものにすると、「これはもちろん私のものである。私はライオンなのだ。」と言い、もう一つの部分を取ると、「これもとうぜん私のものだ。どうしても知りたければおしてやるが百獣の王の権利によってである。」さらに、彼は3つ目の部分を脇に置いて、「それは最も勇敢な人のためのものだ」と言い、「残りの部分については、触る勇気があるならば、それに触れてみろ」と言った。
0476英文多読者 ◆iiDSZQODZc
垢版 |
2021/03/04(木) 04:57:40.91ID:bBtImSs2
第4話 THE STAG LOOKING INTO THE POOL.
A STAG drinking at a clear pool, admired the handsome look of his spreading antlers, but was much displeased at the slim and ungainly appearance of his legs. "What a glorious pair of branching horns !" said he. “ How gracefully they hang over my forehead! What an agreeable air they give my face! But as for my spindle-shanks of legs, I am heartily ashamed of them." The words were scarcely out of his mouth, when he saw some huntsmen and a pack of hounds making towards him. His despised legs soon placed him at a distance from his followers, but, on entering the forest, his horns got entangled at every turn, so that the dogs soon reached him and made an end of him. " Mistaken fool that I was !" he exclaimed ; " had it not been for these wretched horns my legs would have saved my life."
0477英文多読者 ◆iiDSZQODZc
垢版 |
2021/03/04(木) 05:00:05.88ID:bBtImSs2
・A STAG drinking at a clear pool, admired the handsome look of his spreading antlers, but was much displeased at the slim and ungainly appearance of his legs.
a stag雄鹿
a pool水たまり admire Oに見惚れる
handsome均整の取れた見事な
spread大きく広がる antler枝角
(his spreading antlers look handsome)
displease Oを不快にする
be displeased at Oに不快感を抱いている
ungainly不格好な
(his legs look slim and ungainly)
・"What a glorious pair of branching horns !" said he. “ How gracefully they hang over my forehead!
glorious華麗な
branch枝分かれする
gracefully美しく優美に
hang over Oから突き出ている
・What an agreeable air they give my face! But as for my spindle-shanks of legs, I am heartily ashamed of them."
agreeable心地よい
air雰囲気
shanks脚、脛
spindle-shanks細く長い脚
heartily心から
be ashamed of Oを恥じている
・The words were scarcely out of his mouth, when he saw some huntsmen and a pack of hounds making towards him.
次の構文では主節に過去完了,従属節に過去形が多いが主節に過去形がくることもある
S1(scarcely/hardly)V2(when/before/than)S2V2:S1V1するとすぐにS2V2,S1V1するかしないかのうちにS2V2
thanはno sooner...than---との混交によって生まれたもの
S1 no sooner V1 than S2 V2:S1V1するとすぐにS2V2
※この形はno soonerを前置することにより、倒置を発生させる次の形が基本
No sooner V1S1 than S2V2
この構文ではS2V2が意外だったり、望ましくないことを表すが、次の構文ではその含みはない
S1V1 as soon as S2V2するとすぐにS1V1
huntsman猟師 a pack of Oの群れ
make towards Oへと進む
・His despised legs soon placed him at a distance from his followers, but, on entering the forest, his horns got entangled at every turn, so that the dogs soon reached him and made an end of him.
despise Oを軽蔑する
place O M:OをMに置く
a distance from Oから離れた場所
on ingするとすぐに、同時に
be entangled絡まっている、
turn曲線部分
so that SV:その結果SV
結果の場合カンマを伴い", so that"となることが多いがこれはto不定詞なども同じ。目的であれ結果であれ、thatは省略可能。少なくとも目的であれば、soを省略することもある
make an end of Oを殺す
・"Mistaken fool that I was !" he exclaimed ;
★このセリフの文法的な構造はなんだろう
譲歩や理由を表すものではないように思われる
it cleftのIt wasが省略されているのだろうか
it was mistaken fool that I was
「私は何者か→勘違いした馬鹿者である。」
be mistaken思い違いをしている
mistaken〜:誤解した〜
fool馬鹿もの
exclaim叫ぶ
・ " had it not been for these wretched horns my legs would have saved my life."
if it were not for 〜がなければ
倒置によるif省略が起こった普通の仮定法
wretched 〜:むかつく〜

和訳
透明な池で水を飲んでいた雄鹿は、大きく広がるの自分の見事な角に見とれていたが、同時に、自分の細く、不格好な足に非常に不快感を抱いていた。"枝分かれした角が何と美しいことか!」と彼は言った。"私の額には何と優雅な角が生えていることでしょう!」と彼は言いました。"なんと心地よい雰囲気を私の顔にもたらしてくれることでしょう"でも細長い脚に関して言えば、私は心から恥じている" 彼がその言葉を口に出すか出さないかのうちに彼の視界に、こちらに向かってくる猟師と猟犬の群れが入ってきた。先程軽蔑されていた彼の足はすぐに彼を追手から遠ざけたが、森に入ると同時に彼の角はあらゆる箇所で絡み、犬はすぐに彼に追いついて彼の生命を終わらせた。"私は思い違いをした愚か者であった!"と彼は叫んだ、 "もしこの忌々しい角がなかったならば、この足は私の命を救っていただろう"
0478英文多読者 ◆iiDSZQODZc
垢版 |
2021/03/04(木) 06:12:33.25ID:bBtImSs2
第5話 THE COCK AND THE JEWEL.

A BRISK young Cock scratching for something with which to entertain his favourite hens, happened to turn up a jewel. Feeling quite sure that it was something precious, but not knowing well what to do with it, he addressed it with an air of affected wisdom as follows :-" You are a very fine thing, no doubt, but you are not at all to my taste. For my part, I would rather have one grain of dear delicious barley than all the jewels in the world."
語法・構文
・A BRISK young Cock scratching for something with which to entertain his favourite hens, happened to turn up a jewel.
brisbきびきびとした
a cock雄鶏
scratchひっかく
for Oを求めて
something with which to は不定詞関係節
entertain Oをもてなす、楽しませる
hen雌鶏
happend to Vb:ぐうぜんVする
turn A upを発見する
・不定詞関係節について
まず関係代名詞について確認する。よく、関係代名詞節が不完全文を導く、というがあまり正確ではない。
This is the office which he works in.(α英文法より)はhe works in a officeのofficeがhe works in whichのようにwhichとなりそれが前に出てwhich he works in となったもの。関係代名詞は接続詞兼代名詞であり、節内では主語や目的語の欠落はない。わざわざwhcihなどを無視してそれよりあとだけを見れば不完全ではあるがWhat do you want?のdo you wantだけを見て不完全とは言わないだろう。欠落しているのではなく単に移動しており、痕跡としてφの位置がわかればよい。(まぁ接触節は欠けていると言っていいのかな)
関係詞節は節として完全に要素を備えてる。
次に不定詞節の形容詞的用法を確認する。
VO関係での修飾を見てみる。
for something to entertain his favorite hens with
関係詞節の場合と違い、不定詞節はwithの目的語を節内に備えておらず、不完全である。これが関係詞節と不定詞節との大きな違いである。不定詞節では、目的語はその節の外側に存在しており、目的語であるsomethingはそれだけで別の機能、今回はfor の目的語という機能を果たしている。
関係詞にもどる
This is the office which he works in.ではwhichを前に移動させていたが前置詞句の塊、つまりin which の状態で前に出すこともできる。
This is the office in which he works.
これは規範文法に則り、前置詞+空所という形で文を終わらせないためというのがメジャーな理由である。このとき関係詞節はinからもう始まっている。
規範文法に則るのであれば、不定詞節でも前置詞を前に移動させる必要が出てくる。しかし、先程の"something to entertain his favorite hens with"で前置詞の移動を行おうとしても前置詞の置き場所がない。somethingは既にscratchの目的語として機能しているためwith somethingにはできないからである。その結果、関係詞の助けを借りる必要が生じwhichが突然現れる。
something with which to entertain his favorite hens
前置詞+関係代名詞ではその後のSVを不定詞節に変えることができると覚えておけばよいし、通常通り後ろに前置詞+関係代名詞を代入するだけで意味は把握できるため、関係代名詞の後ろにSVではなく不定詞節を取ることがあるということにさえ注意すればよい。
0479英文多読者 ◆iiDSZQODZc
垢版 |
2021/03/04(木) 06:12:38.12ID:bBtImSs2
・Feeling quite sure that it was something precious, but not knowing well what to do with it, he addressed it with an air of affected wisdom as follows
ここでは分詞構文の並列から始まっている
address Oに話しかける
affect Oを装う
wisdomより例文
affect great interest とても興味があるふりをする
知恵があるように装う→装われた知恵→知恵があるように装った雰囲気で

・:-" You are a very fine thing, no doubt, but you are not at all to my taste. For my part, I would rather have one grain of dear delicious barley than all the jewels in the world."
fine上等な
no doubt確かに(副詞句)
be to one's taste:〜の好みに合っている
for one's part:〜としては
would rather V1 than V2 ≒ would sooner V1 than V2≒ would as soon V1 (as V2)するよりV1したい
どれもwouldの代わりにhadが使われることもある
否定に関してはV1の直前にnotを置けば良い
thanのあとには通例原形不定詞であるが、to不定詞やVingもないわけではない。例えばV1が進行形であればV2はVingが好まれる。
ここでのratherはsoonerの意味
soonはここでは時間的順序ではなく願望の順序、つまり優先順位を表している
この構文では今回のように単に優先順位を表したものに加え、V1にdieのような明らかに望ましくないものを置き、V2を暗に否定する修辞的なものがある。(いつも北村先生のツイートをもとに書いてるものが多いけど、ここほぼそのままツイートからです)
thanのあとにはhaveが省略されている
dear愛しの
barley大麦
allに関しては名詞を修飾する場合
All X/ All the X/ All of the Xなどが考えられる。
可算名詞であれば複数形

和訳
活発な若い雄鶏が、お気に入りの雌鶏をもてなすためのものを探してあたりを引っ掻いていると、たまたま宝石を見つけました。貴重なものだと確信していたが、それをどうすればいいのかよくわからなかったので、彼は利口ぶった雰囲気で、それに対して次のように言いました。"私としては 世界中の宝石より愛しの、おいしい大麦一粒の方がいい"

かばん屋先生と北村先生に感謝
0480英文多読者 ◆iiDSZQODZc
垢版 |
2021/03/04(木) 06:47:57.63ID:bBtImSs2
第6話 THE WOLF AND THE LAMB.

A HUNGRY Wolf one day saw a Lamb drinking at a stream, and wished to frame some plausible excuse for making him his prey. "What do you mean by muddling the water I am going to drink ?" fiercely said he to the Lamb. " Pray forgive me," meekly answered the Lamb; "I should be sorry in any way to displease you, but as the stream runs from you towards me, you will see that such cannot be the case." "That's all very well," said the Wolf; "but you know you spoke ill of me behind my back a year ago." " May, believe me," replied the Lamb, "I was not then born." " It must have been your brother then," growled the Wolf. " It cannot have been, for I never had any," answered the Lamb. "I know it was one of your lot," rejoined the Wolf, "so make no more such idle excuses." He then seized the poor Lamb, carried him off to the woods, and ate him.
構文・語法
・A HUNGRY Wolf one day saw a Lamb drinking at a stream, and wished to frame some plausible excuse for making him his prey.
a lamb子羊 ラム肉食べたい
★frameは作る、練るの意味じゃなくていいよね
frame Oを述べる
plausibleもっともらしい
prey餌食
fall prey[victim] to Oの犠牲となる
・"What do you mean by muddling the water I am going to drink?" fiercely said he to the Lamb."Pray forgive me," meekly answered the Lamb;
muddle : to make turbid or muddy
Prayは古い言い方でpleaseの意味
meeklyおとなくしく、従順に
・"I should be sorry in any way to displease you, but as the stream runs from you towards me, you will see that such cannot be the case."
displease Oを不快にする
★in any wayの意味がわからない
be sorryを「謝罪する」のように動作動詞でとり、「何らかの方法で謝る」の意味?
★わからん↑
to displease youが条件を表し、shouldは仮定法のshould
wisdomより
If I had money, I should [would] be glad to go with you. お金があったら私は喜んで君と一緒に行くのですが
the caseは実情の意味
Such was the case with him. 彼についてはそういう事情だった
・"That's all very well," said the Wolf; "but you know you spoke ill of me behind my back a year ago." " May, believe me," replied the Lamb, "I was not then born." " It must have been your brother then," growled the Wolf. " It cannot have been, for I never had any," answered the Lamb.
That's all very well, but...「それは大いに結構だが…」
speak ill of Oの悪口を言う
behind A's back:Aのいないところで
★このmayの意味はわからない
祈願のmayで主語が省略されているのか?
★わからん
wisdomより祈願のmayの例文
May you both have long and happy lives! あなたたち2人が長く幸せに暮らしますように
・"I know it was one of your lot," rejoined the Wolf, "so make no more such idle excuses." He then seized the poor Lamb, carried him off to the woods, and ate him.
one's lot:〜の仲間
lot:a group of people
Cambridge Dictionaryより例文
My lot (= children and family generally) won't eat spinach.
rejoin言い返す
idle無益な
seize Oを掴む
poorかわいそうな
carry A offの命を奪い去る
という意味もあるが今回は違う
carry A offを攫う 
to woods森へ
0481英文多読者 ◆iiDSZQODZc
垢版 |
2021/03/04(木) 06:48:01.33ID:bBtImSs2
和訳
飢えたオオカミがある日、小川で水を飲んでいる子羊を見て、彼を獲物にするためのもっともらしい言い訳をしたいと思いました。"私が飲もうとしている水を濁すとはどういうことだ?"とオオカミは猛然と子羊に言いました。"どうか許してください"子羊は大人しく言いました。"私はあなたを不愉快にさせたならばどのような方法を持ってしても謝りますが、小川はあなたから私に向かって流れているので、そのようなことはあり得ないことがわかるでしょう。"それは大いに結構 "狼は言った。"しかし、ほら、1年前にお前が私の陰口を言っただろう" "どうか信じてください" 子羊は答えた "私はその時生まれていませんでした" "であるならばお前の兄弟だったに違いない" と狼は唸った "そんなはずはないです。私には一人も兄弟はいません"子羊は答えた "お前の仲間であることは分かっている"狼は言い返した "だから、これ以上そんな無駄な言い訳をするな" 彼はそれから哀れな子羊を捕まえて 森に連れて行き 食べました
0482英文多読者 ◆iiDSZQODZc
垢版 |
2021/03/04(木) 07:14:43.71ID:bBtImSs2
途中でstop
あとでつづき

第7話 THE PEACOCK'S COMPLAINT.

THE Peacock complained to Juno that while every one laughed at his voice, an insignificant creature like the Nightingale had a note that delighted everybody. Juno, angry at the unreasonableness of her favourite bird, scolded him in the following terms : "Envious bird that you are, I am sure you have no cause to complain. On your neck shine all the colours of the rainbow, and your extended tail shows like a mass of gems. No living being has every good thing to its own share. The falcon is endowed with swiftness, the eagle, strength, the parrot, speech, the raven, the gift of augury, and the nightingale with a melodious note, while you have both size and beauty. Cease then to complain, or the gifts you have shall be taken away."
構文・語法
・THE Peacock complained to Juno that while every one laughed at his voice, an insignificant creature like the Nightingale had a note that delighted everybody.
whileは対比を表す接続詞
insignificant取るに足らない
note鳴き声
Junoユノ、Jupiterの妻
delight Oを喜ばせる
・Juno, angry at the unreasonableness of her favourite bird, scolded him in the following terms :
angry…の部分は分詞構文のbeingが省略されていると考えれば良い
unreasonable理不尽な
unreasonableness理不尽なこと
terms言い方、言葉遣い
・"Envious bird that you are, I am sure you have no cause to complain. On your neck shine all the colours of the rainbow, and your extended tail shows like a mass of gems.
envious嫉妬深い
C that SV「SVCであるので(理由)SVCだけれども(譲歩)」
譲歩や理由を表す構文であり、名詞の場合は無冠詞で用いる。似たような構文としてX though SVとX as SVがある。Xには名詞や形容詞の他に動詞なども考えられる。
譲歩のalthoughとthoughのうちthoughは文学的表現として次のようになることもある。
Fail though I did, I would not give up the plan. (私は失敗したが、その計画をあきらめない)
譲歩を表すas節では義務的にX as SVとなる。ここでも名詞を使う場合は無冠詞にする。
Freezing as it was, we went out.
=Freezing though it was, we went out. =Though it was freezing, we went out.
X as SVに関しては理由を表す場合もある
Freezing as it was, we entered the house. =Because it was freezing, we entered the house.
(凍えるような寒さだったので、私たちは家に入った)
米では as C as SVともなりうる
As freezing as it was, we went out.
As greatly as I admire him, his new paper is rather unsatisfactory.
X as SVに関しては
being as C as SV 「現実としてSが、
このようにCである状況で」
doing as 副詞 as SV「現実としてS
が、このように副詞にvする状況で」
が根底にあるため、理由にも譲歩にもなりうる。
X as SVは省略、X though SVは前置によって生まれたわけである。
また、although as C as SVという形もありえる。
cause(正当な)理由
On your neck shine all the colours of the rainbowはMVSの倒置
show(〜に)見える
a mass of O:大量のO
a gem宝石
ーーーーーここまで確認しましたーーーーー
・No living being has every good thing to its own share. The falcon is endowed with swiftness, the eagle, strength, the parrot, speech, the raven, the gift of augury, and the nightingale with a melodious note, while you have both size and beauty. Cease then to complain, or the gifts you have shall be taken away."
★share分け前、割り当
wisdomにあるhave A to oneselfみたいな感じかな
falcon鷹
swiftnessすばやさ
ここでのspeechは「言語能力」のこと
ravenオオガラス、ワタリガラスaugury予言giftは才能でいいかな
augury raven ってカードがMTGにあるみたい。
占い鴉と訳されていて能力は予顕
melodious旋律の美しい、心地よい旋律の
whileは対比
cease to Vbするのをやめる
thenは「従って、それゆえ」の意味
0485英文多読者 ◆iiDSZQODZc
垢版 |
2021/03/04(木) 21:56:41.35ID:Tn7MmMRb
西先生がYouTubeでポレポレを解説するらしい
なかもんと仲良かったはずだしコラボしないかなぁ
0486英文多読者 ◆iiDSZQODZc
垢版 |
2021/03/04(木) 22:41:00.01ID:Tn7MmMRb
最後までやりましましたバージョン

構文・語法
・THE Peacock complained to Juno that while every one laughed at his voice, an insignificant creature like the Nightingale had a note that delighted everybody.
whileは対比を表す接続詞
insignificant取るに足らない
note鳴き声
Junoユノ、Jupiterの妻
delight Oを喜ばせる
・Juno, angry at the unreasonableness of her favourite bird, scolded him in the following terms :
angry…の部分は分詞構文のbeingが省略されていると考えれば良い
unreasonable理不尽な
unreasonableness理不尽なこと
terms言い方、言葉遣い
・"Envious bird that you are, I am sure you have no cause to complain. On your neck shine all the colours of the rainbow, and your extended tail shows like a mass of gems.
envious嫉妬深い
C that SV「SVCであるので(理由)SVCだけれども(譲歩)」
譲歩や理由を表す構文であり、名詞の場合は無冠詞で用いる。似たような構文としてX though SVとX as SVがある。Xには名詞や形容詞の他に動詞なども考えられる。
譲歩のalthoughとthoughのうちthoughは文学的表現として次のようになることもある。
Fail though I did, I would not give up the plan. (私は失敗したが、その計画をあきらめない)
譲歩を表すas節では義務的にX as SVとなる。ここでも名詞を使う場合は無冠詞にする。
Freezing as it was, we went out.
=Freezing though it was, we went out. =Though it was freezing, we went out.
X as SVに関しては理由を表す場合もある
Freezing as it was, we entered the house. =Because it was freezing, we entered the house.
(凍えるような寒さだったので、私たちは家に入った)
米では as C as SVともなりうる
As freezing as it was, we went out.
As greatly as I admire him, his new paper is rather unsatisfactory.
X as SVに関しては
being as C as SV 「現実としてSが、
このようにCである状況で」
doing as 副詞 as SV「現実としてS
が、このように副詞にvする状況で」
が根底にあるため、理由にも譲歩にもなりうる。
X as SVは省略、X though SVは前置によって生まれたわけである。
また、although as C as SVという形もありえる。
★でも譲歩というのが少しわからない。意味的に譲歩じゃない気がする。わからん。
cause(正当な)理由
On your neck shine all the colours of the rainbowはMVSの倒置
show(〜に)見える
a mass of O:大量のO
a gem宝石
・No living being has every good thing to its own share. The falcon is endowed with swiftness, the eagle, strength, the parrot, speech, the raven, the gift of augury, and the nightingale with a melodious note, while you have both size and beauty. Cease then to complain, or the gifts you have shall be taken away."
being生き物、存在
for good (and all)これから先、永久にいつまでも
★has every good thing to its own shareの語法はよくわからないが次が参考になる
fall to A's share Aの分け前になる
share分け前、割り当
falcon鷹
swiftnessすばやさ ここでのspeechは「言語能力」のこと ravenオオガラス、ワタリガラス augury予言
ここはくびき語法(zeugma)であり空所化が発生している。他はis endowed withを補えばよいだけなのに対して、nightingaleのところではwithが存在しているのでis endowedだけ補う。
melodious旋律の美しい、心地よい旋律の
whileは対比を表す接続詞
cease to Vbするのをやめる
thenは「従って、それゆえ」の意味
orは「さもなければ」という意味で命令や要請を表す表現のあとに「好ましくないこと」を持ってくる
or elseはより強意的な表現
take A awayを奪う
shall「〜させよう/してやろう」の意味の助動詞
0487英文多読者 ◆iiDSZQODZc
垢版 |
2021/03/05(金) 04:43:55.67ID:Srdq8KDQ
hardly‥.whenは納得しにくいけどThe building work has hardly begun.にあるような「少し前」それをもっと極端に「一瞬、前に」の意味で捉えて納得している
実際Longmanは4....したばかりでの項目にhardly whenがある

She had hardly sat down when the phone rang.ならば

Xの一瞬前に彼女は座った
基準時のXはbeforeやwhenで明示
X:携帯がなる

彼女が座り込むやいなや携帯がなった

という認識

Hardly had he returned to London than he was again in trouble.みたいに倒置できるから否定語ではあるのだろうけど

no soonerV1thanV2
これは納得しやすい
0488英文多読者 ◆iiDSZQODZc
垢版 |
2021/03/05(金) 04:54:24.39ID:Srdq8KDQ
(1人称主語)助動詞would で、控えめ(話者の希望考えを述べる用法
…したい
0489英文多読者 ◆iiDSZQODZc
垢版 |
2021/03/05(金) 05:46:00.36ID:Srdq8KDQ
微修正

第8話 THE CAT AND THE MICE.

A CERTAIN house was much infested by Mice; the owner brought home a Cat, a famous mouser, who soon made such havoc among the little folk, that those who remained resolved they would never leave the upper shelves.
infest Oにはびこる
be infested with Oが群がっている(今回はbyになっている)
a mouserネズミを捕る動物
make Oを生じさせる
havoc大混乱、大損害
such 〜 that …とても〜なので…
★folk:(archaic): a group of kindred tribes forming a nation「種族」
この意味でいいかな
resolve that SVしようと決意する
resolution決意
would:意志のwillが時制の一致でwouldになっている
The Cat grew hungry and thin in consequence, and, driven to her wit's end, hung by her hind legs to a peg in the wall, and pretended to be dead. An old Mouse came to the edge of the shelf, and, seeing through the deception, cried out, " Ah, ah, Mrs. Pussy! We should not come near you, even if your skin were stuffed with straw."
driven...は分詞構文
wit機知
be at wit's end「途方に暮れて」:to be so worried, confused, or annoyed that you do not know what to do next
bring someone to his wit's end:〜を困らせる手こずらせる
この表現におけるbring がdriveになっている
hung to Oに掴まる
the hind legs後ろ足
a pegフック
pretend to Vのふりをする
see through Oを見抜く
deception欺瞞
pussycat≒pussy≒ネコちゃん(小児)
pussy腰抜け
仮定法のshouldでwouldと同様の働きをする
be stuffed いっぱいである、詰められている
straw麦わら
0490英文多読者 ◆iiDSZQODZc
垢版 |
2021/03/05(金) 05:55:59.97ID:Srdq8KDQ
微修正
第9話
THE DOG AND HIS SHADOW.

A DOG, bearing in his mouth a piece of meat that he had stolen, was crossing a smooth stream by means of a plank. Looking in, he saw what he took to be another dog carrying another piece of meat. Snapping greedily to get this as well, he let go the meat that he had, and lost it in the stream.
bear Oを運ぶ in his mouthという前置詞句がbearingの目的語であるa piece of meat that he had stolenの前に前置された形、おそらく目的語が長いため。
smooth波立たない
a smooth sea穏やかな海
by means of Oを用いて
a plank板
look in覗く
take O to be Cと見なす
he saw what he took to be another dog carrying another piece of meat.

he saw (O:what he took φ to be another dog) (C:carrying another piece of meat)
彼が別の犬と見なしたものが別の肉の欠片を抱えているのを見た
greedily貪欲に
snapかみつくパッと動く
as well…もまた
またlet go ofのofがない
let O Vbの目的語後置かな
let A goを手放す
0491英文多読者 ◆iiDSZQODZc
垢版 |
2021/03/05(金) 08:49:12.57ID:P0mHjMrd
第10話 THE ANT AND THE FLY.

An Ant and a Fly one day disputed as to their respective merits. " Vile creeping insect !" said the Fly to the Ant, " can you for a moment compare yourself with me? I soar on the wing like a bird. I enter the palaces of kings, and alight on the heads of princes, nay, of emperors, and only quit them to adorn the yet more attractive brow of beauty. Besides, I visit the altars of the gods. Not a sacrifice is offered but is first tasted by me. Every feast, too, is open to me.

・An Ant and a Fly one day disputed as to their respective merits. " Vile creeping insect !" said the Fly to the Ant, " can you for a moment compare yourself with me?
dispute言い争う
as to Oに関して
merits長所、美点 通例複数形
ちなみに
virtue長所、利点、美点
by[in] virtue of Oのおかげで
by virtue of whichやby virtue whereofなどの関係詞が絡むことも
one's respective〜:それぞれの〜
vile下劣な
creep地を這う
似た表現に
crawl這う、ゆっくり進む、クロールで泳ぐ
地を這うドラゴンの英語版
https://i.imgur.com/nppr4Wa.jpg
soar滑空する、上昇する
soar on the wing
翼で空高く羽ばたく
手段を表すonかな
on the wing飛行中で
palace宮殿(しばしばPalace)
alight on Oに降りる
nayいや、むしろそれどころか

It is my pleasure, nay (my) privilege, to introduce tonight's guest speaker.
quit Oを立ち退く
adorn Oを飾る、引き立てる
★ここは「単にそこから離れるだけで(その結果)より一層魅力を引き立てる」ってことでいいのかな
themはheadsのこと
yet -erでより一層〜
brow: forehead
of beautyはof 抽象名詞でbeautifulの意味でいいかな
altar祭壇、聖餐台
sacrifice生贄供物
feastごちそう
★Not a sacrifice is offered but is first tasted by me.
倒置が起こってないし、notは主語に掛かってるよね
not aはnot anyみたいな感じの意味になることがあるし
No sacrifice is offered to me but a sacrifice is first tasted by me.
「一つも私には供物が捧げられていないが、まず私が供物を味わうのだ。」ってこと?
0492英文多読者 ◆iiDSZQODZc
垢版 |
2021/03/05(金) 09:01:50.61ID:P0mHjMrd
補うとしたら
No sacrifice to me is offered but a sacrifice to the gods is first tasted by me.
「私への供物は一つも捧げられていない
が、私が最初に神々への供物を味わ
うのだ。」か
0493英文多読者 ◆iiDSZQODZc
垢版 |
2021/03/05(金) 10:45:21.23ID:P0mHjMrd
Twitterで教えていただきました
疑似関係詞ですね
「私が最初に味わわずに捧げられる
供物は一つもない。」
0494英文多読者 ◆iiDSZQODZc
垢版 |
2021/03/05(金) 12:17:25.66ID:P0mHjMrd
・I eat and drink of the best, instead of living for days on two or three grains of corn as you do."
eat and drink of the bestのofは「部分のof」であり、partなどを補えばよいhttps://twitter.com/41isyoichi/status/1152488992723492864?s=19
days on two...のonは「支えのon」であり「〜を食べて」の意味
live on vegetables 野菜を常食とする
as you doは「(実際に)あなたがするように」の意味であり、分詞構文に入り込むas S doと同じだと思う
Living as I do in the suburbs, I have to commute a long distance.(読むための英文法より)
・listen to me. "All that's very fine," replied the Ant; " but You boast of your feasting, but you know that your diet is not always so choice, and you are sometimes forced to eat what nothing should induce me to touch.
boast of Oを自慢する(二重母音)
diet食事
★choiceは形容詞として「極上の」の意味があるが、「名詞の前に置く」とwisdomにはある。Oxfordの辞書にも[only before noun]とある。叙述の形容詞として使えるならば悩まずに済むのだが。
sometimes時々
induce O to Vするように仕向ける
★このshouldは仮定法のshouldでありwouldと同じように解釈して問題ないよね?
「何があっても私が触らないような物」
だとしたらこれも主語に仮定が含まれるパターンの一種?
・As for alighting on the heads of kings and emperors, you know very well that whether you pitch on the head of an emperor, or of an ass (and it is as often on the one as the other), you are shaken off from both with impatience.
as for Oについては
pitch on Oにたどり着く
ass尻、肛門
()のitは「pitchする場所」の意味であり、一方が他方と同じ頻度ということ
でもas the otherのところas on the otherにしなくていいのかな
https://twitter.com/5chan_nel (5ch newer account)
0495英文多読者 ◆iiDSZQODZc
垢版 |
2021/03/05(金) 12:24:00.70ID:P0mHjMrd
徹夜でしんどいから後で続き
And, then, the altars of the gods,' indeed! There and everywhere else you are looked upon as nothing but a nuisance. In the winter, too, while I feed at my ease on the fruit of my toil, what more common than to see your friends dying with cold, hunger, and fatigue? I lose my time now in talking to you. Chattering will fill neither my bin nor my cupboard."
0496英文多読者 ◆iiDSZQODZc
垢版 |
2021/03/05(金) 18:29:20.15ID:O0wjDgtw
今回の英文でイソップ物語やめようかな
現代英文法講義通読したい
イソップは休憩にして、かんたんな英文だけにする
0499名無しなのに合格
垢版 |
2021/03/06(土) 11:43:09.44ID:pPB6zkjz
648 検索順位ランキング[sage] 2020/02/15(土) 18:58:56.73
ちょっとまとめてみました
The galaxy express 999 和訳 の検索結果(2020/02/15)
https://otokake.com/matome/WXxbIh
現状一位。最初のフレーズの訳は「終点にたどり着いたと思ったんだ」と文法上特に問題ありません。
基本的には映画の内容も考慮して
「一つの旅が終わったが、まだ人生という長い旅は始まったばかりだ」という雰囲気で解釈する、
映画公開当初からアニメファン・松本零士ファンの間で一般的だった解釈を踏襲しています。
「特に999ファンというわけでもない記者が書いた、ありがちな薄いまとめサイト」という印象ですが、
特に大間違いは書いていないので問題ない記事と言えるでしょう。

https://ameblo.jp/sonnykim/entry-12272548174.html
現在二位の問題のブログです。最初のフレーズは「終点にたどり着くと思った」で明白な時制違いの誤訳。
後半では「I thought I reached the endではおかしいのでreachedを過去完了に変える」という解説がありますが、
変えても「終点にたどり着くと思った」という意味にはならない為まったくの意味不明。多数の例文もツッコミどころが増えただけでした。
大変問題のある内容であると同時に、間違い方が半年ほど前のあるブログに酷似しておりパクリ疑惑が持たれています。

三位 日本語版の歌詞がヒットしました。
四位 動画サイトへのリンク。訳は無い様です。
五位 Yahoo!知恵袋。日本語版英語部分のみの訳です。
0500名無しなのに合格
垢版 |
2021/03/06(土) 11:44:00.77ID:pPB6zkjz
649 検索順位ランキング[sage] 2020/02/15(土) 19:01:54.30
六位 https://utaten.com/specialArticle/index/682
訳なし。日本語版の解釈・感想です。こなれた感想でまとまっている印象です。

七位 https://songs20thcentury.hateblo.jp/entry/2019/06/05/224359
訳なし。日本語版の解釈・感想です。文脈の整理に難があり混乱している印象です。

八位 https://column-usukuti.hatenadiary.jp/entry/2016/07/18/202424
ようやく英語版の話ですが訳は断片的です。出だしは「僕は長い長い旅の終わりに着いてしまったと思っていた」
解釈は一位と同じ一般的・伝統的タイプで、どう見ても正常な999ファンです。

九位 https://www.pixiv.net/artworks/60338349
本来絵を投稿するところであるピクシブから謎の刺客が9位にエントリー。一番と二番に別の訳をあてるという荒技を披露し、
歌い出しはなんと「僕が考え事をしていると、最後にたどり着くのは 遠い、遠い旅の果ての事...」
と言っても完全なデタラメではなく、英単語に複数の意味を持つものがある事を利用して滅多に使わない方の意味だけで訳す、
逆張り式翻訳法とでも言うべき仕様となっている様です。
また、サビだけ語調を変えて誰か(メーテル?)からのメッセージを思わせる訳に仕上げたのも印象的。
一番の訳で言葉遊びに全力投球した分、二番では一般的な解釈が踏襲されます。
0501名無しなのに合格
垢版 |
2021/03/06(土) 11:44:46.26ID:pPB6zkjz
650 検索順位ランキング[sage] 2020/02/15(土) 19:04:36.96
10位 https://bellthrough.com/diary/gingatetudou999
二位のパクリ元となった疑いがあるブログが十位圏内に。訳は一応「もう長い、長い旅の終わりにたどり着いたと思ったんだ」となっていますが、後から

「『以前、終わりに着いた、とそう思った。』(中略)これだと終わりに着いて終了してしまいます」
「終わりのない旅に出ているわけですから。これは、歌詞の意味にそぐわないと思うので キチンと文法を書くのであれば 『終わりに付くものだと、ずっと思っていた。』」

と、わけのわからない理屈で「歌詞が文法的に間違っている」と主張します。
実際にはもちろん「終わりに着いたと思ったが終わっていなかった」という話に不自然な点はなく、
変なのは歌詞の文法ではなく「終わりに着いたと『思った』だけで終了」という解釈の方です。

また、「昔の僕は『終わりがあるんだろうな〜』と思い続けていた」という内容にする為に
歌詞をI thought I had reached the endにする事を提案していますが、どうでしょうか、高校生以上の皆さん、

I thought I had reached the end



「『終わりがあるんだろうな〜』と思い続けていた」

という意味になっているでしょうか?かなり簡単な問題ですね。
これは解釈や芸術性や言葉遊びとは関係ない、「テストならバツ」の純粋な文法の間違いです。
英語講師を名乗る べるさんの実力は、実際の記事を見る限り「高校二年生への進学が危ぶまれる程度の能力」です。
非常に問題がある記事と判断できます。
0502名無しなのに合格
垢版 |
2021/03/06(土) 11:46:49.49ID:pPB6zkjz
>>1 >>498-501

652 名無しさん@英語勉強中 2020/02/08(土) 20:23:18
というか、銀河鉄道999って目的地の終着駅までちゃんと着く話だから。
そこにある予定の永遠の命が詐欺だっただけで。

だから「終点に着いた時はこれでハッピーエンドだと思ったんだけどな…まあ、また旅に出るのもいいか!」って歌だと考えると、
そもそも過去完了で大過去にする必要ないのよ。
着いたと同時に思う方が自然だから。

502 Name_Not_Found 2020/02/13(木) 14:14:07
一般的というか、旧来の松本零士ファンの間では、本編の内容と絡めて

アンドロメダに着いた時は、これで旅も終わるかと思ったが

一回の旅で世の中のすべてを見た訳ではないと気づき

俺達の、人生という終わらない旅はこれからだ!(完)

という様な解釈が普通だと思う。この歌に関して以前調べたのなんて10年以上前だと思うが、
当時は「終わりに着くと思った」なんて訳す人はネット上でも見かけなかった。

467 愛蔵版名無しさん 2020/02/13(木) 01:49:39
999の歌だから999で例えると
鉄郎君は目的地のアンドロメダの駅に着いた時に思いました
「やった!今、旅の終わりに着いたぞ!」

こういう「着いた」と「思った」のタイミングが一致するシチュは両方過去形の方が普通

469 愛蔵版名無しさん 2020/02/14(金) 15:24:11
まあ映画のエンディングとして聞けば大体そういう意味だと思うんじゃないか
0503名無しなのに合格
垢版 |
2021/03/06(土) 12:26:48.89ID:1p3semXM
「つくんだろうな〜(予測)」だから過去完了を使え、というのは確かに不思議すぎる理屈。
0505名無しなのに合格
垢版 |
2021/03/06(土) 21:09:41.59ID:VNzQTpXU
>>504
とてもためになるので、できれば続けて欲しいです…
0506英文多読者 ◆iiDSZQODZc
垢版 |
2021/03/06(土) 21:34:28.24ID:JjwDv0bU
>>505
誤訳、誤読が多すぎるので現代英文法講義を読んで修行してきます
あと一週間で読み切りますので、そしたらたぶん再開します
例えば>>477のmistaken fool that i wasとかは感嘆文的なやつですね(まだこの事項についてよくわかっとらんけど)
0507英文多読者 ◆iiDSZQODZc
垢版 |
2021/03/06(土) 21:36:30.77ID:JjwDv0bU
ていうかSOP'S FABLES以外に著作権フリーのやつってないかな
TED-EDとかkurzgesagtのスクリプトを勝手に使っていいなら嬉しいのだけど
■ このスレッドは過去ログ倉庫に格納されています

ニューススポーツなんでも実況